Jump to content

Wikipedia:Reference desk/Science

From Wikipedia, the free encyclopedia

This is an old revision of this page, as edited by 128.239.177.28 (talk) at 13:26, 15 September 2008 (→‎Help identifying flowers). The present address (URL) is a permanent link to this revision, which may differ significantly from the current revision.

Welcome to the science section
of the Wikipedia reference desk.
Select a section:
Want a faster answer?

Main page: Help searching Wikipedia

   

How can I get my question answered?

  • Select the section of the desk that best fits the general topic of your question (see the navigation column to the right).
  • Post your question to only one section, providing a short header that gives the topic of your question.
  • Type '~~~~' (that is, four tilde characters) at the end – this signs and dates your contribution so we know who wrote what and when.
  • Don't post personal contact information – it will be removed. Any answers will be provided here.
  • Please be as specific as possible, and include all relevant context – the usefulness of answers may depend on the context.
  • Note:
    • We don't answer (and may remove) questions that require medical diagnosis or legal advice.
    • We don't answer requests for opinions, predictions or debate.
    • We don't do your homework for you, though we'll help you past the stuck point.
    • We don't conduct original research or provide a free source of ideas, but we'll help you find information you need.



How do I answer a question?

Main page: Wikipedia:Reference desk/Guidelines

  • The best answers address the question directly, and back up facts with wikilinks and links to sources. Do not edit others' comments and do not give any medical or legal advice.
See also:


September 9

HIV taxonomy

To which kingdom does it belong? I have been able to find the genus and family but not the kingdom. Not even on Virus Taxonomy Online or the ICTVdb Index of Viruses

Thanks in advance!

Sorry for signing with an IP200.63.228.51 (talk) 01:19, 9 September 2008 (UTC)[reply]

Viruses aren't classified in Kingdoms. Which is why you can't find one! - Nunh-huh 03:56, 9 September 2008 (UTC)[reply]
So, I get the 'how' but not the 'why'. What's the issue with designating a separate virus kingdom? --Kurt Shaped Box (talk) 06:21, 9 September 2008 (UTC)[reply]
(edit conflict)The ICTV system starts at Order and goes down from there. See Virus_classification#ICTV_classification.--Scray (talk) 04:00, 9 September 2008 (UTC)[reply]
A lot of biologists would dispute that viruses are even life-forms. It's a thin line between complicated chemical agents and actual life. SteveBaker (talk) 04:45, 9 September 2008 (UTC)[reply]
Does anyone who's anyone still believe that prions are alive? I know that there was a bit of debate about that a few years ago, when Mad Cow was in the news. --Kurt Shaped Box (talk) 06:16, 9 September 2008 (UTC)[reply]
Wikipedia has an article on this: Virus classification. Also see Kingdom (biology) for the way things have changed over time in the taxonomy of living things. -Arch dude (talk) 12:49, 9 September 2008 (UTC)[reply]
From that article and others, we can see that there is no scientific consensus on a taxonomy with a single root, so there is no virus "kingdom." For the other kingdoms, it is reasonable to assume that each kingdom arose from a common ancestor and that the kingdom divided by evolution and natrual selection into finer and finer divisions, each with a common ancestor (approximately.) This is not likely for viruses. Viruses are likely to have arisen multiple times in multiple hosts. Therefore unless a bunch of viruses appear to be similar enough to suggest evolution from a common ancestor, scientists are unwilling to assign them to the same grouping. -Arch dude (talk) 14:07, 9 September 2008 (UTC)[reply]

Check this out?

I know it seems ridiculous, but could someone with some scientific background look into this? It appears to be unlikely, but I'm just wondering, does it have any scientific truth?

Official Site Ebay Page PerfectProposal 01:00, 9 September 2008 (UTC)[reply]

Only truth that springs to mind starts "A fool and his money...." ArakunemTalk 01:09, 9 September 2008 (UTC)[reply]
What is there to say about it? He never really says how he did it, or what evidence there is it works. He's just asking you to blindly hand over 25M...Someguy1221 (talk) 01:19, 9 September 2008 (UTC)[reply]
Just tell him he has to pick it up last Tuesday. Dragons flight (talk) 02:41, 9 September 2008 (UTC)[reply]
Now there's a potential scam, ripe for the exploiting. Tell the mark that you're collecting payment in cash, as arranged, for time-travel related services rendered in the future and that all will be made clear if they go check the eBay auction/website on <date in future> and make contact... --Kurt Shaped Box (talk) 06:14, 9 September 2008 (UTC)[reply]
Oooh! That's good! You wear a shiney silver costume and knock on people's doors: "Hello! I'm a time traveller from 20 years into your future. The future-you told me to come back in time and tell you to invest $10,000 in this company so that when the stock price skyrockets next week you/he will become a multi-billionaire." ...very good! SteveBaker (talk) 15:12, 9 September 2008 (UTC)[reply]
I wonder why time travel is so expensive. He doesn't even mention needing any equipment. APL (talk) 02:17, 9 September 2008 (UTC)[reply]
Beware of Those who Capitalize Lots and Lots of Words. Besides, if it worked, this person could have gone back in time, invested in the stock market (selling Fannie Mae and Freddie Mac short comes to mind) and cleaned up. Clarityfiend (talk) 02:48, 9 September 2008 (UTC)[reply]
Wow! That's a pretty bold con trick! I like the way that there is ZERO information on any of the pages. I've filed a fraudulant sales report with eBay.
He's given himself "3 to 5 years" to spend the cash and find someplace to hide from his investors. If it really worked, his future self would already have gone back in time and told himself which horses to bet on to get the cash to develop it...and probably given himself the plans and the dilithium crystals to make the work take less time. I guess that's how we know it's not real.
SteveBaker (talk) 04:43, 9 September 2008 (UTC)[reply]
Ahh, A a little GF wouldya? The guy might just be your common-or-garden internet nutcase - or just another attention/lulz-seeking hoaxer. ;) Seriously though, if that auction stays open, it will attract bids from people with no intention of paying (that's not intended as an incitement to do that, seriously, guys!) - these things always do. I don't know offhand what the Final Value Fee on $25million will be - but it won't be small... --Kurt Shaped Box (talk) 06:10, 9 September 2008 (UTC)[reply]
WP:AGF is a Wikipedia rule - it doesn't apply in real life. When someone is so clearly attempting to scam some gullible investor out of millions of dollars - I regard it as my responsibility as a rational human being to do something about it. In this case he's clearly breaking eBay's bylaws - so I reported him. If he breaks Wikipedia guidelines, I'll report him for that too. There aren't many rules on the Internet - but where there are, let's try to enforce them. SteveBaker (talk) 06:29, 9 September 2008 (UTC)[reply]
Didya not see the smiley, Steve? I wasn't being strictly serious... :) --Kurt Shaped Box (talk) 06:33, 9 September 2008 (UTC)[reply]
Oh come on, be fair!! He clearly does say how it works - dimensional tunneling, whatever that is. I am a little puzzled why he does not just send a completed machine back in time though. It would avoid all those tedious development costs. SpinningSpark 15:54, 9 September 2008 (UTC)[reply]
You cynics. I will have had a lovely meal at Milliways, or just got back from will having had one. --- OtherDave (talk) 21:47, 9 September 2008 (UTC)[reply]
I wonder what sort of fee eBay must have charged him for a $25,000,000 auction price. ~Amatulić (talk) 00:06, 13 September 2008 (UTC)[reply]

I have a question about Reno, Nevada

My daughter is doing a school report and needs to find out Reno's proximetry to the ocean. (removed e-mail address to protect from spambots) 75.15.211.224 (talk) 03:28, 9 September 2008 (UTC)Brandi[reply]

We aren't supposed to do people's homework - but I would suggest going to Google Maps - enter "Reno, Nevada" then back out until you see some ocean. Pick a city on the coast and ask Google to plan a route from Reno to that city. That would be the driving distance - the "as the crow flies" distance will be a bit shorter. SteveBaker (talk) 04:33, 9 September 2008 (UTC)[reply]
And to find the crow's distance easy, if you have Google Earth, you can click on the ruler button to drag a line between two points, and the program will tell you the distance you've just drawn across. Someguy1221 (talk) 05:02, 9 September 2008 (UTC)[reply]
(When you look at the map - it's pretty clear what the nearest city is (hint: it has a really pretty orange bridge) - and that driving route is pretty direct...in this case). SteveBaker (talk) 05:28, 9 September 2008 (UTC) [reply]
Or use this excellent tool. Bazza (talk) 13:02, 9 September 2008 (UTC)[reply]
You'll get extra points for originality if you don't choose the same boring ocean as everyone else. Try the Arctic or Indian. --- OtherDave (talk) 21:50, 9 September 2008 (UTC)[reply]
In this education system? I think even a distance to the Atlantic will get you marked down. --antilivedT | C | G 00:15, 10 September 2008 (UTC)[reply]

Birds with Blue Eyes

Are there any birds (especially raptors) that have blue eyes as their population's predominant phenotype? 76.121.209.96 (talk) 05:18, 9 September 2008 (UTC)[reply]

The Blue-eyed Cockatoo, as the name implies, has blue eyes. Not a bird of prey, though. Fribbler (talk) 16:19, 9 September 2008 (UTC)[reply]
Jackdaws have particularly striking, steely-blue eyes. Not a bird of prey either though... --Kurt Shaped Box (talk) 18:57, 9 September 2008 (UTC)[reply]
Harpy eagles seem to fit the criteria. Matt Deres (talk) 19:54, 9 September 2008 (UTC)[reply]
And heres an article on the Philippine Eagle that may also fit the bill. Fribbler (talk) 22:24, 9 September 2008 (UTC)[reply]

Diet of the Canada Goose

Do Canada Geese ever eat fish? 71.113.3.76 (talk) 07:30, 9 September 2008 (UTC)[reply]

Yes, according to this. Go to the section on "diet". Zain Ebrahim (talk) 08:09, 9 September 2008 (UTC)[reply]

slip of tongue

How and why people have slip of tongue? —Preceding unsigned comment added by Adityamendel (talkcontribs) 10:51, 9 September 2008 (UTC)[reply]

I think it's generally caused by being distracted while speaking, you're thinking one thing and saying another and you end up saying something else (often related to what you were thinking). You may like to read Freudian slip. --Tango (talk) 10:59, 9 September 2008 (UTC)[reply]


Check my chemistry questions

I have the following questions and the answers Ive gotten are below each one. Tell me if they are correct:

  1. Write the full equation for sodium reacting with bromine to form sodium bromide
    • Na2 + Br2 -> 2NaBr
  2. Write the half equations for the same reaction
    • Na2 -> 2Na+ + 2e- and Br2 + 2e- -> 2Br-
  3. State which species is being oxidised and which is being reduced
    • Br2 is oxidised. Na2 is reduced.
  4. State which is the oxidant and which is the reductant
    • Na2 is the oxidant. Br2 is the reductant

I think its pretty much right but one thing Im definitely not sure of is this: in questions 3 and 4 should my answers read, for example, as I said "Br2 is oxidised" or "2Br- is oxidised"
Thanks --RMFan1 (talk) 12:26, 9 September 2008 (UTC)[reply]

You're on the right track as far as balance. Check the redox page for definitions of the various terms and roles of the reductant and oxidant. Regarding notation, something written as Xy means a single molecule (or polyatomic ion) consisting of y atoms of element X together. Writing yX means a collection of y distinct X atoms or polyatomic ions. So you have to figure out whether there is such thing as, for example, an actual molecule composed of two bromine atoms or whether bromine exists as single atoms (and therefore you'd just have two of them). You can read our bromine and sodium pages to see what their stable forms are. DMacks (talk) 14:32, 9 September 2008 (UTC)[reply]
Is it me or are the reduction and oxidation the wrong way round?59.100.192.85 (talk) 23:28, 9 September 2008 (UTC)[reply]
Redox has the defs:) I usually try to think about it in terms of the charge on each atom and whether it's going down (being "reduced") or whether at atom is doing something to cause another atom's charge to go down ("reducing" it). DMacks (talk) 01:25, 10 September 2008 (UTC)[reply]
Helpful mnemonic is OILRIG: Oxidation Is Loss (of electrons); Reduction Is Gain. Gandalf61 (talk) 13:12, 10 September 2008 (UTC)[reply]
There are two major mistakes in your homework. However the Wikipedia reference desk is not supposed to be for your homework questions. Axl ¤ [Talk] 20:12, 11 September 2008 (UTC)[reply]

alien

could the alien exist in real life, could it be a serious threat if it came out to earth, and could we kill it or would they win? Bradley10 (talk) 13:03, 9 September 2008 (UTC)[reply]

If memory serves, the alien in Alien wasn't capable of space travel, it only became a threat because the humans on the film went to its planet. I very much doubt an alien like that can exist anywhere in this solar system (except possibly Earth, but then it wouldn't be an alien!), so until we invent interstellar travel we should be fine. After that, it's anybody's guess! --Tango (talk) 13:22, 9 September 2008 (UTC)[reply]


The real Soylent green? —Preceding unsigned comment added by 71.100.10.144 (talk) 14:11, 9 September 2008 (UTC)[reply]

It's a kind of CO2 sequestration technique - except that it's really not going to work long term.
The idea is to capture the CO2 produced by some kind of nasty coal fired power station - use sunlight to cause algea to convert that CO2 into carbohydrates - then to harvest the algea and convert the carbohydrates into methanol or something - which . However, when you use the resulting fuels - the CO2 is released into the atmosphere again...so you didn't sequester it. Someone will probably argue that when you burn these new fuels, you could use the same technique to sequester that CO2 - but now you have an infinite regress because you're continuing to pump CO2 into the "system" so more and more and more algea are needed in order to sequester it. The only answer is to NOT convert the algea into any kind of a fuel at all - but instead to dump it back underground. The trouble is that the cost of doing all of this is so high that you can't afford to just dump the results back into a hole in the ground.
If you are going to invest in a "solar to ethanol" plant - then do it without the nasty power plant generating the CO2 - let the algae take the CO2 from the air - that would be a truly "carbon neutral" solution.
So - this is another effort by the "Clean Coal" people to pursuade us that they can continue to build big, polluting coal-fired power stations without killing the planet. They just hope that by confusing the science like this that the general public will fall for it...and the annoying part is that they probably will because the problem with it is pretty subtle.
SteveBaker (talk) 14:57, 9 September 2008 (UTC)[reply]
It sounds like the idea is to take advantage of the concentrations of CO2 that you get at the source in order to grow the algae more efficiently, it's not a CO2 sequestration technique (if it's being described as one somewhere, I expect that's the press not knowing what they're talking about). --Tango (talk) 15:11, 9 September 2008 (UTC)[reply]
Yes, you are right. Although the algae could be used to permanently sequester carbon by turning it into biochar, bio-oil and syngas the idea here is much more sophisticated. Some algae species produce up to 50% of their body weight in vegetable oil, which can be extracted first and even tailored to produce compounds that lend themselves better to one form of end product than another such as jet fuel versus heating oil versus salad oil. The remaining components can then be used as a dietary supplement or as cattle feed or as feed stock for biochar, etc. —Preceding unsigned comment added by 71.100.10.144 (talk) 16:28, 9 September 2008 (UTC)[reply]

Well you're not adding to the problem you're just recycling stuff that's already been released from fossil fuels. It's not a solution but you seem to insinuate that you're making it worse when you're not you're just keeping the problem the same and making some energy at the same time right? Dereconstruct (talk) 22:15, 12 September 2008 (UTC)[reply]

If it were being sold as "This is a way to make some fuel from solar power that (incidentally) has to be co-located at some nasty fossil fuel power station - but which doesn't improve the carbon footprint of fossil fuel power stations at all." - then you'd be right. However, if it's sold as "Look! We're taking all of that nasty CO2 out of the flue gasses of a "Clean Coal" power station and turning it into ENERGY!!!" - then it's a very nasty way to trick people into believing that we can still have fossil-fuelled power stations without killing the planet - when in fact this does NOTHING to help the carbon footprint of the power plant at all! If the latter is the case (and it undoubtedly is) - then this is a nasty trick that won't help the world at all - and in fact (because it'll fool those stupid politicians with their very tiny brains), it'll do us all a great deal of harm because it'll foster the idea that we can still burn megatonnes of coal every year. SteveBaker (talk) 13:55, 14 September 2008 (UTC)[reply]

Symbols

Hi! Whics symbols (Xn, T, C, N, T+ ...) are on : - Liqui Moly (alloy for fuel) - chlorine - cyanohydrin acid ? Thank you! —Preceding unsigned comment added by Atacamadesert12 (talkcontribs) 15:54, 9 September 2008 (UTC)[reply]

You generally need a specific product name, then you can use Google to search for "xxx hazard symbols". For instance, I was able to find the safety data sheet for one Liqui Moly product: [1] and the symbols are Xn, R42, S2, S23.3, S45 and S51. You can reference the meaning of the symbols here: [2]. Franamax (talk) 00:28, 10 September 2008 (UTC)[reply]

crossbreed dogs

I'm pretty darn sure it's possible to breed two different species of dogs together, to create a mixed-breed dog. Can all species of dogs mate with all other species of dogs? Or are there cases where it doesn't work? Bradley10 (talk) 15:56, 9 September 2008 (UTC)[reply]

Well, there's only one (sub)species of dog, canis lupus familiaris. All breeds should be able to interbreed, however. — Lomn 15:58, 9 September 2008 (UTC)[reply]
i would counsel against attempting to cross a Great Dane dog with a Chihuahua bitch, however. DuncanHill (talk) 16:00, 9 September 2008 (UTC)[reply]
Duncanhill, is that sort of thing possible, if inadvisable? Is it because of the potential size of offspring, or the physical difficulties of such a coupling? Not to mention the tie? Bradley10 (talk) 16:14, 9 September 2008 (UTC)[reply]
I am not a dog breeder, but I suspect that all the things you mention would conspire to make it physically impossible, though of course, as members of the same species they would theoretically be capable of producing viable offspring. DuncanHill (talk) 17:12, 9 September 2008 (UTC)[reply]
It is possible to crossbreed all dogs (assuming the male and female you use can breed). In a professional sense, any obstacles with dog size and temperament do not matter. The sperm is collected from the male and the female is artificially inseminated. Of note, wolves can be crossbred with dogs as well, but you do not get tame wolves. You get dogs that will snap and attack anyone and anything at any time. -- kainaw 18:00, 9 September 2008 (UTC)[reply]
And, of course, we have an article: Wolf-dog hybrid with a section that goes to great lengths to claim that while wolf-dogs have an erratic temperament, it is not possible to know exactly how any dog will act in all situations. Way to rationalize there! -- kainaw 18:03, 9 September 2008 (UTC)[reply]

Medicval term for 'facial flushing'

Niacin says "People taking pharmacological doses of niacin (1.5 - 6 g per day) often experience a side-effects that can include dermatological complaints such as facial flushing and itching, dry skin, skin rashes..." (my bold). What is the medical term for facial flushing? RJFJR (talk) 16:15, 9 September 2008 (UTC)[reply]

Facial erythema. But in reality "flushing" is used just as often or more often in cases when the pathology is not the skin itself. Fribbler (talk) 16:20, 9 September 2008 (UTC)[reply]
"Plethora" and "florid" also are used.--Scray (talk) 01:08, 10 September 2008 (UTC)[reply]

Oh no, no sunspots!

There haven't been any sunspots for over a month now. Is this a bad thing? 31306D696E6E69636B6D (talk) 16:42, 9 September 2008 (UTC)[reply]

Nah, we're right smack in between Cycle 23 and 24, so things are at a minimum right now. They should begin to pick up again soon, (hopefully for my DX'ing), but its not unheard of for a minumum to stretch out for a bit (see Maunder Minimum) ArakunemTalk 17:05, 9 September 2008 (UTC)[reply]
Yes, high solar activity improves skywave propagation because the increased radiation helps ionize the ionosphere that long-wavelength radio waves reflect off of (see http://www.blackcatsystems.com/propagation/solar_flux.html). On the other hand, solar flares are more frequent during the solar maximum and can cause propagation to vary wildly.
Arakunem: out of curiosity, how great of an effect does the solar cycle has on a DX'er? --Bowlhover (talk) 04:52, 10 September 2008 (UTC)[reply]
It has a huge effect, with high sunspot parts of the cycle propagation of much higher frequencies occurs even into the low VHF bands. The signals bounce off the inside of the ionosphere back to the earth, and then up and down of the earth again. Though if you want to receive low frequencies from Space say 1 MHz, or get a clear GPS signal, then now it the opportunity. Now all the shortwave stations have to pack themselves down the low frequency end where there is more interference and absorption, and more static. Graeme Bartlett (talk) 05:12, 10 September 2008 (UTC)[reply]

The article on the Maunder Minimum says that this occurred at the same time as the coldest part of the little ice age. That's what i'm worried about. 31306D696E6E69636B6D (talk) 13:10, 10 September 2008 (UTC)[reply]

Neutron stars

The first direct observation of a neutron star in visible light. The neutron star being RX J185635-3754.

The article neutron star states the radius of a neutron star is usually between 20-30 km. However, this image shows a visible neutron star. How is it possible for us to see objects this small so far away in space? (although that's an assumption, maybe it's not that far away) —Cyclonenim (talk · contribs · email) 17:06, 9 September 2008 (UTC)[reply]

It's not the size that matters, it's the luminosity. Stars are generally unresolved in astronomical images. Similarly, you can see the light from my flashlight from hundreds of yards away, even though the filament might be smaller than your eye's resolution. -- Coneslayer (talk) 17:10, 9 September 2008 (UTC)[reply]
Here's some perspective: the largest known star is thought to have a diameter similar to the orbit of Saturn, around 2.8 billion km. The neutron star you linked to is about 450 light years away, and you say 20-30 km diameter for a typical one of those. Let's pretend those two stars are the same distance away from Earth. So using my decrepit memory of trigonometry, I can compute:
   radius_canis   =  2800000000 * 1000
   radius_neutron = 20 * 1000
   distance = 450 * 3*10^8 * 365 * 24*60*60
   angle_in_sky_canis   = tan(radius_canis   / distance)   = 0.00000067         degrees
   angle_in_sky_neutron = tan(radius_neutron / distance)   = 0.0000000000000047 degrees
   angle_in_sky_pinhead = tan(.001           / 1000)       = 0.000001           degrees
So the angle that the neutron star takes up in the sky is indeed *much* smaller than the angle that the big star takes up, as you observed, but consider this: if there were a pinhead being held one kilometer up in the sky it would easily eclipse both stars. --Sean 20:08, 9 September 2008 (UTC)[reply]

You must understand that when you see an astronomical photograph with stars in it, the brighter ones will often look bigger, but this is not because the photo is showing their true diameter. It's because no matter how perfect the telescope is, you never get all the photons from the same distant point perfectly focused into the same spot on the film (or digital sensor). With a bright star the scattered photons are sufficient to expose the film and make a slightly larger image. As far as photography is concerned, practically any star is a point source -- only a few of the very largest stars have had their diameter directly measured. The diameters that you see given for stars as well as neutron stars are computed based on theory, not measured directly. --Anonymous, 21:58 UTC, September 9, 2008.

It's also possible (especially with a White Dwarf) that the light you see is being scattered off of dust and gas surrounding the actual object's planetary nebula. That would make it seem a lot bigger than it actually is. SteveBaker (talk) 23:45, 9 September 2008 (UTC)[reply]
In the case of a neutron star, you may be seeing light emitted in it's neighbourhood, generated in the same way as the radio pulse, by electrons accelerated by the spinning magnetic field. We need Sean to calculate what temperature the surface would have to be to get that luminosity, which looks to me to be around 1016 times hotter. Suggesting that the radiation may not be thermal. Graeme Bartlett (talk) 05:18, 10 September 2008 (UTC)[reply]

Core skills of a chemist

Lately I've realized I really have a sincere interest in chemistry. Been reading a little on my own, in my own pace. I've tried college before and it's just not for me, I simply can't stand the boring stuff that inevitably comes with an otherwise interesting topic. Still, I began to think if maybe distance studies - not sure if this concept exists in English, but essentially means studying from home, basically no classes to attend - would work for me. That way I could focus on one semester at a time, so if I felt fed up after one semester I could take a break and just work until I felt motivated again, instead of being on a forced 3-4 year course.

Aaaaanyway, that's just the background. My question is simply this: What are some of the core skills of an aspiring chemist, without which a person would experience difficulties going through an undergraduate level of chemistry education.

Thanks in advance for your answers, I'll be waiting eagerly. Jack Daw (talk) 18:00, 9 September 2008 (UTC)[reply]

I think the main problem you would face would be practical kills. It's not just enough to have the required knowledge, you also have to actually be able to perform your experiments in an actual lab. You have to know that that's just a whole different ballgame. Another problem of course is that the " boring stuff " is often required to understand the interesting stuff. Most chemists I talk to hate quantum physics and higher level mathematics with a passion, yet they agree that it's pretty much required knowledge. Just my two cents PvT (talk) 18:08, 9 September 2008 (UTC)[reply]
I'll second that. Not only is it the case that the "boring bits" are necessary to learn the skills - but also, if you were to become a practicing chemist, a large fraction of your working week would be spent on doing those very "boring bits". If you don't like the boring bits of learning - the odds are very good that you'll hate the job too. It's not common for college courses in the hard sciences to teach things that you won't need to both know - and use on a frequent basis. A similar issue comes about with your desire to take a break until re-motivated...doing that in a daily job would make you pretty unpopular really fast! SteveBaker (talk) 19:01, 9 September 2008 (UTC)[reply]
Additionally, these days, I think it is difficult to get a good job as a chemist without a degree. ike9898 (talk) 19:14, 9 September 2008 (UTC)[reply]
Hmm perhaps I was a bit unclear about what I meant about core skills. Obviously, even with distance studies I would get lab work, do the boring stuff, and get a degree, which I think were your combined points. What I meant with core skills was, if there are certain ways to think that are valuable, certain ways to look at things, e.g.; does a chemist have to be analytical, have a sense for details, able to see the big picture, be creative, have an excellent working memory, etc etc, the list could go on for very long.

Also, btw, it's been my experience that once you actually get out to work, even the boring parts become enjoyable, SteveBaker ;) ....... Jack Daw (talk) 19:26, 9 September 2008 (UTC)[reply]

That depends a lot on the individual - and (perhaps) the career. I don't know chemistry - but in my line of business (computer programming), it's necessary to document your software as well as design, code and debug it. I know an awful lot of programmers who love the programming bits - but utterly loathe doing documentation. Having to actually write lots of English prose is one of the "boring bits" for those people. So perhaps you don't happen to have any bits of your job that really were boring in the first place...that's the same with me - I'm a programmer who doesn't mind doing documentation in the least. My point is that if you think you just have to get through the boring bits so you can get your degree and they you won't have to worry about it anymore after that...then you're kidding yourself. SteveBaker (talk) 21:04, 9 September 2008 (UTC)[reply]
Aside from the traits you mentioned I would say that a good head for numbers is a big plus. Other helpful skills would include: good manual dexterity, good people skills (you rarely work alone), technical aptitude (you work with machines) and lots of motivation. Note that these aren't requirements, they're just handy traits to have PvT (talk) 20:04, 9 September 2008 (UTC)[reply]
(ec)Probably the biggest one would be to understand and appreciate the Scientific Method. You also won't get far unless you understand the Atomic Theory (things are made of atoms) and the nature of the chemical bond (atoms share electrons). Understanding the difference between a chemical change and a physical change is key too. After that, what you'll need to know varies based on the discipline, e.g. analytical chemistry versus organic chemistry versus biochemistry. Not only is the knowledge base different, it's been my experience that the scientists in each have slightly different mindsets. Back on a general note, to deal with practical issues one should be intimately familiar with SI (the metric system), as well as scientific notation, along with general concepts of measurement, including significant figures/propagation of uncertainty and the distinction between accuracy and precision. On a side note, learning a dimensional analysis system, such as the factor-label method, will likely save you lots of grief when doing calculations. -- 128.104.112.147 (talk) 20:12, 9 September 2008 (UTC)[reply]
(ec #2) Excellent list. These are all things that are pretty necessary for a bachelor's degree in general chemistry. Of course, there's always sepcialization for higher level degrees. For example, us theoretical/computational people need very good high-level math, and computer programming - but lab skills aren't terribly important. For analytical, good "lab hands" and attention to detail, along with a very good mechanical and scientific knowledge of what instruments you are using, but you won't use much math beyond basic calculus, if that (most of the instruments will do the math for you nowadays).
For me personally, visualization is important, with a healthy dose of memorization ability (but not necessarily a lot). Being able to visualize structures and reactions (in three dimensions), along with how to do things in a lab, is an incredibly helpful skill in almost all areas of chemistry. --Bennybp (talk) 20:23, 9 September 2008 (UTC)[reply]

Help identify this plant

I'm trying to identify a bush/shrub. I'll provide photos if we can't figure this out, but I'll try to describe the plant. It is a medium sized bush (the one I see is probably 4 to 5' all around) which I think is common in the southern United States. At least it can be found in central and south eastern Virginia. The leaves are roughly 3-4" big and a darker green on the top, lighter green on the belly. They have distinctive serrations on the entire edge of the leaf. They feel almost the same as a butter knife. The leaf, when turned sideways, looks almost like lips (elongated oval) that comes to a point at one end. The leaves are thick and waxy. This is also a flowering bush. They are not flowering now so I cannot give much more detail, but I believe they have golf ball sized buds that open up to rather large flowers with lots of pedals that turn brown and fall off. (I could take a picture of one of the brown, dead flowers). Any ideas? Anything else I could look for to help someone identify the plant, or does anyone have some ideas what this could be. Thanks.-Andrew c [talk] 18:03, 9 September 2008 (UTC)[reply]

Rhododendron perhaps? They're rather common in the area you mention... ArakunemTalk 18:09, 9 September 2008 (UTC)[reply]
That would be my guess. --Tango (talk) 18:29, 9 September 2008 (UTC)[reply]
That's a good guess, but not it :) The leaves are not quite that elongated, and the flowers are individual buds, not bunches of flowers (the rhododendron flowers look more similar to bunches of azalea flowers, where the plant I'm talking about has a single large bud and looks like something that may be on a corsage). Plus I'm not sure rhododendron has the serious serrations on the edge of the leaves. Any other ideas? Thanks for the suggestions so far.-Andrew c [talk] 18:32, 9 September 2008 (UTC)[reply]
Peony?
All the peonies I've seen (which admittedly are in the Great Lakes region) are far smaller than this plant. A pity, because the rest sounds right for a peony. Nyttend (talk) 19:43, 9 September 2008 (UTC)[reply]
Wait, "all around": what does this mean? If you mean that the circumference is four or five feet, that would be quite reasonable for the peonies I've seen. Nyttend (talk) 20:18, 9 September 2008 (UTC)[reply]
Some kind of magnolia? I'm not aware of one with serrated leaves, but ... ? --ColinFine (talk) 20:06, 9 September 2008 (UTC)[reply]
Camellia or Azalea? DuncanHill (talk) 20:52, 9 September 2008 (UTC)[reply]
DING DING DING! We have a winner. Camellia it is (or at least I'm pretty sure). Thanks!-Andrew c [talk] 00:29, 10 September 2008 (UTC)[reply]
If it's Camellia sinensis you could make tea! DuncanHill (talk) 00:58, 10 September 2008 (UTC)[reply]

Blue-yellow colourblindness

Is the colouring for the attached image likely to be a problem for blue-yellow colourblind people? I know there's a potential problem for red-green people (I myself am one), but I've done the best I can by making the blue/purple and red/green borders as short as possible. Nyttend (talk) 19:40, 9 September 2008 (UTC)[reply]

I'm not blue/yellow color blind, but at the end of the article color blindness there is a link to a website "Colorblind Web Page Filter", which allows you to simulate a webpage for different types of color blindness. Check it out yourself, but for blue/yellow blindness, your picture seems to look fine. -- 128.104.112.147 (talk) 20:36, 9 September 2008 (UTC)[reply]
Actually, there is more than one kind of red/green colorblindness (I think at least three or four). But the thing is that even if you are totally colorblind, you can still distinguish brightness. Yellow is a much brighter color than "computer-screen-primary" blue - so even utterly "every-color"-blind people who can only see in shades of grey will be able to see the difference between primary blue and primary yellow. However, if you'd chosen a brighter shade of blue - or a darker yellow - it would not have been so clear. My son is red/green colorblind - but he can easily tell the difference between red and green - even if they are at the same intensity because his particular variation is that he has a weak red sensor. His problem is between more subtle hues between red and green - and he has a lot of trouble telling red from orange or (to a lesser degree) orange from green. Our article lists at least seven different variations on color blindness - two of those are yellow/blue-related. But the colorblind web page filter is an excellent resource for the normally-sighted to get an idea of what your graphic will look like to each of the different variations. SteveBaker (talk) 20:56, 9 September 2008 (UTC)[reply]
A nice and probably easy MediaWiki hack would be a module that warns you when you upload an image with only a few colors that they're not colorblind-friendly and suggests a substitute pallet with brightness variations. --Sean 21:11, 9 September 2008 (UTC)[reply]
It looks okay with protanopia, deutanopia, or tritanopia. If you are concerned, you can test images yourself at this site. It also has some tips for building color-blind accessible web pages. Plasticup T/C 02:17, 10 September 2008 (UTC)[reply]
Thanks for the various comments: I'd not considered the brightness factor. I'd appreciate the filters, except that (as I noted above) I'm somewhat red-green colourblind (although I'd never read the article, and thus never knew that there were multiple types; I just thought it varied in intensity), so I don't know whether I'd be able to make productive use of them. Nyttend (talk) 05:28, 10 September 2008 (UTC)[reply]
And for the, ahem, rest of us? Those colors? Please. No. Saintrain (talk) 00:20, 11 September 2008 (UTC)[reply]

NOAA weather radio voice

All,

The voice of the NOAA weather radio was previously known as "Paul" (which sounds somewhat like "Mike" from MS TTS). The current one is known as "Tom." I was listening to a local stream of the NOAA All Hazards Weather Radio; who is the voice for this [3] one? The voice sounds like neither "Tom" nor "Paul" (see this [4] to listen to samples).

jdstroy (talk) 20:30, 9 September 2008 (UTC)[reply]

Weather or not

Was San Francisco overcast on August 14, 1972?

Wanderer57 (talk) 23:21, 9 September 2008 (UTC)[reply]

The more important question is where such information may be looked up. Wanderer57 (talk) 23:40, 9 September 2008 (UTC)[reply]

Such records certainly seem to exist [5], but it looks like you would have to go to Washington to look them up, they don't seem to be on-line. [6] Franamax (talk) 00:02, 10 September 2008 (UTC)[reply]
That's so cool! It's there! And all I had to do was "san+francisco"+weather GOOGLE it! Saintrain (talk) 00:04, 10 September 2008 (UTC)[reply]
Specifically, here. Saintrain (talk) 00:07, 10 September 2008 (UTC)[reply]
Dang! I forgot about weatherunderground. It doesn't say if it was overcast though ;) Can you infer that from the visibility? Franamax (talk) 00:36, 10 September 2008 (UTC)[reply]
(Sheepishly) Yes, but also from the temperature, humidity and steady 15 kt wind and it was the middle of August. But you know what Mark Twain didn't say?. (I do remember his story about the S.F drunk who froze to death under a blooming rose, though, don't I?) Saintrain (talk) 01:50, 10 September 2008 (UTC)[reply]
I clearly remember that a few years ago, I somehow missed an occultation I was planning to time and later tried to determine whether I could have seen it. I found a site that listed the temperature, cloud cover, and other conditions for every hour. I can't remember how far back the archives extended or whether they covered any city except my own (the Greater Toronto Area). I also can't seem to find it again. --Bowlhover (talk) 04:20, 10 September 2008 (UTC)[reply]
Ah, another datum for my planned great-identity-stealing machine, Bowlhover is a Hogtowner! Please tell, haven't you just had a record-setting amount of rainfall in my dear departed province? Franamax (talk) 06:15, 10 September 2008 (UTC)[reply]
Thanks. The METARS information link available at the bottom of the weather underground page gives the sky condition as scattered cloud and broken cloud, but not overcast.
Wanderer57 (talk) 13:15, 10 September 2008 (UTC)[reply]
PS - Is there any weather underground? Very high pressure and overcast would seem to be the normal condition.
Warm, humid, occasional drizzle. DuncanHill (talk) 14:56, 10 September 2008 (UTC)[reply]
So what's so special about San Francisco on August 14, 1972? Dragons flight (talk) 14:59, 10 September 2008 (UTC)[reply]
I don't know. I asked about the weather on that day in SF because of another editor asking what phase of the moon was seen in San Francisco that day. Wanderer57 (talk) 15:15, 10 September 2008 (UTC)[reply]
It's possible that it's the day the person was born/married/parents married/whatever. I can imagine the phase of the moon will mean something to those who believe in astrology. I've never heard of anyone caring about what the weather was like though Nil Einne (talk) 11:42, 12 September 2008 (UTC)[reply]

Global Warming

I've been thinking: wouldn't inducing a rapid growth of algae or something similar in the world's oceans (by putting large amounts or iron or something similar) make considerable progress in reducing the carbon dioxide content or the atmosphere? Before you ride me off, I think I remember reading on Wikipedua (I can't find the article) that a certain algae, because they were found in such large numbers, caused an Ice Age.

It's phosporous. I think algal blooms are not that appreciated.--Lenticel (talk) 23:35, 9 September 2008 (UTC)[reply]
The problem is that algae pretty much shut out all of the other lifeforms - so sure you'd maybe reduce CO2 - but maybe you'd also kill off all of the fish. The problem with all of these "radical" solutions is that we don't have a good enough understanding of the consequences of doing them. It's very likely that we'd make matters much worse. SteveBaker (talk) 23:39, 9 September 2008 (UTC)[reply]
It is iron that the OP's thinking of, and we have an article: Iron fertilization. -- 128.104.112.147 (talk) 23:41, 9 September 2008 (UTC)[reply]
I see, but according to the article it is still in the experimental stage. However, we could already see the effects of the human induced algal blooms.--Lenticel (talk) 23:51, 9 September 2008 (UTC)[reply]
It is experimental because we don't yet know all the effects, such as whether the carbon actually gets sequestered to the ocean floor, and what the nasty side effects might be, such as creating anoxic regions, as Steve notes. If you email me, I can send you a few papers and articles from Nature. Franamax (talk) 00:05, 10 September 2008 (UTC)[reply]
On a side note, and for your future information, and I'm not grammar naziing, but just letting you know, the phrase is "write me off", not "ride me off". --Sean 00:29, 10 September 2008 (UTC)[reply]
Oh wow, you don't know how long I've been using ride... —Preceding unsigned comment added by 65.92.231.82 (talk) 03:40, 10 September 2008 (UTC)[reply]
Also, algae can be poisonous, for example red tide or the poisonous algae found in Quebec lakes last year. ~AH1(TCU) 00:55, 14 September 2008 (UTC)[reply]

Antimatter

Hello. I'm a high school student interested in physics and have just read about the concept of antimatter. Unfortunately I got lost very quickly in the preface of the article on this subject and am having real difficulty grasping in my mind the concept of antimatter. could somebody explain it very simply to me, perhaps with a helpful illustration? 79.75.199.175 (talk) 23:54, 9 September 2008 (UTC)[reply]

Have you read antiparticle? I think we're going to need to know what it is that's confusing you particularly, otherwise we're going to end up explaining it the same way as the articles. --Tango (talk) 23:56, 9 September 2008 (UTC)[reply]
Well as i understand it antimatter is the opposite of matter, so one would assume antimatter does not have mass while matter does. However i'm not sure if particles such as photons which have no mass are considered antimatter so i might be wrong in my understanding here. If matter is "stuff" then is antimatter is "non-stuff" which also takes up space or what? 79.75.199.175 (talk) 00:23, 10 September 2008 (UTC)[reply]
Matter has many properties, and antimatter certainly doesn't have the opposite property of each of them. In particular, it has mass. --Sean 00:32, 10 September 2008 (UTC)[reply]
Photons indeed have no mass, and are not normally considered to be either matter or antimatter. Algebraist 00:34, 10 September 2008 (UTC)[reply]
Specifically, antiparticles have the same mass as their matter counterparts; they just have opposite charges. I believe charge is the only difference between matter and antimatter. --Bowlhover (talk) 04:29, 10 September 2008 (UTC)[reply]
No, other quantum numbers such as spin and colour charge are also reversed in an antiparticle. If only the (electric) charge were reversed then the antineutrino would be identical to the neutrino. Gandalf61 (talk) 10:59, 10 September 2008 (UTC)[reply]
There is no real difference between matter and antimatter overall (well, there might be if you get to really complicated stuff, but that's not important to start with), there is only a difference when you compare a particle with its antiparticle. For example, some matter is positively charged (eg. a proton) and some is negatively charged (eg. an election). Likewise, some matter is positively charged (eg. a positron [anti-electron]) and some is negatively charged (eg. an anti-proton). So, it's not that matter has one property and antimatter has the other, that only applies for a particular particle/anti-particle pair. --Tango (talk) 00:38, 10 September 2008 (UTC)[reply]
Did you mean 'antimatter' in the 3rd sentence? Wanderer57 (talk) 01:53, 10 September 2008 (UTC)[reply]
Adding onto the concept of symmetry between matter and antimatter, why the Big Bang didn't form an equal amount of the two was a mystery until CP-violation was discovered. --Bowlhover (talk) 04:29, 10 September 2008 (UTC)[reply]
Matter is a poorly defined word. It usually means anything that has mass and takes up space. By this definition, antimatter is not the opposite of matter; it's just a confusingly-named kind of matter. (And, for that matter, antiparticles are not the opposite of what we usually call particles; they're just confusingly-named kinds of particles.) --Allen (talk) 00:53, 10 September 2008 (UTC)[reply]
Perils of Modern Living
Harold P. Furth
Well up above the tropostrata
There is a region stark and stellar
Where, on a streak of anti-matter
Lived Dr. Edward Anti-Teller.
Remote from Fusion's origin,
He lived unguessed and unawares
With all his antikith and kin,
And kept macassars on his chairs.
One morning, idling by the sea,
He spied a tin of monstrous girth
That bore three letters: A. E. C.
Out stepped a visitor from Earth.
Then, shouting gladly o'er the sands,
Met two who in their alien ways
Were like as lentils. Their right hands
Clasped, and the rest was gamma rays.
SteveBaker (talk) 02:05, 10 September 2008 (UTC)[reply]
That reminds me of a haiku:
Hippopotamus
Antihippopotamus
Annihilation Algebraist 02:09, 10 September 2008 (UTC)[reply]
well, last night i went to an italian restaurant and ate antipasto followed by pasta, and there is definitely a vast quantity of energy being liberated as they meet. Gzuckier (talk) 16:41, 10 September 2008 (UTC)[reply]
You forgot to quench the reaction with a nice stinky grappa then... Franamax (talk) 03:43, 11 September 2008 (UTC)[reply]


September 10

reason for .b1 and .g1 primer names

In sequencing of DNA, reads will sometimes have .b1 and .g1 for the forward and reverse primers when producing amplicons. Does anyone know why the letter b is associated with forward and why g is associated with reverse? Thanks. --Rajah (talk) 02:39, 10 September 2008 (UTC)[reply]

No one seems to have an answer, however this is what I have found: There are primers called A1 A2 B1 B2 C1 C2 D1 D2 E1 E2 F1 F2 G1 G2, with the 2 primer for use on the alternate strand to the 1 primer. Also different primers are indicated by the same symbols by different researcher. So your primers must be in the context of some system or commercial product - can you say which one? Graeme Bartlett (talk) 21:55, 11 September 2008 (UTC)[reply]
1st column of slide 10 of http://snp.wustl.edu/snp-research/c-briggsae/Files/EMBO-Genetic-Map-Koboldt.ppt --Rajah (talk) 18:40, 13 September 2008 (UTC)[reply]

Carbon dioxide sequestration

There is a lake somewhere inside a volcano cone that sequestered CO2 coming from the volcano at the bottom of the lake do to the temperature and pressure at the bottom. A tremor or some other disturbance released the gas and its spread over the slopes and kill people and livestock. How much risk of this is there in the man-made sequestration of CO2 in ocean bottoms and oil fields and the like? —Preceding unsigned comment added by 71.100.10.144 (talk) 04:20, 10 September 2008 (UTC)[reply]

  • That was Lake Nyos, in case someone wants to read more about this. --Anonymous, 20:08 UTC, September 10, 2008.
The ocean bottom is risky, if the super carbonated water comes to the surface it will foam up in a big way. ANother risk is acidification of the water with dissolving of shells and bones of sea creatures. With underground storage, it presumably will be pumped back into a gas well that is reasonably well sealed, but as you say a rupture could release the gas again. Natural kinds of these ruptures can crack at the rate of about one kilometer per second and erupt at the surface in a special kind of explosive volcano as a Kimberlite pipe. Graeme Bartlett (talk) 05:30, 10 September 2008 (UTC)[reply]
A) The ocean already has ~50 times as much dissolved carbon as the entire atmosphere, so it is implausible that you could pump enough carbon into the ocean to really pose a threat due to gaseous discharge (i.e. Lake Nyos) B) The key word for Kimberlites was "volcano", their source needs to be at least 1000 C. You can't really compare a capped well to a deep, fast moving flow of molten rock and dissolved gas. Dragons flight (talk) 05:48, 10 September 2008 (UTC)[reply]
The issue with the ocean is putting too much in too little water. At pressures of 100 bars you can dissolve plenty of carbon dioxide. Cool carbon dioxide breaking out from a well would be dangerous to humans and animals due to suffocation. It does not need to be heated to 1000 degrees. Carbonatites can be as low as 600° degrees. By the way does anyone know how to get the degree symbol that used to be down the bottom of the edit box? Graeme Bartlett (talk) 05:56, 10 September 2008 (UTC)[reply]
My understanding of ocean storage is that the CO2 will be simply pumped into the ocean depths, rather than stored in the sub-floor as in above-ground CCS schemes. This avoids the catastrophic-release problem, but presents others such as the massive formation of carbonic acid and uncertainty on the speed of return to atmosphere.
And I'll try to make a degree symbol from the default Insert menu right now: ° Does that look right? Franamax (talk) 06:04, 10 September 2008 (UTC)[reply]
Hey thanks that ° works, my menu selection was on wiki markup! To work out the risk there may have to be a proper risk assessment looking at how many people are killed, economic loss, non productive use of energy etc. The lowest risk option may be to put the carbon dioxide in the air. There would not be enough old oil and gas wells to take all of the power station emissions of CO2 in any case. I much prefer the real soylent green method. Another way is to reproduce the azolla event that stripped the atmosphere of carbon dioxide. Also the end of the cryogenian was a time when huge amounts of carbon dioxide were removed, but this cannot be done now. Graeme Bartlett (talk) 06:14, 10 September 2008 (UTC)[reply]
Actually, estimates suggest there are enough geologic repositories to eat all emissions for 100 years or so. Doing so would require drilling new wells, usually into deep (2+ km) saline aquifers, but that's not too big a deal. We annually remove ~3×1012 m3 of natural gas from the Earth. We have about 20×1012 m3 of CO2 per year to store. The room exists, at least for a while. (It shouldn't be a shock that those numbers are on the same order of magnitude since natural gas is one of the three principle fuels used to create CO2.) You seem to have a lot of fear and doubt, but I suspect you've never really looked into the technology in serious way. It's not as scary as you seem to imagine, and has a lot of potential for near term mitigation. Dragons flight (talk) 07:14, 10 September 2008 (UTC)[reply]
Well, one of the sources of FUD is that these solutions will presumably be provided by private corporations, who will sell their sequestration services for a profit. And when their CO2 reservoir fails (especially if they have over-pressurized them by the 20/3 ratio you allude to above), there will be human deaths near the failure zone, there will be no corporate assets left to claim in the damage suit other than the broken CO2 storage reservoir, and all the CO2 will be back in the atmosphere. Yeah, there is some doubt there. But whatever, lets forget about that old energy conservation idea - that was hippy talk anyway. Franamax (talk) 07:25, 10 September 2008 (UTC)[reply]
That 20:3 ratio (as scarey as it is) is nothing like the right ratio though. It assumes that we can efficiently remove the CO2 from all of the other gasses that come out of the power station (or whatever). Since the air used to fire the power plant is only 1/5th oxygen - and the rest nitrogen, most of what comes out of the power plant is nitrogen. If you try to pump all of the exhaust gasses into your underground store then the ratio isn't 20:3, it's more like 100:3. Separating out the CO2 from the nitrogen is difficult and requires an enormous amount of energy...which in turn means generating even more CO2 than we do now. Conservation and renewable are by far the best route. SteveBaker (talk) 18:13, 10 September 2008 (UTC)[reply]
Actually, removing CO2 from a vapour stream is relatively straightforward, at least in gas processing it's done with an amine absorber. I have no great idea on the energetics though, nor what size of vessel you would need to process the output from a power station. Or if that's even the scheme one would use at a power plant. Also, amine contaminates easily, is a high-foaming service, and is deadly poisonous. But anyway, it selectively removes CO2! Franamax (talk) 03:39, 11 September 2008 (UTC)[reply]

follow up question

Is growing algae at power plants a good alternative to ocean or well storage when reclaimed water from septic systems is used as well?

detectors

what is the job of muon chamber in detectors —Preceding unsigned comment added by 121.52.155.50 (talk) 05:22, 10 September 2008 (UTC)[reply]

Are you talking about a muon detector? It detects secondary muons produced by cosmic rays. Applications may include geophysical exploration by measuring absorption by rock in various directions. Graeme Bartlett (talk) 05:35, 10 September 2008 (UTC)[reply]
Or are you talking about the Compact Muon Solenoid in the LHC? Muons are emitted from the experiemnts or formed by decay of other particles, and can penetrate further and last longer than most of the other high energy particles. These can be detected further out form the core. Look at this for how this works and what it is used for. Graeme Bartlett (talk) 05:47, 10 September 2008 (UTC)[reply]
A hypothetical Higgs boson emits four muons. Graeme Bartlett (talk) 10:11, 10 September 2008 (UTC)[reply]

Bayhawk?

What kind of a bird is a bayhawk? 71.113.3.76 (talk) 06:32, 10 September 2008 (UTC)[reply]

You might perhaps wish to consider typing "bayhawk" into the search box on the left side of the page. I think you would be pleased with the results. -- Captain Disdain (talk) 08:08, 10 September 2008 (UTC)[reply]

CERN's experiment

Anyone knows the exact time when they'll start the experiment? -59.95.99.160 (talk) 06:32, 10 September 2008 (UTC)[reply]

It should be just about no
The high intensity experiments aren't coming until the end of the year. Plasticup T/C 07:20, 10 September 2008 (UTC)[reply]
MSN just a a front-page piece about how the LHC might destroy the world (not kidding). Times like these I have an even lower opinion of the media in general.... --mboverload@ 07:25, 10 September 2008 (UTC)[reply]
i'd be more concerned about one of those 30 ton magnets with a defective mounting bracket. Gzuckier (talk) 16:38, 10 September 2008 (UTC)[reply]
I'd say they're just doing the 'end of the world' thing for publicity now (or at least not stopping it). Nadando (talk) 01:21, 11 September 2008 (UTC)[reply]

High Intensity o.O -59.95.99.160 (talk) 07:25, 10 September 2008 (UTC)[reply]

I like todays image at http://www.google.com/ PrimeHunter (talk) 10:57, 10 September 2008 (UTC)[reply]
Is the word "Google" being sucked into a mini black hole in today's logo? :-) Fribbler (talk) 12:38, 10 September 2008 (UTC)[reply]
The world will not end because of the LHC. CERN has prevented such an accident by preemptively destroying the Earth. [7]DanielLC 16:14, 10 September 2008 (UTC)[reply]
it's all part of cern's plan. by creating the web, they led to google; so that when the earth gets destroyed by a black hole, a digital archive will still exist. Gzuckier (talk) 16:37, 10 September 2008 (UTC)[reply]
Thanks for the link. Very calming. Like the end of On the Beach. Saintrain (talk) 16:49, 10 September 2008 (UTC)[reply]

hair dryer voltage dilemma

i bought a dual voltage hair dryer for travelling. it has a black switch on the handle where it allows you to choose 115V or 230V by turning it w/ a screw driver or a coin. where should it be when in the US???? —Preceding unsigned comment added by 68.97.36.155 (talk) 06:34, 10 September 2008 (UTC)[reply]

115V in the US, for sure. 230V is for Europe and you will need a little plug adapter to go between NA flat plugs and Euro round also. Franamax (talk) 06:47, 10 September 2008 (UTC)[reply]
Please could more regulars chime in to emphasize that 115V is the correct setting!? Franamax (talk) 07:33, 10 September 2008 (UTC)[reply]
As stated in our article on Mains power systems, this is indeed the case. -- Captain Disdain (talk) 08:05, 10 September 2008 (UTC)[reply]
Also note that the 115V setting will be correct in bathrooms of many hotels in Europe that cater to Americans. Just look for the "110" or similar label and U.S.-style plugs. I've been surprised how common this is.--Scray (talk) 10:29, 10 September 2008 (UTC)[reply]
Yup, have those in Ireland. We call them "shaver sockets", like this one. Fribbler (talk) 12:18, 10 September 2008 (UTC)[reply]
115V in North America. Check out this map [8]APL (talk) 13:14, 10 September 2008 (UTC)[reply]
Agreeing with 115 volt setting for North America. Edison (talk) 15:10, 10 September 2008 (UTC)[reply]
absolutely correct. you will probably not see a 220 volt outlet during your travels in the US and Canada, and if you should perchance find one, it won't accept the normal north america plug which presumably you have for your shaver. Gzuckier (talk) 15:20, 10 September 2008 (UTC)[reply]
Only place one is likely to find anything other than 115V in US houses is sockets specifically designed for heavy-duty appliances (electric clothes dryer, electric stove, air conditioner). And the sockets are usually shaped differently so that you can't plug the "wrong" thing (something designed for 115V) into them. DMacks (talk) 16:20, 10 September 2008 (UTC)[reply]
I thought the 220v outlets in the US were some kind of funky three-phase arrangement though. I don't think you could plug a euro-style 220/230/240 volt gadget into one of those even if the plug did fit. There are also concerns over the frequency shift (60Hz in the USA, 50Hz in Europe) - most devices don't mind but my 110v laptop charger severely overheats in Europe when I plug it into my 240v to 110v converter widget. I believe that could only possibly be due to the frequency change. SteveBaker (talk) 17:56, 10 September 2008 (UTC)[reply]
As a general rule, anything with a transformer or motor can be used on 60Hz electricity if it was designed for 50Hz, but you can't go the other way around: the magnetic field will saturate, and will stop being an inductive load and start being a short circuit. --Carnildo (talk) 20:32, 10 September 2008 (UTC)[reply]
Standard 240 V (not 220 V) outlets in North America deliver two opposite phases of 120 volts plus neutral. This is not 3-phase power, but it does mean that there are two live slots. A European device would expect one live and one return wire, like a North American 120 V outlet, so the regulations would be different regarding safety insulation. (This is all in addition to the safety ground (earth) wire.) Outlets for 3 phase power also exist but are not found in homes. Just to further confuse things, some places in North America use two phases of 3-phase power, thus delivering 120 V between one phase and neutral but 208 V between two phases. This is most common in larger buildings, including some apartments.
As to frequency, resistive applications (heaters, traditional light bulbs) don't care, but transformers do care (as Carnildo says), some types of motors do, and some types of electronics do. If the hair dryer is dual voltage then very likely it will also be marked as taking either 50 or 60 Hz, in which case there's no problem, but you may notice the blower motor running faster in North America. --Anonymous, 20:40 UTC, September 10, 2008.
Modern portable hair dryers, especially dual-voltage hair dryers, use a small dc motor to drive the blower. The voltage for the motor is derived from a "tap" on the heating resistor (with the voltage divider ratio set to produce, say, 12 VAC. This voltage is then fed through a bridge rectifier and thence to the DC motor. This arrangement is advantageous because it's: 1) Cheap; 2) Very compact; 3) Easily accommodates the voltage-switching arrangement; 4) Is completely insensitive to frequency; 5) Allows the motor to turn much faster than the 3000/3600 RPM that an ac induction motor could turn. And did I mention that it's cheap ;-) ?
Atlant (talk) 16:06, 15 September 2008 (UTC)[reply]
The device being operated would not know whether it had 240 volts on one wire and zero on the other, as in the UK, or 120 volts from ground on each supply lead with 240 between them as in the U.S. A motor winding, a bulb filament, or a heating element would still have 240 volts across it (or 230 volts at the anemic Euro standard voltage). The ground wire is just along for the ride, there to carry fault current only if there is a breakdown of insulation, and does not normally carry load current. Due to various faults and opens which can occur, either of the supply leads in either country could find itself energized at 240 volts from ground, and would have to be insulated accordingly. Edison (talk) 01:18, 13 September 2008 (UTC)[reply]
Now wait a minute, if the two wires are +120V and –120V (i.e., each is 120V AC offset by 180° phase), that certainly gives 240V AC between them just like a 240V and 0V pair. However, how could a line that varies between –120V and +120V "find itself energized at 240 volts from ground"? I thought all the absolute voltage measurements were compared to ground defined as 0V? DMacks (talk) 18:10, 13 September 2008 (UTC)[reply]
Ah, so that's why so many things in industrial things are 208V! Always struck me as a weird number to get from the same power distribution system that gave 120V. DMacks (talk) 00:18, 11 September 2008 (UTC)[reply]
Star vs. delta 3-phase wiring
You get it when your three-phase system is wired in the star-configuration. 120V from the centre to the outside, between outside points = 2 x 120V * sin(120) (the voltages at each "point" of the star are 120 degrees out of phase). I'd link an article but I can't find any good links for delta/star three-phase - and I've been burned enough lately that I'm not going to complain about it either, someone will just point out my spelling mistake! :) Franamax (talk) 03:23, 11 September 2008 (UTC)[reply]
This has been a very illuminating answer. Edison (talk) 03:35, 11 September 2008 (UTC)[reply]

boiling water in the states

hello wikipedia, Once of the (many) reasons i find americans endlessly fascinating is that they don't use kettles. As work is going really slowly today, i thought i'd just ask, if you gyus wanted to boil water for cooking (pasta, potatoes etc) what would you do, just put cold water on the stove and heat it up from scratch? Does this not drive you crazy? thanks217.169.40.194 (talk) 08:18, 10 September 2008 (UTC)[reply]

An electric kettle can boil the water faster than a gas stove. Its even quicker if you heat a third of your water on the flame, and two thirds of the water in a kettle and then combine them. But simpler if you don't use a kettle at all. Do Americans call a kettle by a different name? Graeme Bartlett (talk) 10:09, 10 September 2008 (UTC)[reply]
I'm the American child of East-coast American parents, and we've always had (and called it) a kettle. Same for my wife, a native of California. I think someone is over-generalizing here. Perhaps we need a WP:RS.--Scray (talk) 10:34, 10 September 2008 (UTC)[reply]
Non-use of kettles is fascinating? I can assure the OP that some, possibly many, Americans do use kettles. We have the plain kind that you plop onto the stove; we have the electric kind. In bygone days, people left filled kettles on wood stoves or coal stoves to provide some humidity in the winter. (More recently, people bought cast iron kettles to do the same in a retro kind of way, only to learn that cast iron isn't a great idea for holding water over long periods of time.)
My guess would be that most people here cook pasta or potatoes by boiling the water in a large pot on the stove (like a stockpot or a large saucepan). A related guess is that many kettle-users drink tea; people who drink mainly coffee will tend towards coffeemakers.
I agree with the Tea assumption. My aunt lives in Italy, and had to get a kettle shipped over from Ireland to make the sacred brew. In Italy, being very much a coffee country, kettles were nearly impossible to obtain! Fribbler (talk) 12:24, 10 September 2008 (UTC)[reply]
As for being driven crazy, we have so many options here that the agony of waiting for pasta water to boil doesn't enter into it. (By "kettle" I'm referring to a vessel with a spout -- e.g., teakettle. Another usage is for a large, open vessel, as in "a different kettle of fish," whence also "kettledrum." Without that kind of kettle, there'd be no cartoons with explorers or missionaries being prepped for dinner.) --- OtherDave (talk) 10:56, 10 September 2008 (UTC)[reply]
The big thing with the missionaries is a cauldron. DuncanHill (talk) 15:31, 10 September 2008 (UTC)[reply]
It's significantly more energy efficient to use a kettle, which is something Americans have never been too concerned with (just look at average fuel consumptions of cars in the US compared to Europe, say). That may have something to do with the different practice on the other side of the pond. --Tango (talk) 11:34, 10 September 2008 (UTC)[reply]
I agree with OtherDave. For one, I would call it a tea-pot or tea-kettle. Simply saying "kettle" needs context to know whether you meant for making tea or if you meant something that Elmer Fudd would boil Bugs Bunny in for stew. For two, everyone I know would likely use a large sauce pan or pot to boil water in for potatoes or pasta. And lastly, I would put a lid on the pot in order to save energy and time. Dismas|(talk) 12:25, 10 September 2008 (UTC)[reply]
Electric kettles are very handy for a cuppa. I assumed that the reason they were so fast is the 250V. What is the wattage of a UK e-kettle? Saintrain (talk) 12:34, 10 September 2008 (UTC)[reply]
2-3 kilowatts [9]. Fribbler (talk) 12:36, 10 September 2008 (UTC)[reply]
To avoid confusion, note that (in the UK at least) teapot and kettle (never heard it called a "tea-kettle") are different objects. You boil water in a kettle, then pour the boiling water into a teapot together with tea leaves or tea bags to make tea. Gandalf61 (talk) 13:06, 10 September 2008 (UTC)[reply]
Good thing you don't have Carolina Wrens over there. You'd have no idea what they were talking about. -- Coneslayer (talk) 13:11, 10 September 2008 (UTC)[reply]
When you say "kettle", are you referring exclusively to a "teakettle"? (With a spout and everything. Probably a whistle, too.) Because that's all the word "kettle" means in USA. I notice that the article mentions that it could have a wider meaning in the UK.
For completeness, what would be the pasta making procedure in, say, England? Would you pre-boil some or all of the water in a teakettle before transferring it to a pot? That seems like a good amount of extra effort. APL (talk) 13:12, 10 September 2008 (UTC)[reply]
As an energy-conscious German, yes, I pre-boil water electrically before filling into a pot, be it for rice or pasta cooking. You'll do that too as soon as you pay as much for energy as we here do. --Ayacop (talk) 13:44, 10 September 2008 (UTC)[reply]
I did manage to find a store in Texas that had electric kettles - they aren't easy to find though. It's really crappy by UK standards - but it works. IMHO, the way you heat water depends on the amount. For small quantities (eg to make a cup of coffee), I use the microwave oven. It's faster than a kettle and vastly more energy-efficient. For medium sized quantities (eg to make four cups of coffee), I use our Texas kettle. At those quantities, it's more energy-efficient than the microwave and faster too. For larger quantities, I use the stove-top and a saucepan - more for speed than for energy efficiency. If I had a larger kettle, I'd use that. SteveBaker (talk) 13:37, 10 September 2008 (UTC)[reply]
The Utah teapot - very important to computer graphics geeks!
When you say "tea kettle" in the USA, you're typically imagining an approximately hemispherical vessel with a flat bottom that you set on top of the stove. When we say "kettle" in the UK (they aren't "teakettles") - we know about the US thing - but we're more likely to be thinking of a large cylindrical pot with an electric heating element in the bottom that can boil a couple of pints much more quickly than you could ever do on the stovetop - and with better efficiency too. The fancy modern ones have dual elements for greater efficiency and little float gauges on the side to indicate how full they are. They typically have "curly-cord" power connections (like a short telephone cord) for kid-safety, auto-shutoff when they boil, maybe have a beeper to replace the old whistle. They are sophisticated, highly evolved machines - compared to an American tea-kettle: think "Ferrari versus Trebant". A "teapot" is a (typically) ceramic pot with a spout and a lid in which the tea is actually brewed - you don't ever heat the water in a teapot.SteveBaker (talk) 13:47, 10 September 2008 (UTC)[reply]
More pictures above .... Gandalf61 (talk) 13:51, 10 September 2008 (UTC)[reply]
Some of us Americans might be more likely to refer to the electric water heating apparatus as a "hot pot," as can be seen by the category name in this Amazon listing. Personally, when I need to heat water for tea, I use the quintessential Revereware kettle; if it was good enough for both my grandmothers, it is good enough for me. Personally, I had a hard time searching for a tea pot that I liked, but that's another story. --LarryMac | Talk 14:16, 10 September 2008 (UTC)[reply]
The Revereware kettle looks like the sort of thing one would take camping. A few people in Britain still use such kettles on a gas hob at home. DuncanHill (talk) 15:33, 10 September 2008 (UTC)[reply]
as a bonus, when attack by redcoats is imminent, the whistle blows "the british are coming!" and it prepares a cup of tea for them. Gzuckier (talk) 16:34, 10 September 2008 (UTC)[reply]
The one and only electric kettle we've ever found here in Texas looks pretty much exactly like one of those - except it has three little rubber feet underneath and a wire coming out of the back! SteveBaker (talk) 17:38, 10 September 2008 (UTC)[reply]

I've used an electric kettle in the UK to boil water, and it does a fine job on the 240 volts available in the wall outlet. But in the US the voltage is half as much, and the appliance amperage is usually limited by the 15 or 20 ampere rating of the circuit, so it takes a long time to boil water with an electric kettle. Looking at some adverts, the UK kettles seem to have a 3000 watt rating and the US kettles only a 1500 watt rating. A stovetop burner seems to apply more energy to the water and get it boiled faster. Of course someone could have a 240 volt outlet installed in the kitchen for the electric kettle, but this would be very unusual outside a restaurant. 3000 watts would be 10,240 BTU per hour, and stovetop range burners can be had with 15,000 BTU per hour and higher rating. Granted, the electric kettle should apply the energy to heating the water more efficiently than a kettle setting on a range burner. A microwave might have just a 1000 watt rating. Edison (talk) 14:58, 10 September 2008 (UTC)[reply]

What you're missing with the microwave part of that is that the microwave only has to heat up the water and some kind of container (which in many cases is the container you're going to use for the drink anyway). An electric kettle has to heat up that large metal element too - and when you pour the water out, a lot of heat energy remains in the kettle itself. That's why the microwave is a more energy-efficient solution. Also, if you pour the water out of the kettle into a cold ceramic cup - then the cup cools the water down. With the microwave, the cup is also heated - but that energy isn't wasted. The stovetop is even less efficient for much the same reasons - but also the thermal connection between the heater element and the water inside the pot is a very poor one compared to having the electrical element immersed directly into the water...hence yet more energy goes into heating up the air and the metal stove top. That's why the microwave is the most efficient way to heat water (and also the fastest if the quantities are small) - the electric kettle comes second and the gas/electric stove-top is (by far) the worst.
Yet another problem with kettles and stoves is that while the water is heating, oxygen and nitrogen come out of solution and stick to the sides of the container. As it comes close to boiling, steam bubbles do the same thing as the water closest to the heat source reach boiling point long before the rest of the liquid. Those bubbles are extremely good heat insulators and prevent much of the heat transferring into the liquid - allowing the heating surface to (uselessly) get hotter than 100degC. Microwaves (by their very nature) only heat the water itself - so 100% of the energy goes into the water. You might think this is a negligable effect - but it's really not - and I can prove it (albeit indirectly)...
If your car engine is overheating, you can sometimes get it to run cooler by flushing out the antifreeze and replacing it with plain water. That's because plain water conducts heat better than ethylene glycol. However, this steam-bubble formation issue strikes because the boiling point of water is lower than the glycol/water mixture and the water may boil as it passes through the engine block causing localised hot spots that can actually ruin your engine...maybe even crack the block. There is stuff you can buy called "Water wetter" that lowers the surface tension of the water and raises it's boiling point WITHOUT reducing it's thermal conductivity that fixes this problem entirely.
I agree that the lower mains voltage/current is the likely reason why electric kettles aren't popular over here though...although I have 240 volts going to my cooker - it wouldn't take much to wire up a UK-standard 3 pin plug...hmmm - it's tempting!
SteveBaker (talk) 17:38, 10 September 2008 (UTC)[reply]
UK mains is generally fused at 13 amps, so if you have up to 20 amps that compensates for the lower voltage quite well. 115V at 20 amps gives a maximum of 2.3kW, plenty for boiling water (it's the cheaper end of the UK kettle market, but it'll do just fine). Rather than talking about all these meaningless numbers, let's try some meaningful ones - it takes about a minute to boil a couple of pints in my kettle (far less to boil a mug full for a cup of tea/coffee). How long would it take on your hob (or "stovetop" or whatever you call it)? --Tango (talk) 15:19, 10 September 2008 (UTC)[reply]
Just pointing out the UK mains electricity has been delivered at 230 volts rather than 240 volts for something like the last 10 years. To harmonise European standards, those continental supplies that were on 220 volts have been or are in the process of being upped to 230 volts. I seem to remember some spurious talk at the time of the changeover that it would affect the economy because it would take longer to boil a kettle and people would take longer tea-breaks. It would also make you late for work because it would take longer to make toast. Jooler (talk) 15:21, 10 September 2008 (UTC)[reply]
Well, it's 230V +/- 10%, so that's anywhere between 207V and 257V, so the change from 240 to 230 is less significant than the natural variation anyway, so I don't think you can use that as an excuse when you're late for work! --Tango (talk) 15:49, 10 September 2008 (UTC)[reply]
Hmmm - I always thought the change was not one of the actual voltage provided - but a change from quoting the peak-to-peak voltage to talking about the RMS voltage in order to comply with Euro measurement standards. I guess I'm wrong on that score then! SteveBaker (talk) 17:46, 10 September 2008 (UTC)[reply]
decadent capitalists not inwenting samovar!! Hahahaha! Gzuckier (talk) 15:23, 10 September 2008 (UTC)[reply]
I'm a (naturalized) American, so I'll just answer the question as a data point: if I want to boil water for pasta, I take my 4 dogs' communal water bowl, run the hot water tap into it until the water is hot, then fill a thin-metalled pot with the hot water, put some salt in, and put it on my gas range. It's usually boiling within a couple of minutes. --Sean 00:13, 11 September 2008 (UTC)[reply]
You cook with water from the hot tap? Isn't that supposed to be a really bad idea because it's been sitting in a warm tank and pipes for god knows how long? Boiling it may kill off the vast majority of germs, but it won't remove anything that's dissolved in it (hopefully you don't still have lead pipes, or that could be really nasty!). --Tango (talk) 14:14, 11 September 2008 (UTC)[reply]
Most houses in the US use either copper or plastic for the pipes, and you're supposed to keep the hot-water tank at 140F or above, hot enough to kill anything dangerous. --Carnildo (talk) 21:08, 12 September 2008 (UTC)[reply]
According to this page, the CPSC recommends a setting of 120F, for burn safety. I guess you can't make everyone happy. -- Coneslayer (talk) 01:35, 13 September 2008 (UTC)[reply]

boiling a cup of water

on a related note, whatever happened to those little plug in one cup heater uppers? (let alone the immersion heaters) i can't find any for 110 volts any more, only 12 volt car ones. i'm toying with hooking one up with an old gel cel 12 volt battery and a wall wart, just so's i can get a cup of hot water for beverage purposes at work. Gzuckier (talk) 15:26, 10 September 2008 (UTC)[reply]

This [10] would do for making instant coffee, but as it does not actually boil the water is unsuitable for making tea. I would add that as one can buy a perfectly serviceable ordinary electric kettle for a fiver, it does not look like good value for money. DuncanHill (talk) 15:40, 10 September 2008 (UTC)[reply]
Or you could take a thermos of your favoured drink to work. DuncanHill (talk) 15:54, 10 September 2008 (UTC)[reply]
yeah, but i'm looking for something highly unobtrusive. otherwise the Corporate Preventers will spot it and break into one of those "whatever it is, I'm against it!" routines that they do so well. that's why i've been holding off on the battery/12 volt mug approach.Gzuckier (talk) 16:31, 10 September 2008 (UTC)[reply]
Somewhere on the interweb I saw a USB-powered coffee maker - the USB charges a battery inside that actually heats the water. I'm not sure that helps the issue with the Corporate weenies being a pain. It would be kinda neat on a back-packing holiday though: "I sure could use some hot coffee right now."..."Sure, I'll go grab my laptop and be right back!". SteveBaker (talk) 17:46, 10 September 2008 (UTC)[reply]
Speaking of which this was a great solution for us to run a USB hub in -25 C ice core lab. It's hard to find cheap, cold-rated usb hubs, but you don't have to if they have a built-in heater. ;-) Dragons flight (talk) 17:57, 10 September 2008 (UTC)[reply]
<soapbox warning>I just want to say that I haven't had so much fun since I first came to Wikipedia. Even with the U.S. / U.K. differences, none of the armed-Talmudic screeds about those with disagreeing opinions.</soapbox>
I was born in Canada but raised in the States, and my parents have always had a (non-electric) kettle (for the boiling) and a metal teapot (for steeping the loose tea leaves and leaving on the stove with just a smidgin of heat).
I think the energy-conscious comments are apt; we're still bellyaching about gas costing $3.50 a gallon.
You can find electric kettles (other than the venerable Hot Pot) here, but you have to look hard, or else go to Canada.
I agree completely with the need to have boiling water for tea (and with the need not to have the word "Lipton" within 500 yards), which is why visitors to the U.S. who are in their right minds never order tea, except perhaps in some non-chain college-town coffee boutique. --- OtherDave (talk) 13:21, 11 September 2008 (UTC)[reply]

UK Spiders

What is the name of those orangey-brown and white spiders that become so prevalent in the UK on cold September mornings? Little blighters were all over my car this morning... sparkl!sm hey! 08:57, 10 September 2008 (UTC)[reply]

Are they like this at European garden spider? Julia Rossi (talk) 08:21, 11 September 2008 (UTC)[reply]
Yes, that's the fella, thanks! Does anyone know why they become so plentiful in early September? sparkl!sm hey! 20:49, 11 September 2008 (UTC)[reply]

MEDICINE:GYNAECOLOGIST

What subjects do i require in high school for me to become a GYNAECOLOGIST?Mrs.Rushaksy (talk) 12:06, 10 September 2008 (UTC)[reply]

Gynaecology is a medical speciality. So the course you would be taking at third level is medicine. As I understand, in the US (you said high school, so I'm guessing you live there) medicine is a graduate-entry course in that you must first have a relevant degree in say Biology first before you can do medicine. So Biology and Chemistry would be the subjects best suited to pursuing this first degree. Fribbler (talk) 12:14, 10 September 2008 (UTC)[reply]
If you truly want to be any sort of specialist, this is a suggestion that I've seen work with many people since I've been working in the medical field:
  1. Start volunteering at your local hospital now. You will be doing junk work with no (or very little) pay. But, you will be around doctors and get used to hearing medical-speak. You may realize that you hate the profession and change your mind.
  2. Take all the biology and physics that you can in High School. If yours is like mine was (white-trash redneck tiny-town school), you'll have to take classes at a local community college instead of high school.
  3. By your senior year, pick the medical schools you want to go to. They will have "sister colleges" that offer the pre-med courses. Now, you know what undergraduate colleges you want to go to. You can work on getting into them.
  4. As an undergrad, continue volunteering at the medical schools you want to go to - including applying to do any work you can in research projects. You'll get paid for that work. The goal here is to get your name known with as many people in the school as possible.
  5. As you near the end of pre-med, study and study and study for the MCAT. Your score on the MCAT will be a major factor into getting into med school as well as getting free funding for the school.
  6. Assuming you do well on the MCAT, you'll be accepted to the medical school. You'll be a general medical student (specialist courses don't matter at the beginning). Try to get an advisor that is in the field you want to work in. He or she will guide you through the courses you need and possibly get you into a volunteer position in the field.
  7. From that point, you are set - do well in your classes, keep volunteering, and all those years of working for no pay will finally pay off as you will have the education and the years of experience to bubble up to the top of your class and get a good job anywhere you like.
Keep in mind that that is only one way to do it. You could just have rich parents to donate a building to the medical school in exchange for a degree. -- kainaw 12:48, 10 September 2008 (UTC)[reply]
Also: you're in high school, so please forgive me if your intentions are pure, but, speaking as a one-time high school boy, let me advise you that if you're thinking that this is a good way to see lots of vaginas, I can't tell you how wrong you are. Under no circumstances should you (or anyone else, for that matter) click on this link: Google image search for "diseased vaginas". --Sean 01:10, 11 September 2008 (UTC)[reply]

physics

whether man can run fast or slow on moon.Plz explain with respect to on earth

—Preceding unsigned comment added by Khubab (talkcontribs) 17:13, 10 September 2008 (UTC)[reply] 
Not fast in short distances, because each step takes you away from the surface for too long, so you can't accelerate fast enough. Long distance is different because no friction happens while in the air, but I'd guess it difficult to accelerate further when already at high speed. --Ayacop (talk) 17:24, 10 September 2008 (UTC)[reply]
An additional problem for lunar hiking is one of mass. An astronaut must haul around a substantial portion of body mass in life support equipment. While the weight of all this is fairly low (with the 1/6g at the lunar surface), its mass and inertia is unchanged. It's much harder to start, stop, or change directions on the moon than on Earth. — Lomn 18:00, 10 September 2008 (UTC)[reply]
An Earth-bound similarity is the high-speed cars that try to break ground-speed records. The ones that use tires as propulsion have to deal with the friction between the tire and the ground. If they gun it at the start, they spin out and don't move much. That is a lot of wasted fuel and energy. So, they start slow and build up speed. At high speeds (around 300mph), the tires no longer have the ability to push the car forward. They just spin out if any attempt to accelerate is made. So, the car has reached a top speed, not based on the engine, but based on the tire's friction. Translate that to a man on the moon and you have the situation explained by Ayacop. -- kainaw 18:03, 10 September 2008 (UTC)[reply]
A perhaps-similar question was answered empirically in the greatest scientific experiment in history (before the LHC, of course): can you swim faster through a swimming pool full of syrup than through one of water?--Sean 01:20, 11 September 2008 (UTC)[reply]
Much much much faster. Drag force is and just as sprinters lean into a curve by about 10-degrees, a sprinter could lean forward almost 45 degrees if running on the moon and reach incredible speed and without falling forward. They would have a huge advantage not only in decreased drag force, but could have a much more beneficial angle to use their feet to push against the ground (assuming they have good shoes, on a concrete or asphalt surface for example). Also if you notice the drag equation, rho is the air density (or fluid density) so if you reduce this by about 90%, you could triple your velocity. Also, this assumes you train for several days, getting used to the new feel and nuances of sprinting on the moon. Sentriclecub (talk) 09:54, 14 September 2008 (UTC)[reply]

light

Does light acclerate on approaching a black hole? —Preceding unsigned comment added by 117.197.209.41 (talk) 17:49, 10 September 2008 (UTC)[reply]

No (provided the area around the black hole is a vacuum). Gravity can affect the trajectory of light but not its speed. — Lomn 17:56, 10 September 2008 (UTC)[reply]
Which is a "yes." I agree that the black hole does not change the speed at which light travels, but remember that a change in direction, even at constant speed, is also an acceleration. Chuck (talk) 21:24, 10 September 2008 (UTC)[reply]
While true, I doubt this is the context in which "accelerate" was used. — Lomn 21:34, 10 September 2008 (UTC)[reply]
It is, however, blueshifted. --Tango (talk) 18:05, 10 September 2008 (UTC)[reply]
Not from where I'm standing (I hope). Saintrain (talk) 19:17, 10 September 2008 (UTC)[reply]
If you're standing somewhere a significant distance from the black hole (as I would hope you are, seeing as I'm on the same planet as you!), you won't be able to see any light approaching the black hole, since you would need to be inbetween the light and the hole. --Tango (talk) 14:19, 11 September 2008 (UTC)[reply]

CERN AL-Zanders' dropping some mighty LHC yo!

That's science-ghetto-geek talk for my own reference of one: CERN, and two: the Large Hadron Collider.

Question: Will this effect the way we make toaster ovens?

Meaning, do we the little people of science know the uses this collision will do for us in the proletariat section?

Not the Little-Black-Hole-Bang talk, but on a slippery slope, could this be a stepping on a butterfly, as in the Effet de Papillon?

Cheers,--i am the kwisatz haderach (talk) 17:51, 10 September 2008 (UTC)[reply]

Wikipedia is not a crystal ball. It cannot be used to tell the future. If you are asking for references (as this is a reference desk), a search of periodicals at all three local universities shows absolutely no hits on articles containing both "hadron collider" and "toaster oven". -- kainaw 17:56, 10 September 2008 (UTC)[reply]
He is asking if any of the LHC research will do anything for your average joe. Plasticup T/C 18:24, 10 September 2008 (UTC)[reply]
instead of mountain climbing you can go wind surfing on a lake, the positive side, for the average Joe, the problem with rising sea levels is solved.Mion (talk) 18:48, 10 September 2008 (UTC)[reply]
So, we split the atom, then blew up folks with some a-bombs. I'm just saying, what are the negatives and positives, most likely the negatives that cripple mankind will be extorted first. Considering history. Ok, simply: We collide some Particles. Okay., and the point is??? That's the question. The answer I take from the round abouts is, "We'll all see when it happens." And that's an alright answer. So the premise is, we can do something we have never done before, to our collective memory of 4000 years, but we don't really know what we're doing. And that's ok. But if any scientists here actually know some simple E=mc2 answer, Energy equals Mass, what of it? Is this to understand Mass more? Understand Energy? Yes, we are Technological here, yes, we have some smart answers w/ or w/o sarcasm. But brass tacks: What are we doin' here with this LHC? 'Explain your answer to a 12 year old.' <--true sciences. Please. Cheers, --i am the kwisatz haderach (talk) 19:19, 10 September 2008 (UTC)[reply]
LHC is a tool that will be used to examine subatomic particles and the forces with which they interact, and to study some areasa of physics where certain theories were not testable experimentally until now. It's a basic-science thing, not an engineering/applied-science "build a better mousetrap" goal. One could certainly speculate about new devices that could be built using whatever exotic stuff they find, but that's...speculation, even on the part of those actually doing the experiments (who no doubt had to somehow justify this hugely expensive special-purpose gadget to numerous governments and scientific agencies). DMacks (talk) 19:29, 10 September 2008 (UTC)[reply]
It's unlikely the LHC will have any benefits to individual consumers, if that is what you are asking. It is likely that the biggest overall benefit to society will be in the form of keeping a lot of scientists on the payroll in general, and of those scientists a certain number will eventually come up with something helpful or practical. Many of the scientists will gain in prestige from their work with the LHC, which will help them get future funds, and the whole cycle continues. There's always the rare, rare chance that something will directly come out of work like this but it doesn't happen very often, and I don't think anyone's expecting it to happen here. Big Science is as much about creating a Big Infrastructure as anything else, which can serve as an end to itself. That being said, there's a strong argument for, "surely we could spend the money better elsewhere" which might be true, though frankly most governments burn through money in the most inefficient ways anyway, and the amount of money spent on the LHC is a drop in the bucket. --98.217.8.46 (talk) 19:40, 10 September 2008 (UTC)[reply]

Thanks much. Digression: There's a really cool video game at the time, late 90's-Early 00's(Early Zeros), that had this Science David Banner-like guy going into the CERN or CERN like particle accelelerator. Opens a mini-black hole, another dimension of time and space, then you go about on this adventure. End of game you come back chasing a villian, I think you end up blowing him up, more imploding him up with the facility. I forgot the name of the game, but it was fun. And the first time I heard of a Particle Accelerator. And I think there was a Jean-Claude Van damme movie with one in it. Reading more about it I see the basic-science questions of MORE INFO on the DarkMatter/Energy thing, Matter/Antimatter, GUT, etc. GO SCIENCE!!! --i am the kwisatz haderach (talk) 19:44, 10 September 2008 (UTC)[reply]

Just a question, is there some news on Composite Fermions ? Mion (talk) 19:57, 10 September 2008 (UTC)[reply]
In case the 12 year old doesn't follow all that, the idea of the Large Hadron Collider experiment as I understand it is to whip some incredibly tiny particles up to incredibly high speeds and then let them crash into each other, to see what happens and perhaps learn something about the universe.
The experiment is not considered dangerous because the amount of energy involved in each "crash" is not quite enough to run a 60 watt light bulb for a second.
(That is if I did the calculation correctly. Perhaps someone will check it.) Wanderer57 (talk) 20:00, 10 September 2008 (UTC)[reply]
The 1991 video game I referred to earlier is called 'Out of this World'. A great 2D game, with exceptional graphics for that time. Wikis, mosdef check this one out. You'll like it. Tschuss, --i am the kwisatz haderach (talk) 20:42, 10 September 2008 (UTC)[reply]


It's testing a scientific theory. Science isn't just going around saying "what if?", it's going around saying "why's that?". Once there's an understanding of "why" then there can be predictions of "what else".
There are precise predictions of what's going to happen during the collisions, based on the current (prevalent) model of how the world works. The predictions will be borne out, confirming the model, or they won't, providing new insights into how the world works. If confirmed, researchers will have greater confidence in the predictions that will lead to engineering advances.
There have already been significant output from CERN. The WWW, for one. Knowledge learned through the development of really big magnets may lead to fusion power sooner. (No, not at LHC. Don't start that rumor.)
Einstein's "why's that" lead directly the A-bomb; and to transistors and solar cells and DVDs; and the LHC.
Hubble's to the big bang; see LHC.
Curie's "huh?" lead to the bomb; and atoms and cancer treatments and cat scans.
Pasteur's "why does wine sour?" to pasteurization and antibiotics. He's also directly responsible for much of the population explosion, but population control through food poisoning and plagues seems, well, you know, natural.
Franklin's "what's lightning?"; he got the sign of the electric charge wrong but the vibrator was one of the first "household appliances" to be electrified.
These are just a very few of the famous ones. We've been asking "why's that", and finding the answers, for a long time. Saintrain (talk) 20:54, 10 September 2008 (UTC)[reply]
While it's true that science has obviously led to some very practical things, it's not at all been established that pouring billions of dollar into single facilities will do something like that. It's considered pretty unlikely that the LHC will provide any sort of breakthroughs like the ones you have listed above. (And Einstein's work didn't actually lead directly to the atomic bomb, incidentally. Curie's work is a bit more on the mark in that respect—the atomic bomb emerged out of work in experimental work with radioactivity and particle physics, not the sort of fundamental theorizing Einstein did.) --98.217.8.46 (talk) 00:12, 11 September 2008 (UTC)[reply]
Well, if you consider the work involved in affixing his signature to a letter ... --Sean 01:29, 11 September 2008 (UTC)[reply]
Before that, he pretty much nailed the coffin shut on the "atoms don't exist" thing and his mass-energy equivalence proved pretty handy to Meitner et al. As for predictions, would you please forward tomorrow's FTSE close?  :-> Thanks. Saintrain (talk) 16:07, 11 September 2008 (UTC)[reply]
All of you are using one of CERN's inventions, HTML, the internet browser and world wide web. This is a case of scientists working on something not directly related to particle physics, but ending up useful. Graeme Bartlett (talk) 21:20, 10 September 2008 (UTC)[reply]
A SCIENTIST MUST SEE; here's a short 2.5 min vid on an example of a Particle Accelerator with a twist of a B.Franklin/M.Shelley Lightning bolt: [World (Out of this World) Video Game Intro] bon jour, --i am the kwisatz haderach (talk) 21:31, 10 September 2008 (UTC)[reply]
Somewhat obviously: Specialagent777 why are you posing questions, when as the self-claimed Kwisatz Haderach surely it is us who are supposed to seek your "absolute powers of prescience" and ability to "bridge space and time" - I shall remain severely disappointed in your abilities if you can't answer a simple prediction of what equivalent of "Apollo moon landing effort gave us Teflon non-stick frying pans" we shall get out of the LHC :-) David Ruben Talk 22:10, 10 September 2008 (UTC)[reply]

Dentistry question: not advice and not homework

Why is it inadvisable to drink alcohol or smoke within 24 hours of a tooth extraction? —Preceding unsigned comment added by 79.76.128.237 (talk) 17:55, 10 September 2008 (UTC)[reply]

Smoking has statistically been related to a high number of complications, primarily dry sockets. Alcohol (as well as hot liquids) will dissolve the blood clot and open the wound to the possibility of infection. -- kainaw 18:07, 10 September 2008 (UTC)[reply]

If by chance the experiment is failed are there any chances of explosion or any harm to human life? —Preceding unsigned comment added by 06riya (talkcontribs) 18:42, 10 September 2008 (UTC)[reply]

It's just possible, if something goes very wrong, that they might break some of their equipment. There is no risk to humans. Algebraist 18:47, 10 September 2008 (UTC)[reply]
Not true! One human has already been killed by this device (mentioned in the article)! But as Algebraist said, it was from an equipment/breakage accident, not from Particle Physics Gone Wild. DMacks (talk) 19:47, 10 September 2008 (UTC)[reply]
Well, yes, someone was killed by having a whole bunch of equipment dropped on him. Since no-one's going to be going into the accelerator while it's turned on, this is no longer much of a risk. Algebraist 00:12, 11 September 2008 (UTC)[reply]
Just keep an eye on this. Paragon12321 21:25, 10 September 2008 (UTC)[reply]
That's too much work. Just subscribe to the RSS feed. APL (talk) 00:54, 11 September 2008 (UTC)[reply]
Yes! One (unlikely) failure mode would be for some part of the machine to explode or start a fire or wildly shoot the proton beam at an innocent bystander. See here for a description of some of the largish energies involved; a full beam could melt a tonne of copper!. That said, there is no chance of it being more dangerous than a good-sized IED. --Sean 01:41, 11 September 2008 (UTC)[reply]
@ Paragon & APL: LMAO! There's no way it'll say yes anyway because *common sense*? ~AH1(TCU) 00:10, 13 September 2008 (UTC)[reply]

What is a Hadron

Can anyone explain in simple language (not like the article) what a Hadron really is and what is its definition? I mean is it any thing to do with leptons, bosons and quarks and suchlike? Is it a subatomic particle? —Preceding unsigned comment added by 79.76.128.237 (talk) 19:13, 10 September 2008 (UTC)[reply]

Forgot to ask, whats a large hadron. Is it just a proton or neutron? —Preceding unsigned comment added by 79.76.128.237 (talk) 19:16, 10 September 2008 (UTC)[reply]
The Large Hadron Collider is a large collider of hadrons, not a collider of large hadrons. -- Coneslayer (talk) 19:19, 10 September 2008 (UTC)[reply]
Like they would've got funding if they called it the "Small Hadron Collider"! :) Franamax (talk) 03:07, 11 September 2008 (UTC)[reply]
(ec) As it says in the lead paragraph of that article, the best-known hadrons are protons and neutrons, which are subatomic particles. The Large Hadron Collider will collide protons. More generally, a hadron is made of several quarks held together by the strong force. Some hadrons are bosons, others (like protons and neutrons) are not. No hadron is a lepton; leptons are something else entirely, and include electrons. Algebraist 19:20, 10 September 2008 (UTC)[reply]
Is there any truth to the rumour that in Ireland the hadrons are small and green and are called leprickons? Wanderer57 (talk) 22:37, 10 September 2008 (UTC)[reply]
Yes. --Tango (talk) 22:43, 10 September 2008 (UTC)[reply]
I had a small green Hardon hadron once. But the doctor gave me some cream to put on it. —Preceding unsigned comment added by 79.76.128.237 (talk) 23:23, 10 September 2008 (UTC)[reply]
Yes, I was tempted to chime in and say it's a dyslexic erection, but that would have been tasteless and juvenile, so I refrained.  :) -- JackofOz (talk) 03:11, 11 September 2008 (UTC)[reply]
I had one of those once; it was hadroneous. Kittybrewster 03:19, 11 September 2008 (UTC)[reply]

Does heat contribute to global warming?

Does the amount of heat given off by a machine or organism contribute in any way to global warming?

Say we had nothing but solar-powered cars, but their motors cooked away at 200 deg C, or say everyone had a solar powered air conditioner spewing hot air into the streets, would this affect global temperatures in any way?

Thanks!

— Sam 19:28, 10 September 2008 (UTC)

I believe that would violate the first(?) law of thermodynamics. The energy from the sun is already entering our atmosphere (mostly as heat to begin with); harnessing it and converting it will not directly be a problem. It's conceivable that how we go about it can cause some problems, but I don't think this particular worry is well founded. - Lambajan 19:43, 10 September 2008 (UTC)[reply]
It's worth noting that there are two different questions here: "does heat given off by <something> affect global warming?" and "does heat from solar-powered sources affect global warming?" I'm inclined to agree with LambaJan on the latter, but the former is not necessarily true -- the fuel source matters. — Lomn 21:28, 10 September 2008 (UTC)[reply]
On second thought, I don't agree with the solar-powered thing, either. Solar cells capture light energy that would otherwise scatter away, converting more of it (eventually) to waste heat. — Lomn 21:30, 10 September 2008 (UTC)[reply]
What would be the ultimate fate of the energy in that unused scattered-away light? DMacks (talk) 21:49, 10 September 2008 (UTC)[reply]
It heats up whatever it's absorbed by. Some of it will be reflected back into space, but Earth gives an albedo of 0.367, so only 36.7% of sunlight hitting Earth is reflected back, and I believe most of that is from clouds and oceans, so solar cells on land probably don't make much difference. --Tango (talk) 22:46, 10 September 2008 (UTC)[reply]
Not much, no -- but if you want to take the argument to extremes, I would expect some difference. — Lomn 23:29, 10 September 2008 (UTC)[reply]
The difference would probably be less than the natural variation from changes in cloud cover, etc. --Tango (talk) 00:08, 11 September 2008 (UTC)[reply]
(semi-digression) A few jobs ago, we had a bunch of small cartridge heaters that were unlabeled as to wattage. We got pretty close estimates by comparing "how hot does this feel compared to a known-wattage lightbulb?" The net energy coming in eventually becomes heat, regardless of whether it starts "as heat" or "as absorbed light". DMacks (talk) 00:15, 11 September 2008 (UTC)[reply]
Cities are hotter than rural areas (the urban heat island), but this effect is mostly caused by buildings blocking thermal radiation from escaping into the sky. There article's "Relation to global warming" section seems to indicate that the heat island has negligible effect on annual temperatures. --Bowlhover (talk) 00:23, 11 September 2008 (UTC)[reply]
I've actually had similar thoughts. My current stance is that the heat from the sun trapped by greenhouse gases must be more than we produce on earth, but let me share an experience that frightened me: as a special treat, I drove to vegas for my 21st birthday with my brother and a friend. We went during a chilly week and arrived at night, but as we approached the immmediate outskirts of the city, it got SIGNIFICANTLY warmer. I have no doubts the heat of Las Vegas relative to the rest of the dessert is due to energy used by all the casinos, especially the lights on the strip. Electricity use results in heat. Lightbulbs, TV's, computers, cars, hot water heaters....we are surrounded by heat sources, and our electricity consumption might correlate somewhat with the rise in global temperature. Lambajan mentioned the first law of thermal dynamics, but most of our energy is drawn from coal and petroleum: energy from the sun that was stored long ago and is being released long after the fact en masse. Could ground level heat production effect world temperature? It's possible, But I don't know whether it would be significant relative to other sources. --Shaggorama (talk) 03:58, 11 September 2008 (UTC)[reply]
Yes, it has an effect, but no, it is not significant. The solar constant is about 1360 W/m2, or 340 W/m2 if averaged over the surface of the Earth. Even Vegas is much darker than that. The increased temperature you feel is mostly due to stored heat from the day being re-radiated, from decreased cooling due to the more complex artificial landscape, and from decreased air exchange due to buildings. I felt exactly the same effect you describe when entering Kata Tjuta about a months ago, and there are no external heat sources to blame there. --Stephan Schulz (talk) 08:56, 11 September 2008 (UTC)[reply]
With reference to the photovoltaic panels mentioned earlier, you might like to note that they would typically be reflecting more solar radiation back into space than the average albedo of 37% cited earlier. A researcher for a leading PV manufacturer gave me figures a while back of 50% reflected, 15% converted to electricty and 35% kept as heat. This could vary quite a lot though. Efficiency rates are gradually climbing and there are transparent modules available which can be used to replace glazed areas in buildings. If vast PV power stations are to be placed into deserts then the issue would have to be considered. It would probably e better to convert as much of the solar radiation as possible into electricity than to reflect it back into space. But in the solar energy article there is mention of deliberate attempts to increase albedo, simply by painting surfaces white. Itsmejudith (talk) 08:23, 12 September 2008 (UTC)[reply]
All the solar radiation that hits the Earth is radiated back out into space one way or another (some of it gets turned into plant matter, but only a tiny percentage). It is either reflected directly as light waves or radiated as infrared heat. It is the infrared portion currently causing our problems with global warming - increased CO2 and other trapping gases absorb the infrared radiation emitted from the ground, that heat energy has to be re-radiated from the upper atmosphere, and a new equilibrium is established at a somewhat higher overall temperature. The amount of energy absorbed from the Sun by our planet is truly vast though, and it is all heading back out. As far as "vast" PV stations in the desert - think about what vast really means. If you covered even one whole desert for solar power, you have covered very little of the Earth's entire surface, and the area doesn't match natural variations in cloud cover and seasonal snow and ice cover. According to our article, the "usable energy" received from the Sun is 8,000 times 2004 human energy use from all sources. By my figures, we need three spots around the globe, 250 km square, carpeted with 25% efficient solar cells, and we're covered. Franamax (talk) 09:33, 12 September 2008 (UTC)[reply]

Water injection

If water was injected into the bloodstream, would it cause pain, or worse? —Preceding unsigned comment added by 79.76.128.237 (talk) 19:31, 10 September 2008 (UTC)[reply]

No. Blood (and indeed all of you) is mostly water already. Saline injection is standard for treating severe dehydration. Yeah, it's "saline" but only very slightly (not like oceanic salt-water). The main risk for "pure water" would be if huge amounts were injectedf aster than the kidneys (etc) could remove the excess water--then the blood would get diluted and its transport properies would change. DMacks (talk) 19:45, 10 September 2008 (UTC)[reply]
Pure water? Yes - painful. Yes - dangerous. I know of a case where sterile water was infused intravenously by mistake instead of saline. And it was extremely painful. It is also dangerous, because the lowering of the osmolarity of the plasma may lead to hemolysis, and well as damage the endothelium. Large quantities? Lethal. --NorwegianBlue talk 20:53, 10 September 2008 (UTC)[reply]
Okay yeah, didn't think about local effects for pure water:) DMacks (talk) 21:33, 10 September 2008 (UTC)[reply]
What local effects? Wouldn't it diffuse into the blood stream pretty quickly? --Tango (talk) 00:07, 11 September 2008 (UTC)[reply]
Meaning the lysis of red blood cells exposed to that pure water before/while it was diffusing. So a drop or two no big deal (and even if you lose "only a few" cells, probably wouldn't notice) but a syringe-full would take noticeable time to diffuse and noticeable gradient while doing so I'd assume. DMacks (talk) 00:11, 11 September 2008 (UTC)[reply]
You can give yourself an idea of how much your body prefers salt water by first pouring pure water up your nose, and then pouring salt water. The latter is *much* more comfortable. See neti. --Sean 02:27, 11 September 2008 (UTC)[reply]
...which link eventually leads you to nasal irrigation.--Shantavira|feed me 06:33, 11 September 2008 (UTC)[reply]
For reference, a saline solution is 0.9% w/v NaCl. See saline (medicine). --Russoc4 (talk) 02:42, 11 September 2008 (UTC)[reply]
Well, does the same go for tap water, because clorine is, uh, poisonous, and floride causes retardation? ~AH1(TCU) 00:03, 13 September 2008 (UTC)[reply]

Earwax

Is it true that people in olden times chewed their food more with a sideways movement and then didn't get earwax building up problems/ —Preceding unsigned comment added by 79.76.128.237 (talk) 19:36, 10 September 2008 (UTC)[reply]

Uh, no. If your earwax is related to how you are chewing your food, you're probably doing something wrong. --98.217.8.46 (talk) 19:46, 10 September 2008 (UTC)[reply]
I doubt it - where did you hear that? --Tango (talk) 19:58, 10 September 2008 (UTC)[reply]
From this article: "As mentioned above, movement of the jaw helps the ears' natural cleaning process, so chewing gum and talking can both help." -hydnjo talk 20:12, 10 September 2008 (UTC)[reply]
we ha? Chewin and talkin at the same time pardner? Thats mighty advanced for us country folk. —Preceding unsigned comment added by BellyGrease (talkcontribs) 22:44, 10 September 2008 (UTC)[reply]
Nah, just a bit of multitasking is all. Now walkin' and chewin' that's tough. -hydnjo talk 10:57, 11 September 2008 (UTC)[reply]
Chewing anti-nicotine gum for a prolonged period caused me to lose dentine. Bad news. Kittybrewster 03:24, 11 September 2008 (UTC)[reply]
Whose bright idea was it to put antimatter in chewing-gum? —Tamfang (talk) 16:50, 14 September 2008 (UTC)[reply]

Women and How They Think

I'm not trying to be rude in anyway, so please dont take this the wrong way... Why do so many women not have a mind of their own? They'll date a guy or get with a guy that their friends think is good looking. They'll leave a guy they are interested in only because their friends do not like the guy (usually the fat friend that cant get guys). They dont live for themselves, they live for other people. Not all of them, but many of them. Why is this? I don't know any guys that are like this —Preceding unsigned comment added by 75.82.33.31 (talk) 21:44, 10 September 2008 (UTC)[reply]

I don't know women that are like that either - maybe you just hang out with the wrong crowd. --Tango (talk) 22:42, 10 September 2008 (UTC)[reply]
A lot of them also act like Carrie Bradshaw and do not realise that she is a fictional character played by an actress. —Preceding unsigned comment added by 78.149.104.222 (talk) 23:29, 10 September 2008 (UTC)[reply]
Independent, intelligent women pay a price in modern US society. Most men will not put up with it and make such clear in their actions if not always their words. As a result the vast majority of women are socialized into being quiet, submissive, non-threatening. I find it pretty disgusting (and damning of this so-called "free" society). My wife is independent and intelligent and has had to put up with a lot of crap from men over the years. --98.217.8.46 (talk) 23:59, 10 September 2008 (UTC)[reply]
The OP is talking about women being submissive to their (presumably female) friends, not to men. If women in the US are quiet, submissive and non-threatening, then I think I should probably move there - the women in the UK are far from it! --Tango (talk) 00:05, 11 September 2008 (UTC)[reply]
Men actually do the same thing, in their own way. In certain male-only social circles, it can be seen as unmanly and disrespectful to put one's wife or girlfriend (especially a recent one) before one's buddies in terms of social contact and free time allocation. It's the whole (to be blunt, 'cause this is what's said) 'Hos come and go but your bredren are forever'/'Bros before hos' thing... --Kurt Shaped Box (talk) 00:19, 11 September 2008 (UTC)[reply]
I've actually noticed the opposite in quite a few women I know or know of. The amount of crap they're prepared to put up with from guys who are obviously dyed-in-the-wool dirtbags who'll never change astounds me sometimes. Occasionally, when all the cheating/excessive drinking/laziness/disappearing for days on end with no explanation/spending her money or pawning her jewellery without asking becomes too much for her, she'll sling him out on his arse - but it seems like nine times out of ten, she'll take him back after a few days, only for nothing to change at all. I don't know of many (any?) men who'd accept that sort of behaviour from their girlfriends or wives. --Kurt Shaped Box (talk) 00:13, 11 September 2008 (UTC)[reply]
I was mostly talking about girls that are 15-25, I know they become more mature and make better decisions as they "grow up."
Well then, you solved your problem yourself, just wait some years or try to get in touch with older women. --Ayacop (talk) 07:28, 11 September 2008 (UTC)[reply]
Social contacts are important to both men and women. Also, if other people find another person attractive for whatever reason, this tends to increase the perceived 'value' of this other person, and it doesn't matter whether we're talking about males or females. There is always a degree of competition involved in these sort of things, and for better or worse, someone is far more likely to persue a person other people find attractive and for that matter, find the other person attractive. Finally, I don't see any reason to presume these people aren't living for themselves. Their friends are obviously going to be important to them... Why should this man, who doesn't get along with their long term friends be more important to them then their long term friends? Of course it could be the friends at fault, but people are apt to take the side of the person they have known for longer and often miss some of the flaws in the people they know well. Besides that, if the man they're dating is talking about or thinking about "the fat friend that cant get guys" I would say it is actually the man who is at fault, and good on the girl for dumping that jerk. It's likely to be not in their interest to date such a jerk. To put it a different way, if the girl is happy with her life and friends, how can you say they are not living for themselves just because they choose to reject those who don't fit in with their existing life? Of course, as life priorities change, the importance of friends may change and also an increasing priority for finding a stable long term partner (as opposed to simply one that looks good to other people) but I don't think it's that simple. The people they are dating also change/mature and perhaps are less likely to think about "the fat friend that cant get guys" and also, the person herself is less likely to even be interested in such a guy in the first place. In other words, both parties are maturing and changing, you can't just put it down to one. P.S. As others have said, most of this holds true in some way for men as well. Most men actually do care a lot about what other people think of other any girl their interested and are far more likely to pursue someone perceived as hot then someone who is not... And a lot of men are not going to be involved for very long with someone who does not get along with their friends (the may perhaps keep the relationship for a bit longer then would most females if they're at least having sex). Perhaps the biggest difference is that I suspect, for a variety of reasons, a man is far more likely to not get along with a woman's friends then a woman is with the man's friends. Nil Einne (talk) 17:20, 11 September 2008 (UTC)[reply]

Telephone

I remember using a telephone which had a handle one cranked in order to contact the (always a woman at the) local exchange. What was it called? Kittybrewster 23:06, 10 September 2008 (UTC)[reply]

A telephone with a hand-crank
I don't see a specific name for a telephone with a signaling crank, though the contemporary separate-transmitter-and-receiver design was known as a "candlestick". — Lomn 23:28, 10 September 2008 (UTC)[reply]
A search on Google indicates the present term is "hand crank wall telephone." In the days when they were in common use, they were probably just calld a "wall phone." In an 1899 book on telephones "American Telephone Practice" by Miller, a drawing of a wall phone with a crank is just titled "Complete telephone instrument" while the candlestick phone illustration is labelled "Desk telephone set."Edison (talk) 03:24, 11 September 2008 (UTC)[reply]
You're terrific. Thank you. Kittybrewster 03:30, 11 September 2008 (UTC)[reply]
A a child when I heard on tv cop shows about someone receiving "crank telephone calls," I supposed that they must have been made from one of these old instruments. Edison (talk) 18:43, 11 September 2008 (UTC)[reply]
I don't know the etymology, but crank call is used as another name for prank calls. -- MacAddct  1984 (talk &#149; contribs) 05:54, 12 September 2008 (UTC)[reply]
I'm more inclined to the belief that rather than the instrument, it is the caller being referred to: crank (Old English, cranc) ...an eccentric person...; cranky ...full of whims, cross. Hey, there's even a picture of me in the dictionary! :)Franamax (talk) 09:04, 12 September 2008 (UTC)[reply]
My reasoning sprung from my grandparents having phones which hung on the wall and had a crany, and their being frequently cranky. Edison (talk) 01:10, 13 September 2008 (UTC)[reply]

Magnifier

Why magnification related to recpirocal of lens diam? —Preceding unsigned comment added by 79.76.128.237 (talk) 23:21, 10 September 2008 (UTC)[reply]

It will be related to the curvature of the surface of the lens. A small lens can curve at a much faster rate. Graeme Bartlett (talk) 02:57, 11 September 2008 (UTC)[reply]

Melting Some Silver

Ok, Wikipedia, here we go. The chemistry class (highschool level) I am aiding for recently did the "Cu wire in silver nitrate solution makes crystals of silver and Cu solution" experiment. The teacher then had an idea to scrape the silver off of the copper wire and melt it somehow. We scraped it off and we have this pile of nitrate soaked silver to work with. We are both wondering whether a methane flame will be enough to get a crucible up to 1234 K or so to melt the stuff. Here's how I'd do it, gathered from some web surfing: 1. Get more silver from the nitrate solution 2. Dry lump of stuff in the crucible first 3. Get it out of the crucible and melt some borax in the crucible (to give the silver a slippery surface to slide around on so that we can pour it out easily? a good idea?) 4.Put the silver back in the (cooled) ceramic crucible, add flux (I read this is good to have to prevent dissolved gas in the melted metal...what is it?) and but the spurs to it.

Anybody have corrections/ improvements on my procedure?72.219.139.201 (talk)

DMacks (talk) 00:31, 11 September 2008 (UTC)[reply]

Here's some good ideas! My try will be on a smaller scale than that, but the idea of stoking a flame with the blower helps me out...hmm I don't think it would help a methane flame but maybe some mini-charcoal action is due.72.219.139.201 (talk) 05:01, 11 September 2008 (UTC)[reply]

Feline parasite follow-up

So I managed to grab one of the buggers. Any ideas? seems pretty flea-like to me. This one was actually a little larger than the rest of them, ~3mm. Sorry for the poor quality, I don't have a good macro lens and I stuck this guy between packaging tape.

Yeah, that's a cat flea. Nasty things. Check with the vet about giving a kitten the anti-flea stuff, but if he or she is cool with it, really, the Advantage/Frontline stuff is super easy (you put a little squirt of it behind their neck and pow! no fleas for a month). --98.217.8.46 (talk) 00:01, 11 September 2008 (UTC)[reply]
I'm told you should avoid touching the cat for a day after dosing. —Tamfang (talk) 17:01, 14 September 2008 (UTC)[reply]

salt grain physics.

When I was a child , I wondered what would happen If a grain of salt had all of its atoms pushed together so that they all touched.

Naturally there was no internet/physics books in the house/or an expert who could answer this at the time so I never found out.... —Preceding unsigned comment added by 78.149.104.222 (talk) 23:54, 10 September 2008 (UTC)[reply]

"Touched" doesn't really make sense, atoms are mostly empty space so there isn't much to touch. You may, however, be interested in degeneracy pressure. --Tango (talk) 00:01, 11 September 2008 (UTC)[reply]
Under extreme pressure the molecular bonds such as those in a salt crystal breaks. The bonds between electrons and the nucleus in atoms may also break and you may get matter similar to what a Neutron Star is composed of. It may become a superfluid but we're not really sure because we can't create these immense pressures in a lab. EverGreg (talk) 09:29, 11 September 2008 (UTC)[reply]
Tangentially, this reminds me of something Sir James Jeans said: "Put three grains of sand inside a vast cathedral, and the cathedral will be more closely packed with sand than space is with stars." --- OtherDave (talk) 15:46, 11 September 2008 (UTC)[reply]
This does not apply to the center of some globular star clusters or even of our galaxy. --Ayacop (talk) 15:59, 11 September 2008 (UTC)[reply]
Are you sure about that? The back of the envelope calculations I just did show the cathedral would need to be several square kilometres in size to have the same density as the central parsec of our galaxy. --Tango (talk) 18:28, 11 September 2008 (UTC)[reply]
I wasn't the one who said it; Jeans was. He didn't specify which chunk of space. And I did say it was a tangent. --- OtherDave (talk) 10:34, 12 September 2008 (UTC)[reply]


(from op)

So assuming we only have the mass of a salt grain and we pushed the nuclei together [somehow :) ] (overcoming the strong force) would we get a mini black hole or would it flash out as an energy burst ...what would happen? —Preceding unsigned comment added by 84.13.83.29 (talk) 19:45, 11 September 2008 (UTC)[reply]

If you overcome the force holding the electrons apart, you get something similar to a neutron star. If you carry on and force the nuclei together it will eventually collapse into a black hole which would very rapidly evaporate via hawking radiation. --Tango (talk) 22:31, 11 September 2008 (UTC)[reply]


September 11

But why the square head?

The bizarre Tibetan fox, the pica's nemesis.

But why the square head? --Kjoonlee 00:28, 11 September 2008 (UTC)[reply]

In the article Tibetan Sand Fox, ""The Tibetan Sand Fox has a unique face that appears square; this is an illusion created by its large ruff." So there ya go. Julia Rossi (talk) 08:01, 11 September 2008 (UTC)[reply]

But why the large ruff? --Kjoonlee 19:49, 11 September 2008 (UTC)[reply]
The large ruff is necessary to maintain the illusion of a square head. Wanderer57 (talk) 20:09, 11 September 2008 (UTC)[reply]
A mission shared by Henrik Ibsen and Richard Wagner here[11] among others. Julia Rossi (talk) 22:46, 11 September 2008 (UTC)[reply]

Parrots - mistrustful of humans when nesting...

Just something that came up in conversation with a friend today. Why is it that a tame female parrot might be perfectly happy to trust you with her life and safety as her owner/companion, yet completely refuse to trust you when it comes to the safety of her eggs and chicks? Yes, even tame hens generally hate it when you go near their nests (say to count eggs or periodically check on the wellbeing of the young) and go into full-on 'angry protective mom' mode... --Kurt Shaped Box (talk) 00:33, 11 September 2008 (UTC)[reply]

This is not confined to parrots. There are many instances of birds developing an extra protective attitude when nesting. Blackbirds, normally timid, can be very aggressive towards anyone who comes too close to their nest, especially when they have nestlings. And swans, while docile and amenable for the rest of the year, can turn into mean critters in the nesting season. Lapwings (green plover, peewit)develop distractive behaviour during nesting to lure away potential predators of its eggs or young. The latter behaviour, while not aggressive, indicates the ability of some birds to alter their behaviour according to the circumstances. This is presumably an instinctive behaviour to protect its progeny. 86.4.187.55 (talk) 06:20, 11 September 2008 (UTC)[reply]
A completely different hypothesis is that they are pretending when you think they trust you. And most of the time, you're anthropomorphizing, anyway, meaning that behaviour in presence of humans all is learned in order to get at food etc. --Ayacop (talk) 07:36, 11 September 2008 (UTC)[reply]
I think anthropomorphism is definitely a big part of the picture. These are birds, not humans, and they aren't very smart. They don't think, "oh, hey, that nice guy feeds me and pets me, so I can trust him with my eggs", because they aren't really thinking in logical chains like that. I mean, parrots are pretty smart birds, but they're only smart by bird standards -- that doesn't mean they're capable of abstract thought. -- Captain Disdain (talk) 08:35, 11 September 2008 (UTC)[reply]
I don't really think that it's anthropomorphizing too much to suggest that a psittacine which remains totally calm and placid when picked up by its owner and cuddled, or rolls over and allows its belly to be scratched, or willingly places its head inside a human mouth to pick food from its owner's teeth (note: don't do this - our mouth bacteria can make birds very ill), or chooses to sleep snuggled up against its owner's head at night 'trusts' (and perhaps even 'likes') that one certain human. Are they smart enough birds to fake all that for the sake of food? --Kurt Shaped Box (talk) 17:31, 11 September 2008 (UTC)[reply]
I'm not saying that they're faking it. That's kind of the point, actually! They just aren't smart enough to make the logical connection that would allow their trust for you to overcome the instinct that tells them to protect their eggs, even though they have genuine affection for you. That kind of thing would require pretty advanced capacity for abstract thought, and they don't have it -- either that, or pretty serious conditioning. -- Captain Disdain (talk) 22:43, 11 September 2008 (UTC)[reply]
I don't think it's confined to birds either. Many mammals are far more wary of others including humans when they are taking care of their young. Nil Einne (talk) 17:00, 11 September 2008 (UTC)[reply]
We (my friend and I) were discussing the differences in behaviour between pet parrots and pet cats/dogs, which don't really seem to mind when familiar humans interact with their young - even when they pick them up and play with them. On the other hand, something like a pet cockatoo or cockatiel (my friend has some) would go *absolutely fucking ballistic* if you tried the same thing with its chicks. --Kurt Shaped Box (talk) 17:31, 11 September 2008 (UTC)[reply]
I don't think that is entirely true. Many mother cats and dogs will be concerned if you try to handle their kittens/puppies particularly when they are very young. They may either react against you (snarl/hiss/try to bite/scratch) or may move them around to try and get away from you (or in extreme cases, may reject the young). Birds may perhaps be more protective then cats/dogs, but I definitely think all are quite protective of their young. (The obvious question is, how many mother cats/dogs and birds have you interacted with? In my case, none at all really but from what I've read I do think it's the case) It's worth bearing in mind that cats & dogs have both been domesticated for a while now whereas most birds haven't really been domesticated as far as I know so comparing the two isn't perhaps a fair comparison. From what I've seen in shows, you have to be very careful for example, with tigers when they've just given birth, even if you regularly handle the tiger normally. Also, I suspect it will have a fair bit to do with whether the animals in the wild are social animals. If it's normal for their young to interact with others at a fairly young age (as I presume it is for wolves) then they're less likely to have a problem with it. This is not the case for most birds (or tigers) so they're more likely to have a problem with it. P.S. What do chickens do? Nil Einne (talk) 18:09, 11 September 2008 (UTC)[reply]
In my experience, chickens just try to run off and lead the rest of their chicks away from you. They work on more of a numbers approach, rather than defend every single chick. Ducks on the other hand can be a little more aggressive. Franamax (talk) 08:48, 12 September 2008 (UTC)[reply]
Having spoken to my grandmother (who was raised on a farm), it's apparently the cockerel that tends to be the aggressive one when there are chicks wandering around. The hens (as Franamax says) just scuttle away with their chicks following behind. When it comes to collecting eggs, the hens will obviously bite if an attempt is made to remove eggs from beneath them, but they don't seem to even notice it happening if they're not actually on the nest. Again, the cockerel does, though - and gets very angry. --Kurt Shaped Box (talk) 22:59, 14 September 2008 (UTC)[reply]

Cat Induced Allergies

About 2 and a half months ago we got a 5 year old white Persian cat. It has no ticks and flea infestation and we bathe it regularly about every week or 2 weeks as needed. Basically it is a healthy cat.

Recently about a week ago my Dad started complaining about allergy to the cat. But I believe that he is overacting. He is complaining of flu and throat complications. And I am of the opinion that this could very easily be due to viral or bacterial infection. My younger brother has allergic problems but throughout the 2 months stay of the cat he had no allergic reactions but about a week ago when his college opened he started having nose irritation etc. And that of course is because he now regularly breathes polluted (smoke, dust etc) air when traveling in the bus, at least that is what I believe.

On the basis of the above my Dad is insisting that we part with the cat. I am of course opposed to the idea. I have read somewhere that living with a pet cat can actually make your immune system resistant to cat allergies. Therefore I am of the view that the cat should stay.

I am not asking for medical advise but rather I would like to know if any of you have undergone a similar experience and the decisions that you made. Because a majority of people have pets or had them at some point in life. —Preceding unsigned comment added by 203.81.205.162 (talk) 00:50, 11 September 2008 (UTC)[reply]

You're trying to make a decision without data. An allergist would be able to perform a test that would tell whether your father is actually allergic to the cat, or not. It's unreasonable of him to insist you get rid of a pet if he doesn't take the test, just as it would be unreasonable of you to insist on keeping the pet if the test determines he's actually allergic. - Nunh-huh 04:25, 11 September 2008 (UTC)[reply]
An allergist could also explain more exactly the factor that triggers reaction in sensitive people. Mostly it's taken to the be fur, but there's a range of comment/information on Google about the effect of cat spit on the fur from grooming[12] among other things. Julia Rossi (talk) 07:57, 11 September 2008 (UTC)[reply]
Actually as far as I'm aware it's predominantly due to a? protein in the saliva which get onto the fur from the cat licking (and perhaps other means). This seems to be supported by Cat allergy. Nil Einne (talk) 16:57, 11 September 2008 (UTC)[reply]
Yeah, have them see an allergist to figure out for sure what is ailing them. But you should probably work out the terms ahead of time—if it turns out they aren't allergic to cats, keep the cat, but if they are, consider finding it another home. Don't give it to a shelter unless you are fine with the fact that it will probably be put to sleep. --98.217.8.46 (talk) 01:24, 12 September 2008 (UTC)[reply]
I used to believe that cat shelters meant that until I found out otherwise, and further OR is that while I had cats with short hair, it didn't affect the household, but a persian did with its excessive moltability. The cat had such a nice personality I got it a job in a pet store as the cusstomer service kitty, along with the tame parakeet etc. All true and a great itch relief for the rest, Julia Rossi (talk) 08:32, 12 September 2008 (UTC)[reply]
That's true, I found a Siamese to be the only breed of cat I could tolerate in a house. I like cats (and they like me) but a cat allergy is truly miserable. There is another way for all concerned to get along - get rid of the carpets, vacuum the floors and furniture every day, brush the cat every day and wash it regularly (nyuk - just try washing a cat), dust the shelves and tables every three days, and make sure Dad has an area where the cat is never allowed to go (den and bedroom, for instance). If you really want to, it can be done. Franamax (talk) 11:36, 12 September 2008 (UTC)[reply]

What is a Computer Scientist?

My 10yr. son has to do research so a Computer Scientist. I really need some basic information so I can help him. What's involved in this person job? what type of equipment does this person need? Does this person need a uniform to do his/her job?

Thank You......Ms. Demetrius —Preceding unsigned comment added by 71.251.53.33 (talk) 00:56, 11 September 2008 (UTC)[reply]

Start with our article about computer science. It's a pretty diverse field, ranging from engineering to electronics to programming. One might sit at a desk, work in a clean room, etc. Depending on what one does, one might wear a suit, jeans, a uniform, some sort of safety gear or protective covering over "whatever" you want... DMacks (talk) 01:02, 11 September 2008 (UTC)[reply]
Some say that Edsger W. Dijkstra said "Computer Science is no more about computers than astronomy is about telescopes," giving a clue that computer scientists have been around longer than computers have. Computer scientists were around in the 1800s, as Ada Lovelace was the first computer programmer. --Kjoonlee 01:34, 11 September 2008 (UTC)[reply]
Also the first victim of vaporware. Clarityfiend (talk) 06:29, 11 September 2008 (UTC)[reply]
The article Computer scientist tries to explain it. Googling the term "What is a computer scientist?" (keeping the " ) will give you some other attempts at explaining it which are perhaps written better. EverGreg (talk) 09:36, 11 September 2008 (UTC)[reply]
He might try focusing on a small number of real computer scientists as an example. Donald Knuth comes to mind. Or Dijkstra. Or possibly compare a naive solution to a problem to a more advanced solution developed by computer scientists. (The obvious Bubble sort verses the unintuitive Quicksort for example) APL (talk) 18:17, 11 September 2008 (UTC)[reply]
Knuth may be a little above a 10 year old. Maybe try something your son would be more interested in, like game programmers. Paragon12321 03:22, 12 September 2008 (UTC)[reply]

Petrol pumps

In an article in the London Times today, Matthew Parris complains of an incident when he drove up to a petrol pump on the wrong side. As he pulled the hose over the car's roof and attempted to fill up the tank that way, a loudspeaker voice bellowed at him that this was not allowed, and to drive to the other side or she would not switch on the pump.

As flammable liquids are involved I'm tempted to give the cashier the benefit of the doubt. Is there, in fact, any danger in pulling the hose of a petrol pump over the roof of your car and filling up from the wrong side? 81.171.134.226 (talk) 12:01, 11 September 2008 (UTC)[reply]

When I lived in California, there were stations that had signs advertising that their hoses were long enough that you could fill from either side. -- Coneslayer (talk) 12:37, 11 September 2008 (UTC)[reply]
I suppose one might scratch the paintwork, but I can't see any other likely hazards. DuncanHill (talk) 13:32, 11 September 2008 (UTC)[reply]

Sounds like a classic jobsworth. At the worst it'd be a little bit extra wear and tear on the car. People do this all the time (at least in my experience) without concern, and i'm pretty certain if there were any meaningful additional danger that the petrol-station would have clear signs about it (after all the relative danger of being on your mobile phone in a petrol station is small but they still - for now - forbid it.) 194.221.133.226 (talk) 14:08, 11 September 2008 (UTC)[reply]

thank you for expanding my knowledge of British English by using jobsworth! --LarryMac | Talk 14:49, 11 September 2008 (UTC)[reply]
Ooh me too! --jpgordon∇∆∇∆ 15:54, 11 September 2008 (UTC) [reply]
There's no danger to the pump - it is designed to pump wherever the hose is pulled to. There is possible danger to the customer being on the side of the car that traffic drives past as opposed to being between the pump and the car. In all reality, it is most likely a dumb rule that was made after someone sued the station (or company) for scratches on his or her car. -- kainaw 14:19, 11 September 2008 (UTC)[reply]
I'm told the smart thing to do is look at the little fuel pump symbol on the dashboard before you get out of the car - many indicate which side the filler cap is. Bazza (talk) 14:41, 11 September 2008 (UTC)[reply]
Snopes disagrees. Fribbler (talk) 14:54, 11 September 2008 (UTC)[reply]
That's not what Bazza means. Many pump symbols include an explicit arrow pointing left or right (my MINI does, and so do most rental cars I rent). Here's an example on a Toyota (quickest one I could find). -- Coneslayer (talk) 15:07, 11 September 2008 (UTC)[reply]
It actually mentions that in the Snopes article. Whoops! :-) Fribbler (talk) 15:19, 11 September 2008 (UTC)[reply]
So that's what the little triangle on the dashboard of my Toyota is for! 81.174.226.229 (talk) 15:42, 11 September 2008 (UTC)[reply]
I didn't mean an arrow, I did mean the side the pipe is positioned, as described in the Snopes article. The car I drive (VW) does what I said and, from 81's comment above, so do others. Different makes may not. Both Snopes and I, therefore, can be considered correct, especially if I pedanticly change "many" to "some" in my note above. Bazza (talk) 15:46, 11 September 2008 (UTC)[reply]
But since it's basically a fifty-fifty chance of being right, that's not really significant. That you are correct for your car is nothing more than confirmation bias. — Lomn 16:20, 11 September 2008 (UTC)[reply]
(ec) On many cars one can tell which side the filler cap is on by looking at the car before getting into it. DuncanHill (talk) 15:49, 11 September 2008 (UTC)[reply]
Just go back in time maybe 25 years or so, when many cars had the fill-pipe centered in on the rear. The license plate, which was on the fixed frame not on the trunk-lid, was the cover. DMacks (talk) 15:56, 11 September 2008 (UTC)[reply]
Not in Britain they didn't! I had a "solve it yourself" mystery book when I was a boy. The vital clue in one of the stories was to do with Fords having the filler cap under the number plate - now we had a Cortina, and I knew damn well that the filler cap was on the side. Being unable to solve the mystery because the publishers had failed to adapt the book properly to the British market was a great disappointment to me. DuncanHill (talk) 16:00, 11 September 2008 (UTC)[reply]
That was Wikipedia Brown and the Global Perspective, right? I think I read that one. -- Coneslayer (talk) 16:20, 11 September 2008 (UTC)[reply]
I don't think there will be any danger but it sounds to me there might be some reasons why they won't want you doing it. Firstly if the hose isn't far enough to reach the tank, the person may try to stretch it to reach. This is unlikely to be dangerous but may increase wear and tear on the hose. Secondly if the person is extremely lazy, they may not bother to put the hose back properly and may just leave it lying across the road. Again I doubt there will be any danger but it may be ran over (again increasing wear and tear) and/or requires the attendant to go and put it back. Thirdly if someone knocks the hose it may be damaged and of course is likely to damage the car that hits it (again probably not dangerous but may liable to make problems for the petrol station). Nil Einne (talk) 16:49, 11 September 2008 (UTC)[reply]
Take a look at that picture of the gas pump up above. The hose is partly filled with gasoline from the last person who filled their car up. If you're trying to stretch the hose over to the other side of your car and you lift it the wrong way, isn't there a fairly good chance you'll spill gas all over your car (and yourself)? Franamax (talk) 08:41, 12 September 2008 (UTC)[reply]
Nahh, the shutoff valve in the handle is supposed to prevent that. There's possibly a small amount in the last 8 inches of nozzle, but only if the last person pulled it out REALLY quickly after refueling. --DaHorsesMouth (talk) 22:39, 12 September 2008 (UTC)[reply]
Perhaps the reason that it is unallowed is that if the customer is standing on the far side of the car, they won't be able to reach the shutoff lever on the pump in an emergency. I've stretched the hose over my car once (and mine's just a narrow Corolla hatchback) and I noticed that the extra twisting and manipulations I had to do to get the nozzle into the right position resulted in some spilled petrol when I removed it. I find it curious that the Snopes article thought that putting the fuel cap on the same side of all cars would be a good idea, given that it would make it mean you couldn't have a car on each side of one pump. Maelin (Talk | Contribs) 09:41, 15 September 2008 (UTC)[reply]

polymers

Under what condition does ethzlene gets converted into polyethylene?220.225.211.171 (talk) 12:33, 11 September 2008 (UTC)[reply]

Depends on if you mean LDPE or HDPE.--Stone (talk) 12:39, 11 September 2008 (UTC)[reply]

Is the LHC the coldest place in the universe?

The temperature of intergalactic space is 2.73 K. The operating temperature of the LHC is 1.9 K, and its volume is around 80000 cubic metres. Is there any known region of the universe of this size that is colder than the collider? --Taejo|대조 13:20, 11 September 2008 (UTC)[reply]

According to our article, the Boomerang Nebula is at about 1K. It's the only thing known to astronomers that's colder than the microwave background. Algebraist 13:23, 11 September 2008 (UTC)[reply]
Do astronomers not know about the LHC? :) --Taejo|대조 13:34, 11 September 2008 (UTC)[reply]
Algebraist intelligently included "known to astronomers" because it is impossible to know the "coldest" or "hottest" or "largest" or "smallest" anything when referring to space. Most of it is unknown, so we can only answer with "coldest place in known space". And yes, astronomers know about the LHC. It is important to know about something that is guaranteed to suck up Earth in a black hole by this time next year. -- kainaw 14:16, 11 September 2008 (UTC)[reply]
Well, that's why I thought astronomers would know about the LHC. --Taejo|대조 16:01, 11 September 2008 (UTC)[reply]
Well, there are labs where they've got down to billionths of a degree above absolute zero, if memory serves. The Boomerang Nebula is the coldest known place that wasn't cooled down by man. --Tango (talk) 14:23, 11 September 2008 (UTC)[reply]

Population of the Philippines

COULD YAH PLS. GIVE ME THE POPULATION OF THE PHILIPPINES, CAINTA & RIZAL.....TNX?Jjhg (talk) 13:53, 11 September 2008 (UTC)[reply]

Our article Philippines should contain the information you are looking for. By the way, this sort of question is probably better suited to the Humanities desk. Oh, and try not to use ALL CAPITALS when asking questions - it can look like you are shouting and may annoy some users. DuncanHill (talk) 13:58, 11 September 2008 (UTC)[reply]

Welding under water

What are the processes taken to weld under water? Discuss the differences between welding on surface and under water? What is the trick behind under water welding? What are the equipments used? Are the equipments the same as surface welding? What are the side effects on welder? Which is stronger surface or under water welding? What is the meaning on the abbreviation written on surface electrode? Does the carbon light has effect on eyes as in under water welding? What are the conditions for a surface welder to become an under water welder? Lastly Every other things describe under water welding

Thanks —Preceding unsigned comment added by Advancepunche (talkcontribs) 14:21, 11 September 2008 (UTC)[reply]

Try reading underwater welding and ask if any of your questions are not answered. -- kainaw 14:25, 11 September 2008 (UTC)[reply]

Entropy and the Direction of Time

In the dimension article, it states that "we perceive time as flowing in the direction of increasing entropy." Why is that true? Sappysap (talk) 14:55, 11 September 2008 (UTC)[reply]

As noted prior to that parenthetical expression, it's because physics models generally treat time in that fashion. In a closed system, entropy increases (per the second law of thermodynamics). This is a convenient shorthand for time with regard to otherwise time-symmetric equations. The statement should not be interpreted in isolation, however -- it is not an attempt to say that you and I examine the entropy around us to determine that time exists in the everyday. — Lomn 15:21, 11 September 2008 (UTC)[reply]
It's interesting to note that the 2nd law of thermodynamics, or at least the logic behind it, is time symmetric. Entropy ought to increase in both directions in time from any given point (which isn't actually possible, of course, so I guess it would all just cancel out and stay roughly constant), it's just because the big bang had extremely low entropy, so entropy in the past is fixed at a low point, that it can only increase towards the future. --Tango (talk) 17:20, 11 September 2008 (UTC)[reply]
Try this thought experiment. Suppose you and the whole universe suddenly start to move "backwards in time", all physical processes are exactly reversed, and your memories are erased as you go back. So when you get "back" to one hour ago you have exactly the same memories as you had the first time you went through the time-point "one hour ago", the only difference is that you (and the universe) are now moving in the opposite direction along the "time dimension". How would you know that time had reversed ? How could you tell ? Does this concept of "moving backwards in time" even make sense ? In The End of Time: The Next Revolution in Our Understanding of the Universe Julian Barbour argues in this way that time is an illusion, and that the only reason we peceive time as flowing in the direction of increasing entropy is that this is the only way in which memory can work, given the second law of thermodynamics. It is impossible to remember the future, not because "it hasn't happened yet", but because it is a state of higher entropy than the memory itself. Gandalf61 (talk) 15:25, 11 September 2008 (UTC)[reply]
We have some articles on arrow of time and entropy (arrow of time), although they can be kind of confusing. --Bennybp (talk) 16:36, 11 September 2008 (UTC)[reply]
Shouldn't the psychological arrow of time be pointed in the opposite direction? Being as objective as I possibly can, I'd like to state that my thoughts are far more ordered now than they were when I was 13. Sappysap (talk) 23:36, 11 September 2008 (UTC)[reply]
I don't think you can apply your thoughts in terms of entropy. Anyway your brain isn't a closed system. --Mark PEA (talk) 12:08, 12 September 2008 (UTC)[reply]
Indeed - it's not a closed system, and that's the key detail. In order to store a memory you need to use energy and that's going to increase entropy somewhere (heating the air around you, turning food into carbon dioxide and water, etc). --Tango (talk) 21:27, 12 September 2008 (UTC)[reply]

Are all alleles genes?

Are alleles genes or do they just determine how the gene turns out? Thanks Kim —Preceding unsigned comment added by 71.176.185.204 (talk) 15:08, 11 September 2008 (UTC)[reply]

All Alleles are genes. And all Genes are made of DNA. "How the gene turns out" is referred to as the phenotype. The genes themselves are physical DNA sequences and don't "turn out" to be anything else. Fribbler (talk) 15:16, 11 September 2008 (UTC)[reply]

Disadvantages of needleguns

I added a second question below on September 14. It's been two days, and no one's taken a stab at it, so I've boldfaced it. I will de-boldface it once someone answers.Lowellian (reply) 07:12, 16 September 2008 (UTC)[reply]

In Wikipedia's article on needleguns, there is a section heading entitled "Advantages" which gives a whole bunch of advantages of needleguns. There is no section heading called "Disadvantages", and no discussion of disadvantages in the article. Yet, clearly, there must be significant disadvantages, because if there weren't, then everybody who uses guns would be using needleguns — while in reality, the opposite is true: needleguns are extremely rare in comparison to other types of firearms. So, what are the disadvantages of needleguns, and could someone please add them to the article? —Lowellian (reply) 16:21, 11 September 2008 (UTC)[reply]

I've heard of flechettes, but didn't know people used the term "needlegun" to refer to such things. I'd assumed "needleguns" were sci-fi only. High-velocity, high sectional density projectiles would tend to overpenetrate, which is a disadvantage. But, this is just me speculating, and I'm not a reliable source, so I don't think I'd want to stick this into the article. Friday (talk) 16:35, 11 September 2008 (UTC)[reply]
Well, I would guess the major disadvantage is that they are mostly hypothetical. I'm not aware of an existing system. Talking about handguns, hypothetically of course, the major disadvantage would be the low stopping power. Needles penetrate well, but do little damage. The overall energy of the projectiles would also be rather low, implying low range. Also, it's unclear of how to power such a gun. Chemical with a sabot would eliminate most advantages (rate of fire, ammunition size) while adding complexity and inaccuracy (the sabot needs to be accelerated and drops away in flight). For more Sci-Fi approaches, you would need a portable power plant, which might not be reasonable. --Stephan Schulz (talk) 16:42, 11 September 2008 (UTC)[reply]
Why are they hypothetical? Why couldn't you just replace a bullet with a flechette in a normal firearm and get a flechette rifle?Lowellian (reply) 16:04, 14 September 2008 (UTC)[reply]
A "needle" gun actually doesn't make a very effective weapon, as the small mass of a needle slows down much more rapidly in flight than a bullet and hence has limited range and accuracy. Most flechettes actually look more like nails than needles, and that's largely to provide them enough mass to be practical. Dragons flight (talk) 21:03, 11 September 2008 (UTC)[reply]

Was there German Genetic Engineering using X-rays in 1945?

After World War II, in the years 1945 to 1947 the U.S. took besides many factories all intellectual property that they could from Germany, all patents, the industrial processes etc. (good book on the topic) A U.S. agricultural officer (colonel) in occupied Germany wrote a book about his experiences in occupied Germany, it is available online. This is a quote about one of the things they took in 1945.

"Members of our agricultural staff were in time to discover some rather interesting and intriguing developments in plant science, especially about the production of higher-yielding plants by breaking up certain chromosomes through radiation. Some of this got to the United States and is widely applied in areas of grain production and horticultural research today."[13]
  • Is he speaking about a form of Genetic Engineering by inducing mutation using x-rays, or is it be something more advanced?
  • What is this method/technology that the Americans copied from the Germans called?

--Stor stark7 Speak 17:01, 11 September 2008 (UTC)[reply]

The use of a large variety of mutagens [14] to help generate varieties is extremely common place in modern day plant breeding (check the article) and has been since around the end of WW2. Take a look at this image for example [15]. No, it's not usually considered genetic engineering. However it is rather different from what many people seem to have in their minds (particularly those nuts about organic farming who hate anything involving GE) about someone breeding different varieties they find until they get a plant they want. Nil Einne (talk) 17:50, 11 September 2008 (UTC)[reply]
The systematic use of radiation to induce mutation was done far earlier than World War II. Thomas Hunt Morgan was experimenting with radiation as a way of producing different varieties of fruit fly as early as the 1910s. I don't know precisely who started using it for specifically agricultural purposes but the fact that mutations in large populations could be increased with radiation doses had been figured out pretty early on, and I would not be surprised if it was one of the many approaches taken at agricultural stations in the US pre-WWII, but that's just my own speculation. It's not necessarily the best way to get new varieties, though. --98.217.8.46 (talk) 23:28, 11 September 2008 (UTC)[reply]

Over my head

I have a teacher who has a habit of looking over everyone's head while he's teaching class. He looks somewhere above the windows in the back of the room. I used to know a guy that did this whenever he was talking to someone. He'd never look you in the eye while talking. Is there a term for this? Dismas|(talk) 17:44, 11 September 2008 (UTC)[reply]

I don't think there's a term for it, since it could be any of several unrelated reasons someone may appear to do this. Two social issues come to mind are nervousness (feeling awkward making eye contact) and some sort of aloofness/snootiness (looking down at someone while talking). OTOH, could be a vision problem: a co-worker years ago always looked like he was looking a few feet to the left of where he actually was looking, which if you didn't know it, you'd always think he was ignoring you and listening/talking to the person two chairs over. DMacks (talk) 17:58, 11 September 2008 (UTC)[reply]
I typically don't make eye contact when I talk (and teach as a TA). It's something I'm working on, but never felt like doing. For me, I believe it's because I'm a strong auditory learner - I'm not actually "looking" at or even "focusing" on anything, I'm just devoting more of my brain to hearing than seeing and more or less "shutting down" my eyes for a bit. I don't really need to look at faces/body language - I get almost all of my cues from the tone, inflection, etc of voices. I know it's distracting to some people, but it's who I am. If you force me to look at you, I'll probably forget what you said. Actually, that might even be some very mild autism, but I've never been tested. (I know that didn't actually answer your question for a term, just something I'd like people to know :) --Bennybp (talk) 19:12, 11 September 2008 (UTC)[reply]
Thanks, now I'll know in case I'm in one of your classes.  ;) Dismas|(talk) 20:45, 11 September 2008 (UTC)[reply]
Actually, I think the term(s) you are looking for are eye aversion or gaze aversion. See Eye_contact#Eye_aversion_and_mental_processing and a kind of interesting paper (although maybe unpublished) here. Warning: PDF --Bennybp (talk) 19:19, 11 September 2008 (UTC)[reply]
I always avoid eye contact. Not out of rudeness, but due to poor vision, as mentioned by user:DMacks. Because of my vision I find facial expressions hard to interpret, so avoiding eye contact allows people to know that I don't register their expressions. Fribbler (talk) 22:49, 11 September 2008 (UTC)[reply]
Asperger syndrome may be of interest. —Preceding unsigned comment added by 98.169.163.20 (talk) 00:12, 12 September 2008 (UTC)[reply]

Facts that oppose evidence for evolution

No, this is not a question about intelligent design. I just want to know if some facts seems paradoxal confronted with the idea of evolution. Something that cannot be easily explained considering evolution. Mr.K. (talk) 18:37, 11 September 2008 (UTC)[reply]

The primary argument against evolution is the complexity of life. Here are a few ways of putting it:
  • How could the human eye just 'evolve'?
  • Flight would be incredibly useful. Why can't humans fly?
  • Shouldn't there only be one super-evolved creature instead of all different kinds?
Those seem paradoxical, but are actually based on ignorance. -- kainaw 18:42, 11 September 2008 (UTC)[reply]
Emotions and the concept of good and evil are also popular ones to point to. -- MacAddct  1984 (talk &#149; contribs) 19:30, 11 September 2008 (UTC)[reply]
How come there are so few transitional fossils? Also note that in many cases when pro-evolutionists say "we know the answer to that" it often means that they have come up with a suggestion which they think is plausible - not that they have found evidence that it happened that way. DJ Clayworth (talk) 20:38, 11 September 2008 (UTC)[reply]
Actually, there are quite a lot of transitional fossils. I remember scaring off a Jehovah's Witness when I was a university student by offering to show him some of my palæontology textbooks, after he had made a similar "point". DuncanHill (talk) 20:44, 11 September 2008 (UTC)[reply]
Claiming lack of evidence for evolution is evidence against it is just as wrong as claiming that lack of evidence against it is evidence for it. It doesn't matter if anyone has clear proof of transitional fossils or if anyone can show clear proof of anything. Evidence against evolution must be real evidence, not a claim about lack of evidence. -- kainaw 23:46, 11 September 2008 (UTC)[reply]
That's not entirely true - if you can show that a theory would produce a certain type of evidence with a high probability and you can't find that evidence, that would suggest the theory is false. For example, I theorise that no humans can fly since if any could fly I would almost certainly have heard about it. --Tango (talk) 23:52, 11 September 2008 (UTC)[reply]
A failed test of a hypothesis is evidence against the hypothesis. That is evidence. Claiming that a hypothesis has not been tested is evidence against the hypothesis is not evidence against the hypothesis. On this specific topic, there are claims such as "Evolutionists cannot show me a half-monkey/half-human, so that means there is no such thing as evolution." There is no hypothesis in evolution that there exists half-monkey/half-humans. So, the lack of proof that there are half-monkey/half-humans is not evidence against evolution. Similarly, evolution doesn't claim that each and every fossil will survive through all time and be found. So, lack of evidence of fossils is not evidence against evolution. I have a problem with both sides of this argument. Evolutionist for evolutionst sake do not have a clue what they are talking about. Anti-evolutionists (all lumped together) for anti-evolutionist sake do not have a clue what they are talking about. -- kainaw 00:00, 12 September 2008 (UTC)[reply]
Ah, now you're describing the straw man fallacy. However, I disagree with the statement "There is no hypothesis in evolution that there exists half-monkey/half-humans.", given a sufficiently incorrect definition of "monkey" (you have to interpret words in the way the people saying them mean, rather than what they actually mean) there should be some creature inbetween "monkeys" (proto-hominids, I think would be a more accurate term) and humans. Scientists looking for such a link and not finding on is evidence against evolution (it's not very strong evidence, though, given the fact that most creatures don't end up fossilised so you have to get pretty lucky to find one and there are plenty of places we still haven't looked). --Tango (talk) 17:55, 12 September 2008 (UTC)[reply]
One thing that's always bugged me, not necessarily about evolution per se but our understandings of it, is why humans are so vastly overpowered in terms of cognitive ability compared to other primates. I mean, a human being that was half as intelligence as our species potential maximum is still capable of surviving just fine even in a hostile environment. (I suspect the answer has to do with the fact that there were multiple smart-primate species competing with one another, because if it is just humans against other animals, I doubt we'd have gotten this smart, but humans against other smart things, well, then you've got an arms race.) I've never really seen this fleshed out well, in part because our understanding of the nuts and bolts of human evolution is pretty speculative (a fossil here, a grave there). --98.217.8.46 (talk) 23:20, 11 September 2008 (UTC)[reply]
Are you familiar with Neanderthals? Exactly how they ended up extinct and we ended up thriving is still an open question, as far as I know (although I remember reading recently about some evidence that they were just as intelligent as early Homo Sapiens). --Tango (talk) 23:24, 11 September 2008 (UTC)[reply]
Of course I'm familiar with Neanderthals, but it doesn't really solve it for me. The existence of one other somewhat intelligent primate species doesn't really explain why we're so overpowered. This isn't an anti-evolutionary complaint, just a "boy we really don't understand this" complaint. My deeper suspicion is that the human propensity for intra-species violence—far more than most other animals—is somewhat related to this. (Of course Darwin thought it was probably sexual selection, which is fairly similar.) But it'd be nice to have a better understanding, or at least a better model, of how you get to such a point. --98.217.8.46 (talk) 12:47, 12 September 2008 (UTC)[reply]
You're absolutely right, I didn't mean to suggest Neanderthals would give you the answer, just that they're an interesting topic closely related to that question. It's still an open question, as far as I know. --Tango (talk) 17:48, 12 September 2008 (UTC)[reply]
I read that, apparently, there's evidence to suggest that humans committed mass genocide against the Neanderthals. —Preceding unsigned comment added by 65.92.231.82 (talk) 00:25, 14 September 2008 (UTC)[reply]
Intelligence is not, prima facie, a superior evolutionary trait - and this is something that a lot of people who argue about evolution get hung up on. From an intelligent level, we can look and say, hey, great - with science we can increase fertility, defeat the elements, and hunt to extinction anything vaguely resembling a predator, sure. But that's the conclusion begging the premise. I think a useful read is called "The Sickness Gene" or something along those lines - the trifecta of surviving to breed, breeding "most", and having offspring that hit A+B is all there is. In that respect, humans are a vastly inferior form of life to bacteria and cockroaches (and, from a health perspective, a viral epidemic could effectively obliterate the human population... extremophiles would require an astronomical event to be substantively impacted). I believe Mike Judge's movie Idiocracy was another illustration of the principle of evolution - the most fit to breed is not the qualifier we tend to think it is. Sorry, but there's really no counter-evolutionary evidence - Darwin's finches and micro-evolution are difficult to argue with. The best that can be done is circumstantial arguments - OH, HA HA, it turns out that the velociraptor didn't, actually, evolve into turkeys, take that Science! All your precious THEORIES are now rebuffed! ... no, sorry, it doesn't work that way. No more then forgetting to carry a one, one time when you're doing times tables means you're a fundamental failure when it comes to multiplication. 98.169.163.20 (talk) 00:08, 12 September 2008 (UTC)[reply]
The best explanation that I'm aware of (admittedly, I have no evidence to support it, but it sounds plausible) is that human intelligence reached a sort of critical mass, where we developed sufficient cultural complexity that we bootstrapped the intelligence evolution process. In the ancestral environment, humans weren't just using their intellects to compete against other species, they were also competing against each other - for food, mating rights, social status, etc. The increase of intelligence may have accelerated as intelligence became more important in human societies. Don't forget, also, that our methods of developing and fostering intelligence in each other (i.e. education) have also had thousands of years of evolution. Maelin (Talk | Contribs) 09:54, 15 September 2008 (UTC)[reply]

There is a very useful and easily-searchable web site that makes an effort to respond to all criticisms of the Theory of Evolution. It is called "An Index to Creationist Claims". Saukkomies 09:26, 13 September 2008 (UTC)[reply]

The reason why humans cant fly is they can survive without it as it already is humans are dominant so they did not evolve to have wings becuase evolution isnt about making things easy is making it so you can survive another opinion in why only the humans where dominant and not other things is that humans are almost all the animals and alien animals in the world it is infact possible that we are aliens to this planet becuase there was evidence of life on mars and there are piecies of rock from mars that still have live alien cells and could have mixed into the water since if a meteor hit and when fish where evolving and land animals they where exposed to it so that then they would be very smart have emotions and other worldy things. dont believe what churches tell you science is not leading to religion or anything else to do with it...

Plastic bullet for handgun

Why cops and the like don´t use plastic bullet in their handgun? —Preceding unsigned comment added by Mr.K. (talkcontribs) 18:39, 11 September 2008 (UTC)[reply]

By plastic, you probably mean ceramic since plastics that we experience in everyday use could not sustain the acceleration of being fired from a gun. As far as ceramics, there are some that could easily withstand being fired from a gun better than lead/metal, but they cost much more than lead/metal used in bullets. Since bullets are good enough as is, there is no need to spend more. -- kainaw 18:58, 11 September 2008 (UTC)[reply]
Or maybe the OP meant rubber bullets? -- MacAddct  1984 (talk &#149; contribs) 19:10, 11 September 2008 (UTC)[reply]
(edit conflict) I think Mr. K. actually means rubber. Rubber bullets and other types of nonlethal munitions are more range specific. Depending on the charge, rubber can be lethal or so harmless as to be ineffective. They sell them for different ranges but changing bullets on the spot is impractical for a last resort weapon. That's why they're mostly used for crowd control, when they can plan better on how to respond. There are arms manufacturers working on nonlethal guns where you can 'dial in' your range for rubber munitions (air power), but those aren't widespread yet. - Lambajan 19:14, 11 September 2008 (UTC)[reply]
I suspect the original questioner actually meant what he said, namely plastic bullets. DuncanHill (talk) 20:33, 11 September 2008 (UTC)[reply]
And just so Mr K understands, plastic bullets (or rubber bullets) are not simply a different kind of bullet fired from a normal gun - they are large projectiles fired from a special kind of gun: which explains why cops don't use them in their handguns. DJ Clayworth (talk) 20:35, 11 September 2008 (UTC)[reply]
Yow, plastic bullets are meant to be fired "below the waist" of the target. I don't like the sound of that... Franamax (talk) 08:29, 12 September 2008 (UTC)[reply]
I guess it boils down to whether you would you rather lose something below your waist or lose your life... Nil Einne (talk) 10:54, 12 September 2008 (UTC)[reply]
Some people might ask, is there a difference? Somehow "fill ya full of clay" doesn't have the same punch. Julia Rossi (talk) 11:56, 12 September 2008 (UTC)[reply]
I suspect only a small minority though, when they actually think about it Nil Einne (talk) 12:38, 13 September 2008 (UTC)[reply]

LHC as time after big bang

Regarding the new LHC, how close to the big bang is an energy of 7 Tev per particule? Lerichard (talk) 21:01, 11 September 2008 (UTC)[reply]

It looks like somewhere in the Electroweak epoch, from 10-36 seconds to 10-12 seconds after the Big Bang. That's just based on my reading of the pages linked to from Timeline of the Big Bang, so could be wrong. --Tango (talk) 22:41, 11 September 2008 (UTC)[reply]
Agreed. It's sometime after the end of the inflationary epoch, which ended at about 10-32 seconds, but before the weak force separated from the electromagnetic force at about 10-12 seconds. For a ballpark figure, I think this was the typical energy per particle at about 10-13 or 10-14 seconds after the Big Bang. Gandalf61 (talk) 12:50, 12 September 2008 (UTC)[reply]
I was never happy with the inflationary epoch. "We don't see magnetic monopole, so maybe they are far away!" There was no attempt to apply the scientific method. Although now that I read Alan Guth's article it sounds as if WMAP confirmed some predictions of his theory? The article is vague and unsourced - can someone expand on that for me? Plasticup T/C 16:12, 12 September 2008 (UTC)[reply]

Fantastic Structure?

Is this real? It's seems pretty authentic, but I can't find any other sources. PerfectProposal 21:49, 11 September 2008 (UTC)[reply]

It looks like a giant building proposal, but is probably beyond building technology if you read the comments lower down. Graeme Bartlett (talk) 22:15, 11 September 2008 (UTC)[reply]
It is very common for Architects/Civil Engineers to propose designs for currently unconstructable buildings. Like Frank Lloyd Wright's The Illinois. It is a "portfolio" piece, that is it won't be built but shows the designers abilities. Fribbler (talk) 22:57, 11 September 2008 (UTC)[reply]
I would think a key component of a designer's ability is their ability to design something within set limits - "possible to build" would be a limit I would set on any commission for design work. --Tango (talk) 23:21, 11 September 2008 (UTC)[reply]
What a crazy design. I thought Burj Dubai was pretty excessive but that thing is like three or four times taller. --98.217.8.46 (talk) 23:10, 11 September 2008 (UTC)[reply]
The same company was pitching this idea in Tokyo until there wasn't enough money there for it. Now, they are pitching it in Dubai - until that money dries up. My opinion - they increase their stock revenue by pitching these ideas and profit. Then, they invest elsewhere until they find a new area to pitch an idea to. They buy back their depleted stock, pitch the idea, drive up their stock value, and sell. -- kainaw 00:04, 12 September 2008 (UTC)[reply]
This is approaching the idea for a mini-arcology. It's not really practically feasible with the technology and costs of that technology that we have today. Also, attempting to insure this building would be a nightmare. —Lowellian (reply) 05:11, 12 September 2008 (UTC)[reply]
When an engineer scrutinizes the plans and tells me it's not possible then I'll believe it's not possible. If Kainaw's right and they're going around pitching this thing and the only problem is not enough money then it probably is possible. I'm sure that if Japan turned it down because their engineers gave it a 'thumbs down' then they wouldn't be able to pitch it anywhere else. - Lambajan 13:40, 12 September 2008 (UTC)[reply]
But would we necessarily know if the Japanese engineers gave it a thumbs down? Zain Ebrahim (talk) 13:48, 12 September 2008 (UTC)[reply]
It was never given a thumbs up in Japan. The only reason anyone really heard about it was because it was featured on a Discovery program about really big buildings and some other channel that had a show about really big things. The shows made every attempt to give the appearance that this building was going to built soon in Japan, but the truth is that there are no current plans to build. It is just a small group of guys pitching the idea here and there. I figure that when this idea runs out, they'll start pitching either cities in orbit or under-ocean cities. -- kainaw 14:05, 12 September 2008 (UTC)[reply]

September 12

If the LHC created a black hole...

Or more generally, if a black hole were to appear on or near Earth's surface(presumably a small one, but let's say it's just big enough to avoid collapsing into Hawking radiation), how long would it take to swallow up the entire planet? And what would happen to the rest of the solar system? 68.123.238.140 (talk) 02:10, 12 September 2008 (UTC)[reply]

Longer than the expected life of the solar system, and no effect. Even if it swallowed the Earth in a day, the solar system would be fine -- you'd still have an Earth-ish mass orbiting the sun. — Lomn 02:25, 12 September 2008 (UTC)[reply]

Another related question: how big would a black hole need to be in order for there to be exactly 24 hours before the earth was gone? Nadando (talk) 04:07, 12 September 2008 (UTC)[reply]

Cannot be computed exactly. In fact, a silly question (do you need a 24h message in advance?). Next black hole question, please. --Ayacop (talk) 08:00, 12 September 2008 (UTC)[reply]
So you could use a vacation day, of course... --- OtherDave (talk) 10:40, 12 September 2008 (UTC)[reply]

It's amazing how ill-informed scientific reports, scare stories and other paper selling drivell is generating an unprecedented interest into an amazing scientific work that might have gone unnoticed outside of the ususal aficionados. 190.244.186.234 (talk) 11:47, 12 September 2008 (UTC)[reply]

I think the scientific community has underestimated how well a little irrational fear can benefit them if they spin it right. --98.217.8.46 (talk) 12:42, 12 September 2008 (UTC)[reply]
"Nice planet, mate. Awful shame if somethin' bad happened to it ..." Gandalf61 (talk) 13:52, 12 September 2008 (UTC)[reply]

Rather a high level of WP:BITE here: please avoid pejoritives such as "silly," "drivell (sic)," and "irrational" in responses. Lomn has the right idea in providing a reasoned and informative response without insulting the questioner. Edison (talk) 19:01, 12 September 2008 (UTC)[reply]

Have any scientists calculated the beam energy scale where we would actually say "Hmm, maybe we better not try that"? Could it be the next machine after the SHLC? Franamax (talk) 19:27, 12 September 2008 (UTC)[reply]

Probably somewhere around the Overwhelmingly Large Hadron Collider, which I think comes after the Ginormous Hadron Collider. -- Coneslayer (talk) 19:32, 12 September 2008 (UTC)[reply]
I don't know how higher energies are likely to cause problems, regardless of how high they get (assuming the equipment is built to handle that much energy - obviously if the machine explodes that would be undesirable! It wouldn't destroy the planet though, just Switzerland. ;) ). All the fear with black holes is nonsense, it doesn't matter how much energy is involved, black holes just don't behave in ways that would swallow the Earth like that. --Tango (talk) 21:22, 12 September 2008 (UTC)[reply]
Yes, I'm not that interested in the MBH hypothesis, for one thing, we now have an experiment underway to test it :)
However, to take a ridiculous example, if you collided two particles with the same energy as the Big Bang, you would end up with a new expanding universe in your lab, which could have undesirable results. Somewhere between there and here must be a line where problems could result.
Wouldn't there be an energy where you made a black hole big enough that it would be a problem? Or a sufficiently energetic strangelet or powerful magnetic monopole that they would do whatever bad things they might do?
How many orders of magnitude are we away from those energies? Franamax (talk) 23:15, 12 September 2008 (UTC)[reply]
The energy has to come from somewhere, you would need to extract all energy from the whole universe in order to create an explosion with the same energy as the big bang (and even then, it wouldn't be a big bang - the big bang involved space expanding, not just matter exploding). --Tango (talk) 08:38, 13 September 2008 (UTC)[reply]
Well, I suppose I shouldn't have postulated the ridiculous case of the Big Bang. (Though I'm reminded somewhat of an Asimov story where the first computer was asked "can the end of the universe be prevented?" and the last computer solved it, and "the cosmic AC said 'Let there be light' - and there was light").
I will persist in my question though - is anyone aware of scientific calculations as to the energy scale of a collider which could produce particles or states of matter which could pose a planetary threat? I'm pretty sure that every accelerator ever built has had just such a risk assessment - so what was the risk based against? In fact, in the 60's and 70's, I think there was a specific named committee to examine new accelerators.
And as a secondary question, what are the energy scales where we could possibly produce a more stable phase of matter, a cosmic Ice-9 as it were? Obviously these would be theoretical answers - has anyone done the theory? Franamax (talk) 10:27, 13 September 2008 (UTC)[reply]
Even in less extreme cases, the energy still has to come from somewhere. I don't think putting lots of energy into a few subatomic particles is going to be any more dangerous that releasing the same amount of energy with a bomb, so you would need to get an amount of energy into the particles comparable to the energy released by several large nuclear bombs - that's very difficult to do. As for your second question, more stable than what? Regular matter is pretty stable... --Tango (talk) 11:21, 13 September 2008 (UTC)[reply]

Water bears on Mars

We hear about the harsh conditions on Mars, and how unlikely it is to find life. But Water bears (Tardigrades) have been shown to survive (for 10 days)[16] conditions far harsher than those seen by recent Mars probes, including the vacuum of outer space, temperature extremes, and intense UV radiation.. They can survive ten years without water, temperatures from about absolute zero to 151 Celsius, and 1,000 times the radiation which would kill humans. Could terran Tardigrades survive and reproduce in the Martian polar areas, given that there seems to be liquid water sometimes? (after 6 "database locked" error messages)Edison (talk) 05:16, 12 September 2008 (UTC)[reply]

I don't think there's ever liquid water on Mars (currently, that is), the most it does is sublimate. Also, according to this Wired article: "Tardigrades may even provide insights into adaptations necessary for survival in off-Earth colonies, though they could not live actively in the extreme and nutrient-poor environments of Mars or the moon." -- MacAddct  1984 (talk &#149; contribs) 05:44, 12 September 2008 (UTC)[reply]

See Extremophile. Even the limited spectrum of life seen on this planet shows that there is probably no environment too harsh for life to develop. There is a very real chance that space itself is liberally populated by micro-organisms which could very well seed planets, satellites, asteroids and comets with the first steps on the way to macro-organisms - I deliberately avoid use of the word "higher" or "more advanced", as being very subjective. 196.2.124.253 (talk) 06:47, 12 September 2008 (UTC)[reply]

The bacteria that can survive in space usually do so by becoming dormant, they need less extreme conditions to actually reproduce. I don't think the surface of Mars ever has conditions suitable for terrestrial life to reproduce, although somewhere underground theoretically could. --Tango (talk) 10:32, 12 September 2008 (UTC)[reply]

Maybe deinococcus radiodurans would be a better candidate. -- MacAddct  1984 (talk &#149; contribs) 14:16, 12 September 2008 (UTC)[reply]

If there is never liquid water, coult sometning like a tardigrade live in the water ice layer a few inches below the Martian surface, dormant most of the time, but active and reproducing when conditions are least harsh? Desert plants and animals remain dormant for a year or longer on earth and use the rare less harsh conditions to do their business. Edison (talk) 18:55, 12 September 2008 (UTC)[reply]
Earth-derived life seems to requre exactly two things to flourish: liquid water, and an energy source. We've never found life where one or the other is missing, and everywhere we've looked that has both, we've found life. --Carnildo (talk) 21:35, 12 September 2008 (UTC)[reply]

If one is speculating about the forms that life might take, then there is no reason to suppose that water is a prerequisite for life to develop or exist. 196.2.124.253 (talk) 19:56, 12 September 2008 (UTC)[reply]

There is somewhat of a difference between lasting 10 years and 100000000 years between ice melting. Radiation damage would be far more severe, and any way what is there to eat for those tardigrades on Mars? The perchlorate would poison them anyway! Being animals they also need to respire oxygen . Graeme Bartlett (talk) 22:48, 12 September 2008 (UTC)[reply]
Water on Mars? They also recently found wet rivers that dried up I think.
~AH1(TCU) 23:48, 12 September 2008 (UTC)[reply]

Gamma ray bursts and black holes

Gamma ray bursts are a very large release of energy. Assuming black holes have a limited lifetime because of an upper limit to the mass that they can contain, would an explosive release of that mass be of the same order of energy as observed gamma ray bursts? 196.2.124.253 (talk) 06:31, 12 September 2008 (UTC)[reply]

Much bigger. Who cares, it will never happen. Next black hole question, please. --Ayacop (talk) 07:56, 12 September 2008 (UTC)[reply]
Since no existing theory predicts an upper limit to the mass of black holes, there's nothing we can really say about what would happen when a black hole hit such a limit. Algebraist 09:46, 12 September 2008 (UTC)[reply]
Black holes don't explode... very small ones evaporate by Hawking radiation pretty explosively, but all black holes will evaporate at the same rate in their last X seconds of life, regardless of their starting mass (bigger ones will just take longer to get there). --Tango (talk) 10:30, 12 September 2008 (UTC)[reply]
This possibility has been explored by scientists, but not for black holes formed from stars since they last very very long, but for smaller black holes created in the big bang. I can't find any references but I think they studied what the signature flash would be like. No sightings so far. EverGreg (talk) 16:03, 12 September 2008 (UTC)[reply]
As mentioned, there is no known upper limit, but presumably if a black hole evaporated for long enough, at some point (the Chandrasekhar limit?), gravity would not be strong enough to resist the various forces of matter that enjoy a little lebensraum and it would explode. --Sean 16:40, 12 September 2008 (UTC)[reply]
As far as I know, degeneracy pressure doesn't count for anything once it's collapsed into a black hole - there isn't really any matter left, there's just a singularity (of course, our understanding of black holes isn't really up to explaining what happens to the matter). Current theories don't include any kind of lower limit under which black holes explode, there's just Hawking radiation (which grows exponentially, so the last fraction of a second would seem rather explosive). --Tango (talk) 17:42, 12 September 2008 (UTC)[reply]

Radioactive decay in materials. Schrodinger's cat

The degree of decay in a given radioactive material is measured in half-life. If this is so, then surely it should be possible to predict the point in time at which a material has sufficiently decayed to an extent that it will trigger the release of a poisonous gas, thus killing an imprisoned cat? —Preceding unsigned comment added by 78.32.39.90 (talk) 08:13, 12 September 2008 (UTC)[reply]

Half-life is a measure of a quantity of atoms, i.e., statistical. No way to predict the behaviour of a single atom which is random. Those gedanken experiments you are referring to (see Schrödinger's cat) always involve a single atom, not a quantity. --Ayacop (talk) 08:30, 12 September 2008 (UTC)[reply]
The half-life is basically an average, there is plenty of deviation from that average for either small amounts of the isotope or small periods of time. --Tango (talk) 10:27, 12 September 2008 (UTC)[reply]
As the others say, the problem is that if it is a single atom you have no way of predicting when it will or won't decay. Half-life does not tell you about that. --98.217.8.46 (talk) 12:37, 12 September 2008 (UTC)[reply]
Instead of providing a solution, the statistical nature of half-life is actually what causes the problem. If you have a large quantity of a radioactive material, after one half life 50% of that material will have decayed. Therefore, there is a 50% probability that any individual atom in that material will decay. According to quantum mechanics, that atom actually exists in a superposition of states (decayed/radioactive) until we observe it, at which point our observation collapses the wave function into one state or other. The point of the schrodinger's cat thought experiment was that by including the cat in the system, the cat itself should be thought of as entagled with the atom and therefore as existing in a superposition of states (dead/alive) until we observe it, which is extremely counter-intuitive. Schrodigner originally meant the experiment as a discusison of QM's flaws, but subsequent interpretations of the thought experiment have been used to illustrate how other interpretations of QM operate. Read our article on it, it's not bad. --Shaggorama (talk) 14:18, 12 September 2008 (UTC)[reply]

Unsailable Dead Sea

Our Dead Sea article states that the "salinity makes for a harsh environment where animals cannot flourish and boats cannot sail". Why is that? Is it due to high buoyancy or the effect of the salt on the hull of the boats? Or something else? -- MacAddct  1984 (talk &#149; contribs) 15:12, 12 September 2008 (UTC)[reply]

No... I suspect that the article is wrong. Plasticup T/C 16:00, 12 September 2008 (UTC)[reply]
I've dropped it from the article. Such a surprising claim certainly needs a supporting citation. ike9898 (talk) 16:28, 12 September 2008 (UTC)[reply]
Animals not being able to flourish is certainly true (hence the name Dead Sea), but I can't see any reason why boats would have any difficulty, the added buoyancy should make things easier, not harder. --Tango (talk) 17:44, 12 September 2008 (UTC)[reply]
The very salty water might promote corrosion of metal hulls, but that's hardly the same as "boats cannot sail". --Anon, 22:06 UTC, September 12, 2008.
The Israel Museum certainly thinks it's possible to sail: [17] Now, they're just showing old anchors, so it's technically possible that all you can do is anchor a boat in the Dead Sea. ;) I would think though that the greater buoyancy would decrease draft and maybe make it much harder to control the boat. You might need so much ballast that your boat would be too heavy for the wind to push. Franamax (talk) 23:00, 12 September 2008 (UTC)[reply]
I was thinking that - I think you just need to change the shape of the boat so it has a flatter bottom, that should stabilise it (I've not done much sailing, so I'm guessing here). --Tango (talk) 08:35, 13 September 2008 (UTC)[reply]
This convention center advertising touts "sailing on the Dead Sea itself". Clarityfiend (talk) 02:03, 13 September 2008 (UTC)[reply]

And in case there's any doubt left at all... Site Matt Deres (talk) 15:25, 13 September 2008 (UTC)[reply]

Electroforming solution

Is there anywhere where i can find some recipes for a coppor electrioforming solution. I know that the main components are distilled water, copper suphate and various acids but cannot find any information about quantaties of each. Any help would be very much appreciated. regards Kirk uk —Preceding unsigned comment added by 87.82.79.175 (talk) 16:04, 12 September 2008 (UTC)[reply]

here you go. Watch yourself with those acids! Fribbler (talk) 17:53, 12 September 2008 (UTC)[reply]

Patella/knee jerk reflex

I've knocking my knees every which way with a finger-long hammer (toy, tool? I know not which) and I'm getting no reflex. I've seen the graphics but can't find the spot. Argh. Imagine Reason (talk) 17:40, 12 September 2008 (UTC)[reply]

I think it's just below the knee cap, there's a soft bit. Make sure your knee is bent and you should feel a dip between the knee cap and the top of the lower leg bone (I'm sure someone will tell me what that bone is called, but I don't really care!). I can trigger the reflex by hitting that spot with the side of my hand. --Tango (talk) 17:46, 12 September 2008 (UTC)[reply]
Yes, the patellar tendon is the spot, like tango says (above the tibia). You can also try to enhance the reflex by clenching your teeth or trying to pull your clasped hands apart. Fribbler (talk) 17:49, 12 September 2008 (UTC)[reply]
Huh. What's that based on? I mean, what makes it work like that? -- Captain Disdain (talk) 17:59, 12 September 2008 (UTC)[reply]
Shhhh Captain! :-) Telling you how it works will likely remove some (but not all) of it's effectiveness. It sure works though. I use it regularly. Fribbler (talk) 18:08, 12 September 2008 (UTC)[reply]
Oh, good old misdirection. I dig. -- Captain Disdain (talk) 00:11, 13 September 2008 (UTC)[reply]
It works better if someone else does it to you. Graeme Bartlett (talk) 22:38, 12 September 2008 (UTC)[reply]

chemistry - glassware

I have been working in labs for a couple of years, but I never have understood exactly the qualifications for volumetric glassware to called class "A." Does anyone know what the requirements are? —Preceding unsigned comment added by 204.214.74.84 (talk) 18:12, 12 September 2008 (UTC)[reply]

See Volumetric flask. Strawless (talk) 18:56, 12 September 2008 (UTC)[reply]

I have read that article but I was wondering if there is a more precise definition of the classifications -- all the article says is that the graduation mark is placed more accurately. This begs the question, to what degree of accuracy? —Preceding unsigned comment added by 204.214.74.84 (talk) 19:55, 12 September 2008 (UTC)[reply]

Have a look at this pdf file. It contains a table of how accurate the graduations must be. Fribbler (talk) 20:01, 12 September 2008 (UTC)[reply]

PHEV Battery Economics

How can I calculate the following:

  • Cost of Plug-in Hybrid Electric Vehicile (PHEV) vs. Internal Combustion Engine (ICE) in cents/mile?
  • Total lifetime operating cost of PHEV-vs-ICE, including the decrease in battery capacity and resistance to re-charge with time, as well as the difference in maintenace costs between PHEV and ICE technologies?

Also: Can the batteries proposed for PHEV's be used as a storage system for homes using wind or photo-voltaic power systems? 146.18.173.73 (talk) 18:51, 12 September 2008 (UTC)EC[reply]

The Tennessee Valley Authority had a wind generator which charged battery powered vehicles as a demonstration project over 30 years ago, so clearly wind generated electricity could be used to charge the batteries of a plug-in hybrid vehicle. Any source of electricity could be used to charge any batteries, with varying cost effectiveness. As for the other question, experience may not exist yet for lifetime operating cost. What is the lifetime of a vehicle these days? Edison (talk) 01:06, 13 September 2008 (UTC)[reply]

angular momentum

Is angular momentum just a manifestation of linear momentum (ie you can treat angular momentum as the combination of thelinear momenta of each of the rotating object's particles)? There's a follow-up question. —Preceding unsigned comment added by 65.92.231.82 (talk) 21:04, 12 September 2008 (UTC)[reply]

Short answer, yes. I think it also works the other way around - linear momentum is just angular momentum taken around a point at infinity (don't quote me on that, though!). --Tango (talk) 21:25, 12 September 2008 (UTC)[reply]
This is probably a silly question -- I'm trying to make my quota for the week. If linear momentum is angular momentum taken around a point at infinity, is that point on the line of the linear motion, off to the right, downstage left, or?? Wanderer57 (talk) 21:46, 12 September 2008 (UTC)[reply]
I haven't heard this interpretation before, but it seems like it ought to be the point at infinity at right angles to the line (which of these two points you choose will just introduce a minus sign somewhere). Algebraist 21:52, 12 September 2008 (UTC)[reply]
Wouldn't any point not on the line of motion yield a linear momentum? i.e. hold your reference point steady then allow the mass to continue moving, it's no longer at a right angle but the momentum is still linear. -- Mad031683 (talk) 22:03, 12 September 2008 (UTC)[reply]
Yes, but not a constant one. An object moving in a circle does have a linear momentum, but it changes direction as it moves round. If the point is at infinity (and I did have an infinity at right angles in mind, yes) then that change in direction becomes zero (a radius of curvature of infinity corresponds to a straight line). I'm not sure how rigorous all that is, I haven't really heard it before in this context, but it makes sense intuitively (I have heard of translations being thought of as rotations about infinity - momenta are a natural extension of that, I think). --Tango (talk) 08:32, 13 September 2008 (UTC)[reply]
Okay, there's a reason I asked this question. If I push against a vertical wall, I will gain linear momentum, right? But the earth in response will start spinning. Isn't the sum of the linear momenta of a rotating body equal to 0 (every point has corresponding point going in the opposite direction). So how is linear momentum conserved? Is the question even valid (a force is acting on each of the rotating particles to keep them in uniform circular motion)? —Preceding unsigned comment added by 65.92.231.82 (talk) 21:57, 12 September 2008 (UTC)[reply]
In considering the momentum changes when two bodies interact, you have to use a reference frame based on their common center of gravity. When the objects are you and the Earth, that will be near enough to the center of the Earth. When you push against a wall, you set the Earth spinning and you also set yourself rotating around that center of gravity. The momentum you gain only seems linear because your motion is constrained to be parallel to the Earth's surface and you move a small enough distance that the Earth seems flat. But angular momentum is conserved. --Anon, 22:17 UTC, September 12, 2008.
No. Gravity does not have to have to be intoduced. Besides, you can push strongly enought to surpass the earth's escape velocity. Anyway, even in your example linear momentum wouldn't be conserved. —Preceding unsigned comment added by 65.92.231.82 (talk) 23:26, 12 September 2008 (UTC)[reply]
  • I did not mention gravity, I mentioned the center of gravity, also called the barycenter or center of mass. My apologies for confusing you by my choice from the three synonyms. --Anon, 21:08 UTC, September 13, 2008.
Ignoring gravity, you push on a wall on Earth's surface and fly off into space. Your push gives Earth some angular momentum (around its center of mass) and also some linear momentum (its center of mass moves away from you as it spins around it). -- BenRG (talk) 10:23, 13 September 2008 (UTC)[reply]

Missiles on Main Battle Tanks

Are there any MBT's that mount missiles? Is there a reason why most (if any) don't mount missiles? ScienceApe (talk) 21:36, 12 September 2008 (UTC)[reply]

Missile tank. Dragons flight (talk) 21:51, 12 September 2008 (UTC)[reply]
Price is a factor: a TOW missile appears to have a unit cost (the missile, not the launcher) of $38,000. The M256 round an M1 Abrams MBT fires costs around $4000. The two rounds have comparable weight and lethality (although the TOW has greater range). That cheaper round means you can have more in inventory and thus don't have to be so careful in deciding what to shoot at (and so can shoot at lower-value targets). -- Finlay McWalter | Talk 21:56, 12 September 2008 (UTC)[reply]
Yea, I've considered the costs. Missiles cost so much which is why railguns are so appealing in the future. But missiles can do things that cannons can't. Like being guided. Such a thing would be useful against attack helicopters and such. I would think that kind of extra protection would justify the extra costs. ScienceApe (talk) 22:14, 12 September 2008 (UTC)[reply]
But cannon shells can be guided.[18] Once again expense enters into the equation. Rmhermen (talk) 23:48, 12 September 2008 (UTC)[reply]
It's always tempting (to generals in particular) to keep adding stuff on (until they've build an Apocalypse Tank). But you get much more flexibility if you build the antiaircraft capability separately, so you can a) defend what you like with it and b) it can be a mile or so behind the dynamic tank battle and still give effective AA protection. The jack of all trades is the (super expensive) master of none. -- Finlay McWalter | Talk 22:33, 12 September 2008 (UTC)[reply]
Additionally, you don't really want all your big expensive fragile (and explosive) stuff outside the tank's armor. Even if its destruction won't harm the tank, it's highly vulnerable to soft kills and endangers exposed crew or nearby troops. — Lomn 23:19, 12 September 2008 (UTC)[reply]
Eh. I've considered that too, but I would think there would at least be some more experimentation with such a tank. I guess we could assume it would become a jack of all trades, master of none, but without actual testing it's a little presumptuous. ScienceApe (talk) 03:26, 13 September 2008 (UTC)[reply]
According to IT-1, "The large deadzone around the tank created by the missiles' minimum range combined with the limited amount of ammunition carried made it unpopular with the military." Clarityfiend (talk) 08:10, 13 September 2008 (UTC)[reply]

England MD/MBBS info

Hello every one.I need imformation about England Medical College/Universities for MD/MBBS courses.Please any body tell me the Tuition Fee,Living Fee,visa and other requirement for international students and also scholarship availibility.Any help will be much appreciated.OR someone wantto directly contact me are most wellcomed. thax —Preceding unsigned comment added by 202.125.143.76 (talk) 23:13, 12 September 2008 (UTC)[reply]

Hi 202, I googled here[19] then my eyes decided to swim in opposite directions so it's over to you – but it's a start until someone who knows comes by. Julia Rossi (talk) 07:27, 13 September 2008 (UTC)[reply]
Hi. Individual medical schools in the United Kingdom all charge different tuition fees, the only way to tell is to look at the individual schools themselves. Try looking at List of medical schools in the United Kingdom, picking a few that sound interesting to you, and going to their websites. They always give details of their international rates, and usually give you an idea of living costs. —Cyclonenim (talk · contribs · email) 08:19, 13 September 2008 (UTC)[reply]
This website [20] run by the British Council, has a number of search options for courses, scholarships etc. It would also be worth contacting the British Council, either through the website or through the branch nearest you. DuncanHill (talk) 12:11, 13 September 2008 (UTC)[reply]

September 13

Unidentified Plant

File:UnkFlwr.jpg
Pic1
Pic2

Can you help me identify this plant? What I know: It has feathery light green leaves, 5 petal bloom with yellow centre, perennial, flowers are pink or white, grows tall, seems to spread easily, grows in southern Ontario.

Thanks, *Max* (talk) 00:09, 13 September 2008 (UTC).[reply]

Floated the two photos --antilivedT | C | G 02:33, 13 September 2008 (UTC)[reply]
Is it a form of Cosmos (plant)? Especially Cosmos bipinnatus for the leaves. If you google images "cosmos plant" you'll find some more like it. The volume of petals varies from a simple variety to hybrids. Julia Rossi (talk) 07:20, 13 September 2008 (UTC)[reply]

F-16 Fuel consumption

Quoted from a local paper:

"Depending on thrust and altitude, an F-16 burns anywhere from 200 to 400 kilos of fuel a minute"

Is this true?

The F-16 article doesn't answer the question as far as I can tell. Thanks. Wanderer57 (talk) 00:21, 13 September 2008 (UTC)[reply]

Yes, that's about right. You can google for "F-16 fuel consumption" to verify. There's an interesting table comparing the fuel consumption of different fighter variants. The consumption depends heavily on the mission, the payload at takeoff, the speed and altitude flown, etc. ~Amatulić (talk) 00:36, 13 September 2008 (UTC)[reply]
The table indicates a loiter time of 145 minutes, which would imply a fuel weight of at least (145 min)×(200 kg/min) = 29,000 kg or so, but F-16 indicates a dry weight of 8,670 kg and a maximum weight of 19,200 kg. I conclude, therefore, that the 200 kg/min figure is not actually a lower bound, and the burn rate could be a lot lower during cruise. Maybe 200-400 kg/min is a typical range, not an absolute range. -- Coneslayer (talk) 01:01, 13 September 2008 (UTC)[reply]

Kinetic energy revisited

Hi. Sorry if it looks like I'm being naïve again, but a few months ago I asked a question about kinetic energy, and I remember the following formula. This is not homework. kE=mv2; v=t•a; d=t2•a/2. Sorry I'm not good at LaTex, t= time, a= acceleration, d= distance. Anyway, here's the problem. An apple with a mass of 100g dropped from 1 metre has kE of roughly 1 joule. However, if I follow the distance formula, 1 = t2•5; t=~0.45, v=~4.47, kE=2. It should equal 1, so something isn't working. I'm not going to speculate by assuming that d=t2•a instead of d=t2•a/2, because that is probably wrong. Again, this is not homework, I would like to remind myself for myself. Thanks. ~AH1(TCU) 00:29, 13 September 2008 (UTC)[reply]

Wait, or is it d=t2•a/2 and not d=t2•(a/2)? Oops. ~AH1(TCU) 00:33, 13 September 2008 (UTC)[reply]
Those two expressions are equal. Your mistake is in your first formula: kinetic energy is one half of mass times velocity squared. Algebraist 00:35, 13 September 2008 (UTC)[reply]
Algebraist is (of course) correct. But you can also avoid finding or using the final velocity of the apple if you remember that energy gained = net force x distance (in direction of force). In this case the apple starts from rest and experiences a force of 0.1 kg x 10 ms-2 (if we take g to be 10 ms-2) over a distance of 1m, so its final kinetic energy is 0.1 x 10 x 1 = 1 J. Gandalf61 (talk) 11:02, 13 September 2008 (UTC)[reply]

Turtle Bites and Salmonella

Can you get salmonella from a turtle bite if you didn't bleed? The skin is starting to peel off, but I never started bleeding. Thanks. Nick (talk) 01:09, 13 September 2008 (UTC)nicholassayshi[reply]

You should consult a physician, because we cannot give medical advice here. Keep in mind that salmonella can enter through the mouth, for example, which is why you need to practice very good hygiene any time you're handling reptiles. If you've been distracted by a bite, it's easy to imagine touching your face or eyes without realizing or remembering it, or touching objects or surfaces that you don't think to sanitize later. -- Coneslayer (talk) 01:21, 13 September 2008 (UTC)[reply]
Agreed. Have you seen the article Salmonella especially this bit[21]? It breifly mentions contracting it through handling iguanas and turtles with a reference. Julia Rossi (talk) 05:55, 13 September 2008 (UTC)[reply]
It appears many reptiles have bad mouth flora: see for example Komodo_Dragon#Venom_and_bacteria. --Ayacop (talk) 07:58, 13 September 2008 (UTC)[reply]
Sort of an extreme example though. Komodo dragons are sort of a special case. --98.217.8.46 (talk) 08:12, 13 September 2008 (UTC)[reply]

Bird identification request

At the memoriam to deceased Wikipedian Jeffpw, there is a request to identify the birds in this photo. Please identify the birds in the photo and post the results here. Thanks. -- Suntag (talk) 04:35, 13 September 2008 (UTC)[reply]

You might find something here at Buzzards and Eagles of Thailand[22]. Julia Rossi (talk) 05:51, 13 September 2008 (UTC)[reply]
The larger bird is probably an immature White-bellied Sea-Eagle (Haliaeetus leucogaster) and the two smaller birds are probably Brahminy Kites (Haliastur indus).--Eriastrum (talk) 17:25, 13 September 2008 (UTC)[reply]

electrical circuit solving

1.why is the power transfer curve for a load resistor in a simple electrical circuit(battery,source resistor,load resistor),is not symmetrical over resistance. 2.why thevinen's theorem for solving complex circuits apply only on linear circuits 116.71.183.222 (talk) 10:58, 13 September 2008 (UTC)[reply]

These appear to be homework questions. If you have specific things you don't understand about the questions, you'll probably have better luck getting an answer, but we're not going to do your homework for you. -- Captain Disdain (talk) 12:52, 13 September 2008 (UTC)[reply]

transistor

In a transistor we observe base -collector resistance less then the other b-e junction.why?116.71.183.222 (talk) 11:03, 13 September 2008 (UTC)[reply]

See the answer to your previous question. -- Captain Disdain (talk) 12:52, 13 September 2008 (UTC)[reply]
These appear to be college-level homework questions that you want to avoid learning the answers to. I assume you're not on a scholarship, so I have to ask what is it that you're paying for? --Sean 20:16, 14 September 2008 (UTC)[reply]

Houston muscle

Is the Houston muscle present in all men, or is it only present in some? Thank you in advance for your answer. Leptictidium (mt) 13:41, 13 September 2008 (UTC)[reply]

What is the houston muscle? I've never heard of it, unless it has a more common name. Fribbler (talk) 13:56, 13 September 2008 (UTC)[reply]
Its more common name is the compressor venae dorsalis penis. DuncanHill (talk) 13:58, 13 September 2008 (UTC)[reply]
Aha, yup found it. My Google search was missspelled. Anyhoo, it's a variant of the bulbospongiosus muscle and being a variant, only some men will have it. Couldn't find percentages on how many, though. Fribbler (talk) 14:06, 13 September 2008 (UTC)[reply]

ice caps of the earth

I have seen long ago in a childrens program that if you fill a glass of water half way, and fill the glass to the brim using ice;the question being: will the glass overflow if the ice melts? the answer is no. Would'nt the same rule aply with the Earth?; if the mass of the continents is already in the equasion of the Earths oceans? —Preceding unsigned comment added by 68.114.246.137 (talk) 17:50, 13 September 2008 (UTC)[reply]

By analogy to the example you give, floating ice (like the north polar ice) will have negligible effect, but ice that sits on land and slides into the ocean can have a profound effect. --Scray (talk) 17:57, 13 September 2008 (UTC)[reply]
Glaciers being the main example of such ice. --Tango (talk) 18:11, 13 September 2008 (UTC)[reply]
Ice has a different volume than liquid water. It is not a question of mass. In the example you give, I imagine the reason it doesn't overflow has to do with surface tension, not the fact that the ice and the liquid have the same total volume, as they do not (which you can easily see if you put a fixed amount of liquid into a small container and then freeze it—it'll burst). --98.217.8.46 (talk) 18:02, 13 September 2008 (UTC)[reply]
No, it's nothing to do with surface tension. In order for the ice to float it has to displace an amount of water with the same mass as it (see Archimedes principle). When it melts, it will have the same volume as the water it displaced, so will just take the place of the part of the ice cube that was underwater and the water level won't change. --Tango (talk) 18:10, 13 September 2008 (UTC)[reply]
Huh, I guess my brain isn't working today. --98.217.8.46 (talk) 19:37, 13 September 2008 (UTC)[reply]
I'm not sure quite what you're asking, but the continents aren't floating on the ocean. There is no water underneath, just the Earth's mantle. --Tango (talk) 18:10, 13 September 2008 (UTC)[reply]
The Arctic polar ice cap is frozen ocean and so is indeed floating on sea water Jdrewitt (talk) 20:27, 13 September 2008 (UTC)[reply]
But the ice caps on Greenland and Antarctica are largely on land, so melting of this ice would indeed act to raise sea level. That is the main reason for the concern over global warming causing low-lying land to flood. More detailed discussion at Sea level rise#Glaciers and ice caps. --Anonymous, 21:10 UTC, September 13, 2008.
And don't forget Post-glacial rebound. Wisconsin is still rebounding. Saintrain (talk) 22:33, 13 September 2008 (UTC)[reply]
The raising of the sea levels has as much to do with the thermal expansion of the existing water as it does to do with melting glaciers. Plasticup T/C 02:26, 14 September 2008 (UTC)[reply]
Another issue not accounted for in the ice in the cup analogy is that the ice caps are bright white, and liquid ocean is dark, so a lot more sunlight is reflected off into space when there's ice than there will be in nearish future when the arctic ice cap is only visible in history books. This increased heat will cause more thermal expansion. See albedo. --Sean 20:12, 14 September 2008 (UTC)[reply]

Storage

I am in the process of sketching out a new invention, which I unfortunately cannot discuss the details of (for obvious reasons). However, my invention has certain storage requirements that I am hoping somebody could assist me with. Basically, I need a form of storage with sufficient capacity to store the exact location, and state, of every atom in a adult human's body. Can anybody suggest a form of storage which would have this capacity? If you can help with this, please let me know. Thanks John. —Preceding unsigned comment added by Jbsouth1971 (talkcontribs) 19:05, 13 September 2008 (UTC)[reply]

Um, no. You're talking about something that needs a minimum of like 1027 bits of information, but probably even more than that (that's just the number of atoms in the body—so add to that some sort of x,y,z coordinate system, plus all that can go into "state", e.g. which element, ions, etc., how it is bonded with other atoms into molecules, etc., and you're increasing the total amount of data by five or six times at a minimum). To give you an idea of how much that is, Google's total capacity is more than 10 orders of magnitude (10 million times) smaller than that. In an imaginary science fiction universe the best way to store that sort of information would be in directly making an exact duplicate of the body. --98.217.8.46 (talk) 19:47, 13 September 2008 (UTC)[reply]
are you building, a "teleportation" device? Coolotter88 (talk) 19:30, 13 September 2008 (UTC)[reply]
Hi John. Maybe the guys over at archive.org could help. They need an extra 20TB of space every month.78.148.226.73 (talk) 19:51, 13 September 2008 (UTC)[reply]
Doesn't this run into a major issue with the Heisenberg uncertainty principle? Exact location and state would seem entirely impossible. — Lomn 20:07, 13 September 2008 (UTC)[reply]
There's no such thing as exact when you measure a real quantity with a finite number of bits. But there's some resolution below which it doesn't matter, functionally. —Tamfang (talk) 07:20, 15 September 2008 (UTC)[reply]

OK, many thanks for all your help. It would seem that looking at the atomic level was a little too ambitious! I have thought about this further and I think that just storaging the location and type of each cell would be OK. Is there a form of storage that could handle that? Thanks again! Jbsouth1971 (talk) 20:20, 13 September 2008 (UTC)[reply]

According to Cell (biology), there's about 1014 cells in the human body, that's 100,000 billion cells. A typical large harddrive currently for sale can store about one terabyte of data, ie, 1012 bytes, so you would need several hundred of those to store meaningful data. Equendil Talk 21:28, 13 September 2008 (UTC)[reply]
OK, so this is starting to look doable. I guess I would need about 100 drives just to store a list of the cells, and more to store other information. Thanks. Jbsouth1971 (talk) 00:07, 14 September 2008 (UTC)[reply]
Well, 1014 cells, each of which need location coordinates, not just "a byte of info". Think how big a cell is, and then you know how accurately you have to specify each one's (x,y,z) coordinates. And both exact position and cell↔cell contact information are critical if you're hoping to (re)create some sort of viable organism from the data. Gotta get the right nerves talking to the right muscles, etc. DMacks (talk) 03:31, 14 September 2008 (UTC)[reply]
Thanks. So if it would need this extra information, can anybody suggest how to figure out how much information would be required to create an accurate representation of an adult human? Along with the compression I mentioned before, it is likley that each cell would only need its location to be defined relative to its neighbours, so I am not sure if that helps minimise the information to be stored (i.e. I am not sure which representation would work best). Jbsouth1971 (talk) 14:50, 14 September 2008 (UTC)[reply]
Without knowing what you will do with the info, there's no way anyone can tell you what info you will need or how accurate it has to be. DMacks (talk) 15:52, 14 September 2008 (UTC)[reply]
How much detail do you need on the "type" of cell? How many cell types are there? I am pretty sure we don't have a clue, if you include the state of the cell. What about each cell's contact with its neighbors? There are innumerable details that contribute to the state of the organism. --Scray (talk) 21:36, 13 September 2008 (UTC)[reply]
Be still my heart. And don't breathe! Saintrain (talk) 22:30, 13 September 2008 (UTC)[reply]
I think that it will probably just need the type of cell in terms of skin, blood, brain etc. Of course, there will be sub-types of those cells, but I guess that all of the types are known. I don't think it will need the contact information, just the exact location at a given point in time. It may well be that there could be some compression here too, I think. For example, if there is a block of identical cells (e.g. all liver cells) then you would just need to store the position of the boundaries, if you see what I mean. Thanks Jbsouth1971 (talk) 00:07, 14 September 2008 (UTC)[reply]
Just warn us before you test your mass teleportation device. Plasticup T/C 02:24, 14 September 2008 (UTC)[reply]
Also, before you decide a teleporter is a good idea (since that's pretty much the only thing any of us can imagine you are thinking about, except for maybe soylent tostito dip) watch The Prestige (film) or read the introduction to the book The Mind's I. --Shaggorama (talk) 07:12, 14 September 2008 (UTC)[reply]
Maybe he just needs a lot of storage space and is disguising its purpose by implying teleportation. --Bowlhover (talk) 15:31, 14 September 2008 (UTC)[reply]
Thanks for all of that info, it was very enlightening. Unfortunately I really can't discuss what the invention actually is, at least not at this stage of the process. Once the whole thing is a little more advanced I will be able to unveil it. Thanks again. Jbsouth1971 (talk) 14:50, 14 September 2008 (UTC)[reply]
Assuming it's a teleporter or cloning device you're inventing, rather than, say, simulating a human on a computer, you may be interested in tissue engineering. Also, google organ printing. That is for how you'd assemble the cells. Note that it takes so long to do this that if you're not printing a rather small organ, it will die as it's being built. Also, it would be impossible to build a brain with such a machine, partially because you'd have no way of wiring the axons, and because AFAIK there's information stored on a sub-cellular level. Anyway, why start with humans? Shouldn't you make a prototype for some simple multi-celled organism, like moss, and work up from there? — DanielLC 15:51, 14 September 2008 (UTC)[reply]

Nuclear waste into space

With the problem of nuclear waste being as long lasting as it is, would not the best option be to chuck the stuff into the sun. Chemical rockets and their occasional explosions would make that into a self-inflicted dirty bomb, but, would not a rail-gun type of launch into space be practical? —Preceding unsigned comment added by 68.193.8.247 (talk) 19:46, 13 September 2008 (UTC)[reply]

See Nuclear_waste#Space_disposal. Sure, if you came up with a totally reliable way of launching it into space, that might work. But we're pretty far away from such an option. And personally I'm still dubious that launching it into space is the "best" option. A carefully managed underground repository in a remote area would still probably be easier and cheaper to deal with. --98.217.8.46 (talk) 19:49, 13 September 2008 (UTC)[reply]
Given science fiction technology, we may as well just annihilate our radioactive waste, and not bother with sending stuff into space. "Railguns" applied to space launch ain't practical, we don't have that kind of technology. Besides, cost considerations more so than technology dictate how we can realistically dipose of radioactive waste. Equendil Talk 21:41, 13 September 2008 (UTC)[reply]
Disposing of nuclear waste in space may be worth considering, and has been suggested, but chucking it into the sun would take an enormous amount of fuel (unless you waited until you can get great gravitational assists from various planets). In order to hit the sum you need to get rid of all orbital velocity you've inherited from Earth, a tiny bit left and you'll just loop round the sun and come back. --Tango (talk) 22:46, 13 September 2008 (UTC)[reply]
If you put nuclear waste in a rocket, there is a chance that the rocket can explode and send back the waste to Earth. 72.136.110.93 (talk) 23:10, 13 September 2008 (UTC)[reply]
Or blow the Moon out of orbit. What happens if we figure out how to recover and reuse that material? --—— Gadget850 (Ed) talk - 00:01, 14 September 2008 (UTC)[reply]
Why don't we (America) just sell it to nations that are willing to reprocess it, or better yet, fix our broken nuclear policy and reprocess it ourselves? — DanielLC 15:29, 14 September 2008 (UTC)[reply]
Selling it to other nations to reprocess is sort of a political non-starter, even if one thought it was a good idea to increase worldwide plutonium inventories. Can you imagine the headlines? "US selling plutonium to Russia (or India, or China, or France)"? The only place I could imagine even contemplating getting away with it would be the UK and even that seems unlikely. --98.217.8.46 (talk) 20:13, 14 September 2008 (UTC)[reply]
Why is the UK any better than any of the other existing nuclear powers? Selling it to a country that doesn't already have nuclear weapons might be an issue, but all the countries you list have them already, so what difference does it make? Of course, you can't completely reprocess it - you're still going to have radioactive waste left, just less of it. --Tango (talk) 13:09, 15 September 2008 (UTC)[reply]

Nuclear explosion and X-Ray lasers

The blast element of a nuclear explosion is mostly caused by the sudden release of x-rays correct? So if we were to create an X-Ray laser like the bomb pumped x-ray laser proposed during the 80s, and it was fired relatively close to the ground, what would the effect look like? The X-rays themselves I know are invisible, but I'm assuming we would see something because the X-rays would create blast when they are absorbed by the atmosphere. ScienceApe (talk) 20:34, 13 September 2008 (UTC)[reply]

A nuclear blast has three devises that cause damage: the blast itself/expansion of gases, intense heat, and nuclear radiation. Nuclear radiation traditionally consists of alpha particles, beta particles, and gamma rays (not X-rays.) I don't think you would see much if you released a bunch of X-ray's, just like you can't see anything when they are used to photograph your teeth or bones. Unless by some chance they excite the atmosphere, which I don't think is possible. X-rays are not well absorbed by the earth's atmosphere. --Russoc4 (talk) 20:43, 13 September 2008 (UTC)[reply]
I think you are confused a little. I'm not referring to the total effects of a nuclear explosion, just the blast effects, which is different. The effects of a nuclear explosion consist of,
    Blast—40-50% of total energy
    Thermal radiation—30-50% of total energy
    Ionizing radiation—5% of total energy
    Residual radiation—5-10% of total energy
Now, I'm only referring to the blast effects, not the other three. With that in mind according to, http://en.wikipedia.org/wiki/Effects_of_nuclear_explosions#Blast_damage, blast is indeed mostly caused by the x-rays. It says "shock wave is inside the surface of the developing fireball, which is created in a volume of air by the X-rays.". Additionally the the particles and the gamma rays you mentioned appear to be associated with ionizing radiation, which is another effect that I'm not asking about. ScienceApe (talk) 20:57, 13 September 2008 (UTC)[reply]
That article is rather weak on the actual physical mechanism whereby soft X-rays actually form the fireball, saying only (of the air) "this radiation interacts with and rapidly heats it". How this actually happens is better described in this article. What's not clear to me is what proportion of the visible spectrum light is due to direct incandescence of the superheated atmosphere atoms (I confess to not really understanding what "heating" an atom really means) and what proportion is photoelectric fluorescence. This paper discusses atmospheric fluorescence due to the x-ray emissions of nuclear explosions. I guess in answer to your "what would it look like" the answer is going to lie somewhere between "discrete death beam" and "terrifying glowing blurry death sausage". Either way it's not going to reach very far (perhaps a mile). Russoc4: as these papers show, X-rays are very much absorbed by the atmosphere, the first paper saying "the X rays are absorbed in the immediate vicinity of the burst, and they heat the air to high temperatures. This sphere of hot air is sometimes referred to as the "X-ray fireball." -- Finlay McWalter | Talk 21:48, 13 September 2008 (UTC)[reply]
Interesting off-topic factoid that you should not try to verify: if your eyes are well-adjusted in a darkened room and you look at a strong X-ray source, you can see a ghostly gray light, according to Röntgen. --Sean 20:42, 14 September 2008 (UTC)[reply]

Not making nutrients as an evolutionary "tactic"

While stirring my grits this morning and thinking about incomplete proteins (no, really) it occurred to me that, as omnivores, we didn't need to make many of our "secondary" nutrients since we could just harvest them. What is the cost of making vitamins? Would such a cost-avoidance provide a significant evolutionary benefit? Thanks Saintrain (talk) 21:05, 13 September 2008 (UTC)[reply]

I'm not sure what you are talking about. There are lots of things that we evolved that we didn't "need" or there are better alternatives to what we have. I don't know the cost of making vitamins, but I do know that just taking vitamins without food, tends to not be assimilated properly. You usually need to take vitamins with food in order to absorb the vitamins correctly. Could you clarify what you are asking a little more? ScienceApe (talk) 21:16, 13 September 2008 (UTC)[reply]
I'm talking about the "old days". What is the metabolic cost of making, say, vitamin C. Did avoiding the cost of making vitamin C provide a significant evolutionary benefit? Saintrain (talk) 21:34, 13 September 2008 (UTC)[reply]
If the diet provides all of the vitamin C (your example) needed, then there would be no incremental benefit in developing the mechanisms to synthesize it. So, the question becomes how much our inability to make vitamin C limited our fitness. --Scray (talk) 21:45, 13 September 2008 (UTC)[reply]
(ec) Maybe it did. The other possibility is simply genetic drift: given a diet rich in Vitamin C, our ancestors had no need to produce it, and thus no selection pressure against mutants with inactive L-gulonolactone oxidase. Apparently, the haplorrhini, including humans, all descend from a lineage with such a mutation. —Ilmari Karonen (talk) 21:49, 13 September 2008 (UTC)[reply]
To emphasize what was just said, it isn't that losing the ability to make vitamin C was beneficial, it was that there wasn't any selective pressure to keep the synthesis genes. When random mutations knock out the gene's function, the animal can still survive, and pass on the "dead" gene to its offspring - it's not better than one who can, but it's not dead, either. This was how the essential amino acids were explained to me. Since humans have historically gotten enough of most of the amino acids in the diet, there isn't any pressure to keep the synthesis genes around, and you slowly lose their function. The pathways which require more genes to make are lost first, as there is more chances that one of the genes will be knocked out by random mutation. -- 128.104.112.147 (talk) 22:00, 13 September 2008 (UTC)[reply]
There's a thread on the "Darwin-L Message Log" on this very topic but it, too, was just hand waving and diarrhea and polar bear livers(!?). Noone answered the question "What is the metabolic cost of making vitamin C".
The best that several variations of google(metabolic cost liver ascorbic "vitamin c" synthesis site:.edu) turned up was that vitamin C was so common in other species were the suppositions were that it was very important and/or very cheap to make. (Lots of veterinary schools because humans don't make it. (Calves can make it by day 7!)) Lots of taking Dr. Pauling's name in vain, too, 'til I added the "site:.edu". What's a better set of search terms? Saintrain (talk) 01:09, 14 September 2008 (UTC)[reply]
You might want to look at the papers linked from the pertinent page on OMIM. - Nunh-huh 01:17, 14 September 2008 (UTC)[reply]
That is a very cool site. Thanks! There's all kinds of info there. Never realized how universally important vitamin C is, even to yeast. (There's even a speculation that the lack of vitamin C resulted in a higher oxidant-related mutation rate in Hominids!) But it's hard to find the cost of v-C biosynthesis in humans ('cause we don't do it). I'm about to give up. There is a reason why I didn't study biology. :-) Saintrain (talk) 00:33, 15 September 2008 (UTC)[reply]

Why is 16:9 superior to 4:3?

I look at screens all day. So do most people I know. It seems we have made a choice as a society to go with a wider, shorter resolution rather than the more square-like box. I can think of 2 reasons why this doesn't make sense:

1) We are tall, thin beings. A wider resolution impedes caturing our counteance on screen. It is disturbing to see so much landscape left and right of a human figure in the center. 2) Reading is always easier when done vertically. If a line becomes too wide it is too easy to get lost when trying to find the next line, and, far more importantly, it hinders speed reading. Sucking up short phrases is easily done in a vertical newspaper format, no inner voice required. Sappysap (talk) 22:40, 13 September 2008 (UTC)[reply]

It is not. 16:9 is a compromise TV aspect ratio between standard 4:3 and 2.35:1 movie aspect ratios. It is already not a very good solution for TV broadcast, because now you have black-bars everywhere, and when you apply it to computer monitors it's even worse. Sure now you could watch movies with less black bars but as you have said reading web pages become much more painful and whatever you have gained in less black bars when watching movies you have lost it in fixed-sized web pages that leave 50% of your monitor blank. It's mostly a fad (and this can be extended to those 16:9 cameras), and IMHO what you lose (precious vertical resolution) is much more important than what you gain (less black bars when watching movies, which I hardly ever do anyway). --antilivedT | C | G 23:47, 13 September 2008 (UTC)[reply]
Some computer monitors (have have a rather nice Dell one) allow you to rotate a 16:9 through 90 degrees, making it a stringy 9:16. At least for text-based stuff like web-browsing and word processing that turns out to be rather nice. -- Finlay McWalter | Talk 23:52, 13 September 2008 (UTC)[reply]
Isn't our natural range of vision more oblong? Presumable the most acute portion of our vision is an ellipse inside the larger ellipse of our total range, and the largest quadrilateral that can be inscribed in that ellipse is a rectangle. Plasticup T/C 03:10, 14 September 2008 (UTC)[reply]
It's better for pictures as it corresponds more to how we view the world - up and down aren't as important as left and right. Holding my fingers up I can see about two and a half times as much or more sideways with both in view at the same time as I can up and down. But yes I agree, for reading it's no good currently. There is a proposal to support columns of text better in HTML so one automatically gets one, two or more to fit the screen complete with text flowing between them. At the moment doing that is a pain which is why it isn't done more frequently. Dmcq (talk) 09:22, 14 September 2008 (UTC)[reply]
I've actually held off on buying a new laptop partly for this reason. I'm hoping that the fad will blow over, but maybe I'm deluding myself. I've always suspected that part of the reason is that screens are marketed by diagonal instead of area. A 16:9 screen with a given diagonal is about 11% smaller than a 4:3 screen with the same diagonal and correspondingly cheaper to manufacture. It's a good thing digital cameras are marketed by an areal unit (megapixels) or we might be deluged with widescreen digicams also. -- BenRG (talk) 11:47, 14 September 2008 (UTC)[reply]

Masturbation

At what average age do girls start masturbating? —Preceding unsigned comment added by BellyGrease (talkcontribs) 22:41, 13 September 2008 (UTC)[reply]

See Masturbation#Masturbation frequency, age and sex (fourth paragraph). Zain Ebrahim (talk) 22:49, 13 September 2008 (UTC)[reply]

end of the world : december 2012?

hi guys, i am curious about what is the concensus among scientists about all the predictions that the world may end on 2012 because of all the clues that point to that happening, like the sun spot cicle ending, poles shifting, the mayan prophecy, global warming...and all that...and also the LHC possibly creating a black hole!!! thank you! —Preceding unsigned comment added by 24.91.11.185 (talk) 22:52, 13 September 2008 (UTC)[reply]

See Wikipedia:Reference desk/Archives/Science/2008 August 15#about 20 dec 2012. There are some links to older questions in there too. Zain Ebrahim (talk) 23:00, 13 September 2008 (UTC)[reply]
Grooooaaaaaaan. See [23], [24], [25], [26],[27], and [28]. --Bowlhover (talk) 23:21, 13 September 2008 (UTC)[reply]

lol, thanks guys. but also...i think it is too much of a coincidence that the mayan calendar ends in 2012, terrence mckenas' whatever diagram ends in 2012 too, (the one that is based on the i ching), the LHC poses a minimal threat of creating a dangerous mini black hole and nostradamus has a prophecy about this machine, look it up on youtube. also the sun ending its current sun spot cicle, the solar system aligning itself with the center of..the galaxy i think. so there is a lot of stuff that seems correlated with that date. i am a skeptic too...but the world just seems so close to ending...i mean, the war in irak and stuff...i bet a bunch of countries hate the US and are planning evil things against it. like a nuclear attack , a viral attack and what not. —Preceding unsigned comment added by 24.91.11.185 (talk) 23:57, 13 September 2008 (UTC)[reply]

The Mayan's did not predict that the world would end in 2012. That is a total fabrication. I spoke to Dr. Mary Miller about this just a couple years ago, and she is widely regarded to be the foremost authority on pre-Columbian Mesoamerica. Plasticup T/C 00:01, 14 September 2008 (UTC)[reply]

as far as i know their calendar abruptly ends in december 2012...but i mean, i don't want the world to end. so if all this is just pseudoscience then i'm glad.

thank u guys for all your answers! —Preceding unsigned comment added by 24.91.11.185 (talk) 00:06, 14 September 2008 (UTC)[reply]

Also, the sunspot cycle repeats every 11 years, so this wouldn't be the first time. ~AH1(TCU) 00:39, 14 September 2008 (UTC)[reply]
OK, let's go:
  • sun spot cycle ending
The Sun has a 9-14 years cycle of activity. It's not really precise, and if one were to end in 2012 it would be just a coincidence.
  • poles shifting
Completely unpredictable when it will happen.
  • Mayan prophecy and calendar
The prophecy is a fabrication and this has been debunked several times.
  • Global warming
2012 has no relation to global warming.
  • LHC creating a black hole
Pretty much debunked. There's nothing to fear.
  • Nostradamus prophecy
Nostradamus is a joke. There's no "prophecy" worth bothering. They're always found in hindsight and are usually passages vague enough to fit a lot of situations - just like bible and other prophecies.
  • the solar system aligning itself with the center of the galaxy
This makes no sense. Two points are always "aligned". Planets alignment are worthless too.
  • the world just seems so close to ending
Nah, we've been through much worse times. We're in a relatively peaceful time, really, and throughout history there has always been people claiming "the world is just about to end, you just have to look at the signs". Yet, nothing ever happened. Why do people always forget failed predictions? For example, Christians have been thinking the Second coming was going to happen within their lives for centuries.
So you see, this is not even pseudoscience. It's bullshit. Just move on and don't bother with these apocalypse fears, because they have always been wrong throughout history. But if someday you see scientists reporting an actual natural threat, then you have reasons to believe them. — Kieff | Talk 02:08, 14 September 2008 (UTC)[reply]
If you look hard enough, any year has peculiarities that, if isolated, seem surprising. I can easily claim that 2009 is going to be the best year on record by pointing to the 1909 article. Just look at what happened during that one short year! The People's Republic of China was established, and it was this party that ended 2000 years of monarchy in China. Dessau became the first radio broadcaster. Robert Peary became the first person to reach the north pole. The Pearl Harbor base was founded in November. The person accused of setting fire to the Reichstag was born. All of these events are much more well-known than the "predictions" involving 2012! Who doesn't know about the ROC? Who doesn't know about radio broadcasts, Peary, the Reichstag fire, or Pearl Harbor? But who, without having read about the "apocalypse", would know about some obscure Mayan text or McKennas' graph?
It's true that the events I mentioned do not make it more likely that 2009 will be the best year on record, but celestial alignments, the LHC, the solar maximum, and pole shifts don't increase the likelihood of an apocalypse either.
I assume that this is the Youtube video you're referring to. The Nostradamus prophecy, which does not mention the year 2012, reads:
"All should leave Geneva. Saturn turns from gold to iron. The raypoz will exterminate everything. There will be signs in the sky before this."
Only the first sentence suggests the LHC; "raypoz" is not a French word and is not always interpreted as "positive ray". This website, for example, believes it means "Christ". The poem might as well be construed as a prediction of a war in which Switzerland is taken. As for the other passages the video quotes, none of them accurately describe an engulfment of Earth by a black hole. --Bowlhover (talk) 05:14, 14 September 2008 (UTC)[reply]

From a scientific standpoint, which plant is the most important?

By "important" I mean what species of plant has the most value from a guys in a lab coat researchable, medicinal, and physical law standpoint? Which lends itself best to the most rigorous scientific tests, yet can still make and cure just about anything?Sunburned Baby (talk) 22:58, 13 September 2008 (UTC)[reply]

I'll nominate arabidopsis thaliana aka thale cress. Google Scholar returns 680,000 hits for "arabidopsis", that's a lot of scholaring! It's widely considered a model organism. Doesn't really cure stuff though. Franamax (talk) 01:43, 14 September 2008 (UTC)[reply]
Interesting question. On a whim, I checked soybean and got 996,000 hits. Of course, since oak gives us more than a million hits, I'm not sure how good a gauge Scholar really is. Wheat gives us two million hits. Matt Deres (talk) 02:08, 14 September 2008 (UTC)[reply]
Heh. Turns out there are a lot of people named Wheat doing research out there; forget that one! Matt Deres (talk) 02:09, 14 September 2008 (UTC)[reply]
Maybe so, but all those people named Wheat are undoubtedly descendants of wheat farmers, so if not for wheat we would have a HUGE loss of valuable researchers. Therefore, those hits still definitely count! --Scray (talk) 04:55, 14 September 2008 (UTC)[reply]
But why are the people named Potato being discouraged from going into scientific research fields? Systemic bias? Franamax (talk) 17:09, 14 September 2008 (UTC)[reply]
It very much depends on how you define important. Arabidopsis was the first plant to have its genome sequenced and widely used in genetics studies. (And there are no researchers named Mr. Arabidopsis) Important food crops such as soybean, maize, rice and wheat attract attention from agronomists because of their importance to the world, and also from geneticists because of their very complex genomes. And by historical scientific importance, you could think of the pea also, since that's where Gregor Mendel first studied genetic inheritance. Franamax (talk) 17:09, 14 September 2008 (UTC)[reply]

The effects of Ocean Nuclear Bomb Testing

What are the effects of the Nuclear Bomb Testing on the Pacific Ocean environment? Some water is now radioactive, so how long does it takes for the radioactive water to become non-radioactive water? 72.136.110.93 (talk) 23:04, 13 September 2008 (UTC)[reply]

If the water had become radioactive then more by taking up products of the fission, less by being irradiated. Anyway, the water soon mixes. The problem is the island soil that took up long-lived fission products. See that article for some ideas. On a similar note, there are still forests round Munich where collecting of mushrooms for culinary purposes is still discouraged, twenty years since Chernobyl, and more to the east it must be worse. --Ayacop (talk) 14:01, 14 September 2008 (UTC)[reply]
Our article, Pacific Proving Grounds, does not mention water contamination for the reasons stated above. More precisely, you need to define "radioactive water" and "non-radioactive water" before you can proceed. Seawater is water (Oxygen and Hydrogen) with many disolved substances (ions and some non-ionic molecules.) The readiosctivity results from radioactive isotopes of some of the atom in the water or in the disolved substances. There is a natrual background of radioactive isotopes, so one answer to your question is "never." If you define "non-radioactive" as the point at which the water near the testing ground is at or very near the background radiation, I would guess that this happens within weeks due to radioactive dacay and dilution.If you ar worried about health effects, you will need to analyze individual elements like iodine, but even these will drop to background quickly due to dilution. -Arch dude (talk) 14:23, 14 September 2008 (UTC)[reply]
The water isn't the problem—the ocean is just too vast not to dilute such effects to the level of background radiation within a short amount of time. --98.217.8.46 (talk) 15:31, 14 September 2008 (UTC)[reply]

September 14

Thallium Stress Test (Side Effects)

I am wondering if anyone has had a Thallium Stress Test ? If so; did you have any side effects ?

My cousin just had a test and since then has had terrible discomfort in his chest area that he never had before.

Thank You . —Preceding unsigned comment added by 74.235.199.159 (talk) 00:29, 14 September 2008 (UTC)[reply]

Well, it is a stress test. It stresses the heart, and most people who take the test either have known or suspected heart disease. So a report of discomfort following such a test needs to be made to a doctor, not to the Wikipedia reference desk! If your cousin has chest pain, he needs to call his doctor, so that it can be determined if he's suffering a heart attack or not! - Nunh-huh 00:49, 14 September 2008 (UTC)[reply]

Contact the doctor. That is what they are there for.--mboverload@ 02:18, 14 September 2008 (UTC)[reply]

No, call for an ambulance unless there is a doctor right at hand. "Terrible discomfort in the chest area" is an emergency situation! --Anonymous, 05:10 UTC, September 14, 2008.

If you had a stress test they must have give you an emergency contact number. Use it. Plasticup T/C 02:30, 14 September 2008 (UTC)[reply]

Chemical Name

I'm trying to figure out the name of a molecule, but I'm out of combinations. It looks like cyclopentane, but there is a nitrogen atom in place of one of the carbon atoms. No, this is not homework or anything, I'm just curious.CalamusFortis 04:54, 14 September 2008 (UTC)[reply]

Perhaps Pyrrolidine? You don't mention how saturated the molecule is, but maybe this helps. --Scray (talk) 05:02, 14 September 2008 (UTC)[reply]
That was it exactly. Thank you. Forgive me for neglecting to mention saturation.CalamusFortis 05:18, 14 September 2008 (UTC)[reply]
For future, you can search google or wikipedia for the chemical formula ("C4H9N" in this case) for many organic compounds and find their name and relevant articles. ChemSpider is another good free chemical search engine. DMacks (talk) 15:49, 14 September 2008 (UTC)[reply]
For browsing according to structure, the WikiCommons compounds tree is near perfect. --Ayacop (talk) 16:02, 14 September 2008 (UTC)[reply]

Swimming Pool (Hydrochloric) Acid

What is the gravelly, grinding noise that can be heard when pool acid is poured from the bottle into another vessel, or swirled in its own bottle ? It sounds as if there is a layer of gravel scratching around the bottom of the plastic container. --196.207.47.60 (talk) 05:00, 14 September 2008 (UTC)[reply]

Dark matter

wat is meant by dark matter? —Preceding unsigned comment added by Donlesnar (talkcontribs) 05:47, 14 September 2008 (UTC)[reply]

see dark matter. --Shaggorama (talk) 06:52, 14 September 2008 (UTC)[reply]

Name for animals that can change colour

Does anyone know what the scientific name for this capability among animals is? We have a list at Category:Animals that can change colour and a request to fill in this missing information. (Can;t seem to link to it but it's there in fact) Thanks, Julia Rossi (talk) 11:46, 14 September 2008 (UTC)[reply]

It's at Category:Animals that can change color. DuncanHill (talk) 11:53, 14 September 2008 (UTC)[reply]
(edcon) yes it is thanks DuncanHill, Julia Rossi (talk) 11:58, 14 September 2008 (UTC)[reply]
For the edification of other readers, you link to a Category (or Image) like so: [[:Category:Animals that can change color]] Plasticup T/C 16:08, 14 September 2008 (UTC)[reply]

The mechanisms responsible for colouration strategies differ between species depending upon the nature of the pigment, the control over pigment distribution and the time frame over which pigments change. If different species do it in different ways, there may not be a catch-all phrase. Plasticup T/C 16:11, 14 September 2008 (UTC)[reply]

Oh, I was hoping for something like "changer of colour" or "environment mimic" in Latin, but never mind. Thanks anyway, Julia Rossi (talk) 10:05, 15 September 2008 (UTC)[reply]

Things that weigh on Milligram.

What are some common things that weigh one milligram? —Preceding unsigned comment added by 80.148.22.232 (talk) 12:30, 14 September 2008 (UTC)[reply]

There's one at Orders of magnitude (mass). Fribbler (talk) 12:51, 14 September 2008 (UTC)[reply]
(For those who don't want to click through, it is a mosquito) Plasticup T/C 16:06, 14 September 2008 (UTC)[reply]
Five grains of table salt.--Stone (talk) 17:03, 14 September 2008 (UTC)[reply]

VEGA LAUNCHER

My cousin (a nuclear enginneer) said that Vega can be used also as military rocket.Which distance is it able to do? Thanks. —Preceding unsigned comment added by Vindobona (talkcontribs) 14:02, 14 September 2008 (UTC)[reply]

Not sure what you're refering to. Do you mean Vega (launcher), Lockheed Vega, or Percival Vega Gull? --Ayacop (talk) 14:26, 14 September 2008 (UTC)[reply]
Given that the title of his question is "Vega Launcher", it seems a safe bet that he is referring to Vega (launcher). All of your questions are answered in that article. The first launch is scheduled for 2009. Plasticup T/C 16:04, 14 September 2008 (UTC)[reply]
If you realy want you can use the space shuttle or any other rocket. For military the liquid stage especialy LOX LH2 stage is stupid, you can not leave it in the silo for long an fueling the thing costs time, so most military nuclear rockets are solid fuel and the last upper stage of Vega is only for orbit insertion. So yes, but for which country? France und GB have already what they want, and the other ESA members have no nuces.--Stone (talk) 17:00, 14 September 2008 (UTC)[reply]

Italy has (like other EU contries) is able to build a nuke actually very quickly.It doesn't build because he decided to do not build it and so itself decided it! My cousin teaches nuclear physics in Milan.He says that country like Italy officially haven't the nuke because of NPT,but Italian Army did several years ago what it wanted.It had nuclear research centres only for military aims.Vega derives from Scout so he said it's very good for military aims(it's also too much!).He told me also that EU (by a small law wanted by Italy and Germany;at that time EU didn't exist)can declare itself nuclear power.The same thing i heard by the Italian ambassador in UN.I think they know more than us. PS Nowhere is written that you have to keep the rocket in a silo especally if you set it on something moving. Thanks. —Preceding unsigned comment added by Vindobona (talkcontribs) 18:41, 14 September 2008 (UTC)[reply]

To build a nuce is not much of a poblem if you have a HEU research reactor. But to declare to be a nuclear power would make you a nation nongrata in europe. One thing is you have to get all the others helping you to build the Vega for you, and France and GB for sure will not give you the Vega parts they provide for a rocket. To have a a couple of Vegas somewhere else than Koru makes only sense if you build them for military purpose. The silo or on a truck does not matter the -200°C liquid will give you trouble.--Stone (talk) 20:54, 14 September 2008 (UTC)[reply]


In UK are built no parts of Vega. Italy builds the 65% of launcher and it's able to build everithing it wants.Be sure!I haven't yet received an answer about its distance on Earth! —Preceding unsigned comment added by Vindobona (talkcontribs) 09:53, 15 September 2008 (UTC)[reply]

Thyrotropin Releasing hormone in Gastrointestinal tract

What does TRH do in the gastro-intestinal tract? can someone give further links to reading sources? 117.97.145.59 (talk) 14:06, 14 September 2008 (UTC)[reply]

According to de:Thyreoliberin, any effects are indirect, i.e., through stimulation of nerves in the brain. We have (no references, sorry)
  • through the vagus, stimulation of stomach acid production and peristalsis
  • through the sympathetic nerves, stimulation of insulin secretion
  • stimulation of exocrine function of the pancreas
--Ayacop (talk) 14:35, 14 September 2008 (UTC)[reply]

Dog ESP

I went to my friend's home with him. His home is in another city, some 7 hours away by train. He was away from his home for several weeks. When we got there their dog was waiting at the gate for him. His dad told us that the dog had been standing outside and waiting all day long, as if he knew my friend was coming. He said it happens every time he's about to come home. Any scientific explanation for this? ReluctantPhilosopher (talk) 14:22, 14 September 2008 (UTC)[reply]

I'm willing to bet it's more a matter of confirmation bias and the father just ignores all the other days when the dog is waiting by the gates. -- MacAddct  1984 (talk &#149; contribs) 15:12, 14 September 2008 (UTC)[reply]
Or the dog picks up on changes in the father's behavior on the days when the father expects a visit. Or both. - Nunh-huh 15:19, 14 September 2008 (UTC)[reply]
I'd go with what MacAddct and Nunh-huh are saying. I'd also like to add that this is a classic case of anecdotal evidence, which is unreliable. There are all sorts of reasons why the premise -- that the dog always waits for him all day -- could be wrong: perhaps the father exaggerates (consciously or otherwise), because it makes for a good story, or because it reinforces the idea of a loving and loyal dog, or because he knows it makes your friend feel good, for example. In order for us to even begin honestly entertaining the idea that the dog is actually doing this (and I think the other explanations are far more likely), there should be some kind of actual evidence of the dog really standing by the gate all day long on the days your friend is coming home (and not doing so on any other day). -- Captain Disdain (talk) 15:34, 14 September 2008 (UTC)[reply]
My dog always knows when my wife and I are about to go on a trip—all day long she frets about us leaving in a very distinct way. She also knows when people are going to be visiting and is similarly antsy. How does she know these things? She's not psychic (she's many things, but it's clear she's not that!). She knows we are going to go on a trip because she sees us doing things we don't usually do (e.g. packing). Before people come over, we are always cleaning things up that usually we don't bother with. The dog observes us, and can pick up on very subtle changes in activity and emotion. The dog always knows when one of us is sick. She isn't psychic, she's just able to tell when we're doing things we don't normally do. Dogs are very capable of noticing routines (including the much beloved "time for dinner!"), and are very keen to what differences in them mean. One of the things we love about our dog is that after seven years or so we all understand each other very well. We can tell what she is thinking and feeling just as well as she can tell what we are thinking and feeling. It's not ESP, it's empathy. --98.217.8.46 (talk) 16:14, 14 September 2008 (UTC)[reply]
Another possibility is that your friend's Dad told the dog he was coming home, mentioned his name, opened the door to his bedroom, things like that. When my Mom tells her dog I'm coming to visit, he insists on staying out in the yard with his eyes on the gate until I get there. I bring doggie treats for him you see and pat him a lot. He knows exactly what my name means when it's linked with "is coming". Franamax (talk) 17:39, 14 September 2008 (UTC)[reply]

What's it called when...

You look at a lamp or the sun and it leaves an imprint in your eye, which discolours what you are looking at, making it purple, black-and-white, or just false colour?--Editor510 drop us a line, mate 16:56, 14 September 2008 (UTC)[reply]

It is called an after-image. DuncanHill (talk) 17:01, 14 September 2008 (UTC)[reply]

Help identifying flowers

Hi. I'm trying to identify the species of a couple of plants from my mother's garden. For geographical/climate information, these plants are growing in North Texas.

The first is a vine with bright pink flowers that bloom at night. (These photos are taken during the day):

These others are delicate purple blooms:

If they turn out to be pictures we could use, I'll re-upload them with more useful file names. Thanks in advance to anyone who can help. -GTBacchus(talk) 19:12, 14 September 2008 (UTC)[reply]

Do the pink flowers have a sweet scent? They look like a honeysuckle to me. DuncanHill (talk) 19:32, 14 September 2008 (UTC)[reply]

I think the pink night-bloomer might be Angel's Trumpet (Datura). According to our article on that, it comes in pink, is shaped like the flowers in your picture and blossoms at night. 128.239.177.28 (talk) 13:26, 15 September 2008 (UTC)Hana[reply]

Ornithopters

Lets say we want to turn the F-22 into an ornithopter. Its empty mass is about 19,700 kg. If we assume its wing area remains the same (840 ft² (78.04 m²)), how fast does it have to flap in order to hover? I'm aware of the technological limitations of such things, but for this thought experiment, lets just ignore how we could build such a thing. ScienceApe (talk) 19:36, 14 September 2008 (UTC)[reply]

Simply flapping flat wings won't produce any lift at all, since you're doing a symmetrical motion on both up and down strokes. The wings would have to bend somehow, and the rate of flapping required will depend on how that bending is done. --Sean 12:58, 15 September 2008 (UTC)[reply]

Inverse Compton effect and nuclear weapons

I was told by someone who was very much involved in the US H-bomb program that in the 1950s the inverse Compton effect was regarded as a big secret because it had major implications for hydrogen bomb design.

Can someone more literate in physics explain to me why this might be so? And if the Compton effect is relatively common, would it be much to expect that someone scientifically literate would not happen upon the likelihood that the inverse Compton effect would exist and have relevance to such an endeavor? The article on Compton scattering didn't really help me in wrapping my mind around this question, other than indicating that the inverse effect seems pretty rare under Earth conditions (i.e. it seems like it only takes place in regards to astrophysics, and presumably in nuclear weapons though it doesn't say that). --98.217.8.46 (talk) 20:10, 14 September 2008 (UTC)[reply]

If you google h-bomb and inverse compton effect, there's some documentation to be found. For instance according to [29] the i.c.s. effect created a problem for the H-bomb by cooling the electrons, but the challenge was more in doing the calculations surrounding this, and they may have been kept a secret. I believe i.c.s in itself must have been well known in the 1950s. EverGreg (talk) —Preceding undated comment was added at 21:57, 14 September 2008 (UTC).[reply]
The fellow who told me about it said that it wasn't well-known at all at the time, though he conceded that if you thought about it, it wasn't too hard to imagine that it would exist (he thought the idea of trying to keep something like that a secret was stupid). I interpreted this as meaning that it's the sort of effect that really doesn't come up under earth regimes, and so would be counterintuitive unless you were really trying to figure out things in extreme regimes (like the idea that solid plutonium can be compressed under extreme pressure regimes, which only occurred to Edward Teller because he had been working with Gamow on things relating the core of the earth).
What I'm asking for, basically, is a straightforward explanation by someone who understands the physics, for a non-scientist. --98.217.8.46 (talk) 22:56, 14 September 2008 (UTC)[reply]
Oh, an explanation! :-) That's actually very easy. The compton effect is all about a photon (light) and an electron colliding. These hit each other like billiard balls and recoil in opposite directions. In typical situations, it is the electron that speeds up after the collision and the photon looses energy. It still moves at the speed of light, but with a longer wavelength. "Typical" as in typical photon energies and typical electron speeds. But in some situations, like in astrophysics, some physics experiments and the H-bomb, the electron is moving so fast it is the photon that gains energy after the collision. For an H-bomb this seems to have been a bad thing because the energy "leaked out" of the process with the photons. But I'm not familiar with the details here. If you like, you could perhaps edit the compton effect article to provide a better-worded lay-man explanation. EverGreg (talk) 08:19, 15 September 2008 (UTC)[reply]

Why is gold leaf opaque when it is only one atom thick and each atom is mostly empty space?

Please could someone explain why it is not easy to see through gold leaf, when the material is only one atom thick? It is possible to see through the material if you hold it close but it is not transparent in the same way that air or many types of glass are.

My understanding of the scale of the atom is that if the nucleus was the size of the circle at the centre of Wembley Stadium, then the electons would be vastly smaller and whizzing around within a cloud the size of the whole stadium.

I also understand that gold has 79 electrons, but these are so, so small then surely, even as a fast-moving cloud I cannot believe that they could form the illusion of a solid space to the human eye.

Using the same argument, I also struggle to understand why it is possible to see through vast volumes of air. On a clear day, from a reasonable height, it is possible to see around 20 miles (and much further from the top of some skyscrapers). When looking this far, the observer is looking through a block of air. I would have thought that, as sparse as the atoms/molecules in air are, over this distance the chances are that there would be many atoms or molecules 'obstructing' the observer's line of sight - yet he or she can still see through.

So...what property of materials makes us able to see though some and not others (oher than their depth)? Thank you Jilly (talk) 20:21, 14 September 2008 (UTC)[reply]

One thing that comes to mind is that your gold leaf is thin but it is tightly packed, and the atoms aren't moving. In the air, you have a very low density collection of atoms that are constantly moving about. It would seem to me to be the equivalent of shooting bullets at a tight-knit chain link fence in the case of gold, whereas air would be more like setting up a long tennis court with a bunch of constantly bounding balls. In the latter case you are likely to hit one every once in awhile but a constant spay would make it through pretty well; in the former case each shot has a pretty high chance of getting scattered in the same general area. But this is obviously a very off-the-cuff sort of reasoning. And it doesn't take into account transparent solids like glass, obviously, though I would guess that glass is more like the chain linked fence, but with very wide linkages. --98.217.8.46 (talk) 20:34, 14 September 2008 (UTC)[reply]
A few things here:
  • Is there actually gold leaf one atom thick ?
  • Light interacts with matter through electrons that absorb and reemit certain wave lengths (so basically, what we "see" is the electrons really).
  • Regarding air, not all light makes it through, that's why for instance the sky is blue (as some wave lengths are absorbed), some light is scattered (things become blurry in the distance), ultimately, it's just a matter of distance and density of the air, it just so happens the atmosphere on earth is not "thick" enough to block/scatter light entirely (whereas visible light on Venus is too scattered for us to see anything of the surface). Equendil Talk 20:59, 14 September 2008 (UTC)[reply]
(edit conflict with Gandalf61) I am extremely skeptical that someone could make gold leaf that was just one atom thick. Our article on Goldbeating says that "The finished leaf forms an unbroken sheet of gold with a thickness of 1/250,000 of an inch." Now, that's really amazingly thin, absolutely, but nonetheless, the leaf is still 0.0000001016 meters thick. That's pretty hefty compared to the "size of a typical atom, which is 0.000000001 meters across", according to this RHIC physics primer. That's plenty of room for a whole bunch of atoms to be stacked on top of each other. -- Captain Disdain (talk) 21:27, 14 September 2008 (UTC)[reply]
Our goldbeating article says that gold leaf is 1/250,000 of an inch thick - that's about 10-7 m or 100 nanometres. Typical atomic radius is 0.1 nanometre, so gold leaf is hundreds of atoms thick. Gandalf61 (talk) 21:19, 14 September 2008 (UTC)[reply]
When you get gold leaf extra thing it does permit light to move through and becomes nearly transparent when really thin. Graeme Bartlett (talk) 21:28, 14 September 2008 (UTC)[reply]
Indeed. Most space suits for extra-vehicular activity have their outer visor coated in a thin layer of gold, to protect the occupant from the intense sunlight. The astronauts actually look through the gold layer. (I'm having a hard time finding a WP article that says so explicitly, the best I can come up with is Krechet - note the gold-colored viewing bubble.) -- 128.104.112.147 (talk) 21:52, 14 September 2008 (UTC)[reply]
Well, the article does say, "The outer visor was coated in gold for reflectivity." That's pretty explicit! -- Captain Disdain (talk) 22:00, 14 September 2008 (UTC)[reply]
The outer visor is coated in gold. Not gold leaf. - Nunh-huh 22:53, 14 September 2008 (UTC)[reply]
Gold leaf is just an extremely thin layer of gold. What distinction are you trying to draw here? Algebraist 00:30, 15 September 2008 (UTC)[reply]
While the "diameter" of an atom is about 100,000 times that of its nucleus(!), it's the (outside) electrons that interact with the light rays. The distance between gold atoms in its crystal structure is about 270 picometres. (Gold remains a crystal even after you beat the crap out of it, that's why it's done so much.) Visible light has wavelengths that vary between about 400 (blue) and 700 (red) pm (less than an octave!).
See-through gold (visors etc.) is "sputtered" on to the surface. It's not a continuous crystal layer but a bunch of dots with lots of space in between (if you're a photon).
Transparent materials like glass don't "allow" light to "pass through" (silicon and oxygen atoms, the principle components of glass, are (about) the same size as gold and as tightly packed and made of exactly the same stuff.) The atoms "relay" (I know. I know. I can't think of a better word.) the light all the way through. Saintrain (talk) 01:52, 15 September 2008 (UTC)[reply]

Prostate Treatment

The articles on Prostatitis and on BPH don"t mention treatments such as ejaculation, eating radishes, drinking cranberry juice. Is this type of treatment addressed somewhere? They do mention avoiding caffeine as a prevention. Also, if antibiotics such as cipro and doxycycline alleviate symptoms for a period of months is there an antibiotic that offers longer relief? Bigprostate (talk) 20:35, 14 September 2008 (UTC)[reply]

You can edit article to improve them, this is wikipedia the encyclopedia you can edit. We cannot offer medical advice here though. I have linked your words in case someone want to check out the articles. Graeme Bartlett (talk) 21:40, 14 September 2008 (UTC)[reply]
Do you have any sources that suggest that those treatments are commonly used, or effective? Wikipedia articles need reliable sources to back up their claims, and in this instance I'd say a medical journal would be about the level required. Confusing Manifestation(Say hi!) 23:41, 14 September 2008 (UTC)[reply]

Parallel universes

I parallel universes exist, is it possible to communicate or travel between them? —Preceding unsigned comment added by 79.76.164.210 (talk) 22:03, 14 September 2008 (UTC)[reply]

Well, it depends what definition you give to "universe" and "parallel universe" really, then the answer is pretty much in the definition. If you define "universe" as "everything that exists anywhere", then by definition, there is no "parallel universes". You might want to check the article on existence here. Equendil Talk 23:29, 14 September 2008 (UTC)[reply]
You might find time travel[30] in the article Many-worlds interpretation stimulating. As if my mind doesn't branch enough, along comes your question, so I am stepping inside and may be gone for some time. Julia Rossi (talk) 23:26, 14 September 2008 (UTC)[reply]
Also see Multiverse; there are various ways and senses in which there could be multiple universes. --Allen (talk) 04:31, 15 September 2008 (UTC)[reply]

So, an aging rockstar has taken so much speed and cocaine over the decades...

...that it has somehow permanently altered his entire blood chemistry, meaning that a transfusion of normal, healthy blood would kill him - and turned his own blood so toxic that if transfused into another person, it would kill them instantly.

Urban legend/toughguy publicity posturing or genuine possibility? Not naming any names, a) due to WP:BLP, b) I've heard the same story attached to at least three different people. --Kurt Shaped Box (talk) 23:10, 14 September 2008 (UTC)[reply]

Definitely sounds like urban legend/pathetic boasting. Even people with a long history of heavy polysubstance abuse will clear the drugs from their blood within days. Obviously the addiction lasts longer, but their blood is not toxic per se (assuming no HIV, hepatitis C, etc of course). If they have liver, kidney, or other disease, then some things might be out of whack, but not enough to be fatal with the usual volume of a blood transfusion. --Scray (talk) 23:47, 14 September 2008 (UTC)[reply]
So, file alongside 'singer has pair of ribs removed to aid autofellatio' and 'singer hospitalized after swallowing 5 pints of semen' (hilariously, I can think of two singers who've had both these stories circulated about them)? --Kurt Shaped Box (talk) 00:18, 15 September 2008 (UTC)[reply]

Why? as a scientific question

Is "why" a valid question in science. For example, "Why is there a universe, rather than none?" or "Why does the apple falls?" or "Why does the atomic nucleus does not integrate?" etc.

Please provide examples from scientific papers and the academia.

Is "Why" a valid of question of science? —Preceding unsigned comment added by 98.97.102.6 (talk) 23:12, 14 September 2008 (UTC)[reply]

Yes, certainly. 'Why' asks for an explanation, and one of the purposes of science is to explain things. Exactly what constitutes a scientific explanation is a big topic in the philosophy of science. There's some information at philosophy of science#Scientific explanation. Algebraist 23:42, 14 September 2008 (UTC)[reply]
Why is the sky blue? Why do objects appear to fall straight towards the center of the planet? Why is there so much variety in species? Why is just fine. It might not always be answerable at the moment (like your "Why is there a universe", which is at the moment beyond our understanding), but that doesn't make them non-scientific. --98.217.8.46 (talk) 23:48, 14 September 2008 (UTC)[reply]
Perhaps how is a better-accepted scientific question in general, because it leads more directly to a mechanistic answer rather than a teliological or philosophical one. --Scray (talk) 23:51, 14 September 2008 (UTC)[reply]
Why? ;) --Kurt Shaped Box (talk) 00:04, 15 September 2008 (UTC)[reply]
Isn't there a school of thought in philosophy of science that would answer "no" to the OP's question, and say that science can only describe, not explain? I can't remember the name of that position or the people promoting it, but I'm pretty sure they exist. --Allen (talk) 00:46, 15 September 2008 (UTC)[reply]
Facts describe. Theories explain. That objects fall straight towards the earth is a straightforward descriptive fact. That they do so because of a mysterious force called gravity, or because that is the path-of-least-resistance in spacetime due to the warping of spacetime by the mass of the earth, is a theory. It is an explanation. Tentative, but legitimately scientific. --98.217.8.46 (talk) 01:00, 15 September 2008 (UTC)[reply]
I can't think of any such people offhand either, but one can normally find a philosopher willing to espouse any given position. Such people would, however, be opposed by the vast majority of philosophers of science and all practicing scientists. Algebraist 01:00, 15 September 2008 (UTC)[reply]
All practicing scientists seems extreme to me. In my experience, it seems like a lot of practicing scientists do their work with little attention to its philosophical underpinnings. And among those who think about the philosophy, I think at least some of them would try to construe their work as fundamentally descriptive, even if they use explanatory language as shorthand. --Allen (talk) 04:14, 15 September 2008 (UTC)[reply]
Well, it's the sort of doctrinaire sort of thing that certain philosophers and scientists might like to impose, but any attempt to demarcate what exactly is science and what isn't is going to run into a million practical problems (ergo the demarcation problem, which is no closer to being solved today than it was two hundred years ago). --98.217.8.46 (talk) 02:03, 15 September 2008 (UTC)[reply]
I'm pretty sure it was Richard Feynman who drummed this into my head - that science can tell us how the sky is blue, but not why. It's a subtle but important point - the mechanisms (the how) are much more tractable for research than the reasons (the why). --Scray (talk) 01:44, 15 September 2008 (UTC)[reply]
Except that linguistically speaking why is often about mechanisms as much as how is. And why does the apple fall to earth is much more straightforward and unambiguous than how is it that the apple falls to earth. --98.217.8.46 (talk) 02:01, 15 September 2008 (UTC)[reply]
If you look up why and how in a dictionary (I just did in 3 of them) I think you'll find why wrapped up in 'cause, reason, intention' and how equated with the 'manner' in which something falls. The latter is amenable to analytical approaches, whereas the former is not. I'll agree that the distinction has been blurred by common usage. --Scray (talk) 04:12, 15 September 2008 (UTC)[reply]
Richard Feynman also said [I wonder why]... manya (talk) 03:56, 15 September 2008 (UTC)[reply]
No, it is not valid at all. --proficient (talk) 04:50, 15 September 2008 (UTC)[reply]
It's okay to question, to propel inquiry, perchance to dream but doesn't guarantee there'll be valid answers. Julia Rossi (talk) 10:18, 15 September 2008 (UTC)[reply]
Yes, of course "why" is a valid question in science. Open-ended "why" questions have driven far more fundamental discoveries in science than closed-ended "how" questions. For example, to answer "how does an apple fall to Earth" you observe the paths of falling objects under various conditions and conclude that "a falling object experiences a constant vertical acceleration which is independent of its mass and of its velocity" (you might then go on to correct this initial conclusion to take account of drag forces and terminal velocity). This bit is easy - Galileo knew this in the 17th century.
But answering "why does an apple fall to Earth" in a scientific way is much harder. Newton's answer was "because each object in the Universe exerts a gravitational force on all other objects which is proportional to the product of their inertial masses". This then leads to "why is the force of gravity proportional to inertial mass" - as has been pointed out, answering one "why" question often leads to more "why" questions. The answer to this second "why" question had to wait for Einstein and general relativity. Then you have the question "why do objects even have inertial mass ?", and we are still trying to answer that question - this brings us to the Higgs boson and the LHC. If scientists had not been interested in the "why" questions we would not have advanced beyond Galileo's understanding of "how". Gandalf61 (talk) 10:44, 15 September 2008 (UTC)[reply]

September 15

Somewhere theres music

Somewhere thers music, how faint the tune
Somewhere theres heaven, how high the moon?
But exactly where is Heaven? —Preceding unsigned comment added by 79.76.225.5 (talk) 00:22, 15 September 2008 (UTC)[reply]

It's underneath Charing Cross station. DuncanHill (talk) 00:24, 15 September 2008 (UTC)[reply]
It is a place [31] where Fred dances forever with Ginger, and Louis sings forever with Ella [32]. Note that Ginger does everything Fred does, except backwards and wearing high heels. Edison (talk) 02:28, 15 September 2008 (UTC)[reply]
For Ginger's Heaven, wouldn't it be Fred who was in high heels and dancing backwards? Clarityfiend (talk) 04:11, 15 September 2008 (UTC)[reply]
And maybe having top billing. Julia Rossi (talk) 10:10, 15 September 2008 (UTC)[reply]
The mind is its own place, and in itself, can make heaven of Hell, and a hell of Heaven. --- OtherDave (talk) 12:22, 15 September 2008 (UTC), channelling Milton.[reply]

Triple point

Do all compounds have a triple point? Nadando (talk) 04:42, 15 September 2008 (UTC)[reply]

I don't believe helium does. Does that count as a compound or would that be just an element? --Bennybp (talk) 11:27, 15 September 2008 (UTC)[reply]

How do we know there will be no problems with the LHC?

If it created a black hole, wasn't it supposed to grow gradually? It's only been a week, so it might be only the size of a pinhole or so. I'm afraid in about a month or two from now one of those scientists might be saying "Hey, what's that black spot over there? Charlie, you've got to be more careful with that marker, this machine is expensive! Here, I'll wipe it off.....oh,oh." Dr. Carefree (talk) 05:03, 15 September 2008 (UTC)[reply]

First, the LHC hasn't collided anything yet, and it shouldn't operate with energies larger than of existing accelerators until next year.
Now, our current theories predict that black holes evaporate through Hawking radiation, and small black holes should evaporate faster. We have no experimental evidence of this, but we're pretty confident on it. See the article for more details. If we are correct, the black holes created by the LHC, if any, should evaporate almost instantly.
If the LHC created a stable black hole, it would be extremely tiny (in fact, smaller than Planck length), and it would have a terribly small - practically nil - gravitational pull. The only chance it could accrete more mass would be direct contact with other particles which, given its size and the fact atoms are 99,99% vacuum, it wouldn't happen very often. There are probably some quantum effects that need to be taken into account here, but I haven't read anything about it yet.
Such a black hole would fall immediately towards the center of the Earth and orbit it. It would probably take millions of years for it to be of any concern. But again, chances are no such black holes will be produced.
Also, we can be pretty confident the LHC is safe because we have natural phenomena (cosmic rays) that replicates the experiments we're doing, even with much larger energies. There's a counter argument you'll see around that these natural collisions could produce "fast black holes", and the LHC would be creating "stationary black holes". This is sort of true, but it wouldn't matter. Neutron stars have huge densities and gravitational fields, and such particles or black holes wouldn't get through them. The fact that there are ancient Neutron stars everywhere in the Universe strongly suggests that there is no threat from such collisions. — Kieff | Talk 05:28, 15 September 2008 (UTC)[reply]
Unless I got my math wrong, a black hole of 100kg evaporates in about 8e-11 seconds. A black hole on the order of 1000 proton masses would evaporate in 4e-79 seconds. In that time it could move less than 1.2e-68cm. Now that time is shorter than the Planck time, and the distance is shorter than the Planck length. Things don't get more "instanteneous" than that. --Stephan Schulz (talk) 10:31, 15 September 2008 (UTC)[reply]

views of Thuria and Cluros

Moons of Barsoom. NASA/JPL

Astronomy on Mars doesn't say: Other than transits, have the hurtling moons of Barsoom ever been photographed from the ground? —Tamfang (talk) 07:11, 15 September 2008 (UTC)[reply]

That question sounds suspiciously ER Burroughs'ish. Anyway, the answer is yes: [33]. Franamax (talk) 08:02, 15 September 2008 (UTC)[reply]

Hydrogen gas - analytical test???

Is there any analytical method to detect hydrogen gas? The pop test just tells qualitatively. can hydrogen gas be converted through cheical reaction so that it can be detected easily in lab scale.....? ....Venki —Preceding unsigned comment added by 218.248.46.116 (talk) 08:22, 15 September 2008 (UTC)[reply]

You could burn it like in the pop test, and find a way to get the mass of the water. From there, it's stoichiometry. It would be tricky, since the burning would vaporize the water, and would tend to shoot it all over the place. --Bennybp (talk) 11:30, 15 September 2008 (UTC)[reply]

Why Electromagnetic Wave is Sine Wave?

There are many kind of waves such as saw wave, square wave, triangle wave etc. But why electromagnetic wave is always depicted as sine wave? How to prove that? - Justin545 (talk) 11:34, 15 September 2008 (UTC)[reply]

Hurricanes over warm water, meeting with freezing air over land, making snow?

Some of the rain from Hurricane Ike reached the middle of the U.S., and I recall one in Boston in 1991. So, I got to thinking during the windy remnants last night - suppose a hurricane or tropical storm hit in early November, while waters in the Atlantic Ocean were still warm, but while there was an unusually early cold snap and lows were below freezing over land. Would the foot of rain fall as maybe 10 feet or so of snow?

I'm guessing not, because after reading the article here, it seems like the water would have to be warm enough to warm the air around it to above freezintg - but that the interaction of warm and cold air would be such as to produce some *very* violent thunderstorms. However, I wanted to post the question here, too, to learn more and in case anyone else wondered.Somebody or his brother (talk) 12:17, 15 September 2008 (UTC)[reply]

LEDs or LASERs penetrating the body

Is there any LED or LASER that penetrates the body? —Preceding unsigned comment added by 117.196.163.244 (talk) 13:03, 15 September 2008 (UTC)[reply]

A pulse oximeter functions by shining a LED through the body (e.g. the fingertip). -- Coneslayer (talk) 13:10, 15 September 2008 (UTC)[reply]

Semiconductor diode

What happens to the current when you apply 0.7 volt to a semiconductor diode?

http://static.howstuffworks.com/gif/diode-graph.gif

The current reaches infinity or what?-Abhishek (talk) 13:24, 15 September 2008 (UTC)[reply]